Jump to content

Wikipedia:Reference desk/Humanities: Difference between revisions

From Wikipedia, the free encyclopedia
Content deleted Content added
Line 952: Line 952:


What is a proof of chastity, in this context? The link leads only to [[Chastity]], which doesn't explain. [[User:Marnanel|Marnanel]] ([[User talk:Marnanel|talk]]) 10:41, 3 November 2012 (UTC)
What is a proof of chastity, in this context? The link leads only to [[Chastity]], which doesn't explain. [[User:Marnanel|Marnanel]] ([[User talk:Marnanel|talk]]) 10:41, 3 November 2012 (UTC)

== Why do campaigns bother with internal polls? ==

Every major political campaign (Presidential, Senate and House AFAIK) has internal polling, polls which they pay for and get to choose whether to release. But why bother? If there's one thing that 538 and so on show us is that you can get all the information you need from publicly available polls which are common. Given the campaigns, especially the smaller ones need to spend every dollar they can on advertising and so on, why do they persist in wasting money on frequent internal polls, even when the public pollsters are providing the information for free (from their perspective)? They aren't even very accurate from what I've seen. [[Special:Contributions/86.163.43.112|86.163.43.112]] ([[User talk:86.163.43.112|talk]]) 11:26, 3 November 2012 (UTC)

Revision as of 11:26, 3 November 2012

Welcome to the humanities section
of the Wikipedia reference desk.
Select a section:
Want a faster answer?

Main page: Help searching Wikipedia

   

How can I get my question answered?

  • Select the section of the desk that best fits the general topic of your question (see the navigation column to the right).
  • Post your question to only one section, providing a short header that gives the topic of your question.
  • Type '~~~~' (that is, four tilde characters) at the end – this signs and dates your contribution so we know who wrote what and when.
  • Don't post personal contact information – it will be removed. Any answers will be provided here.
  • Please be as specific as possible, and include all relevant context – the usefulness of answers may depend on the context.
  • Note:
    • We don't answer (and may remove) questions that require medical diagnosis or legal advice.
    • We don't answer requests for opinions, predictions or debate.
    • We don't do your homework for you, though we'll help you past the stuck point.
    • We don't conduct original research or provide a free source of ideas, but we'll help you find information you need.



How do I answer a question?

Main page: Wikipedia:Reference desk/Guidelines

  • The best answers address the question directly, and back up facts with wikilinks and links to sources. Do not edit others' comments and do not give any medical or legal advice.
See also:



October 27

UK House of Commons trivia

Two questions:

  1. When do newly-elected MPs take office? Basically as soon as a winner is officially named by elections officials, or is there some set period (e.g. "Ten days after the election") between election and officially taking office?
  2. What's the salary for the Steward of the Chiltern Hundreds? Presumably there's something, or it wouldn't be good for resignation purposes; is it something like £1 per year?

I've looked around for both of these answers, finding such documents as this, but I wasn't able to find anything through Google or at UK House of Commons, Kissing hands, the Chiltern steward article, or anywhere else that I looked. Have to admit that these questions grew out of reading the thread just above this one. Nyttend (talk) 00:24, 27 October 2012 (UTC)[reply]

On Q. 1: I assume this is covered in Erskine May: Parliamentary Practice but I'm having trouble accessing an online version. Having worked in this part of the Australian bureaucracy, I can assure you that members of our lower house commence their service and are paid from election day, no matter how long it might take for the result in their division to become clear or be officially declared. SeeOdgers' House of Representatives Practiceat "Members' remuneration and entitlements":
  • A Member is paid salary and allowances from and including the day of the election, ...
See also further down the page at "Titles accorded to members":
  • A Member's status as a Member does not depend on the meeting of the Parliament, nor on the Member taking his or her seat or making the oath or affirmation. A Member is technically regarded as a Member from the day of election—that is, when he or she is, in the words of the Constitution, ‘chosen by the people’. A new Member is entitled to use the title MP once this status is officially confirmed by the declaration of the poll.
Australia has a Westminster system modelled after the UK's, so I can only assume it's the same there. -- Jack of Oz [Talk] 01:17, 27 October 2012 (UTC)[reply]
Right, I'm going to have a bash at the Chiltern Hundreds question. For those playing along at home, the post of Steward of the Chiltern Hundreds is asinecure used to permit members of the House of Commons to leave their posts. By tradition, MPs are forbidden to resign. This dates from a 1624 resolution that states that MPs have been entrusted to represent their constituencies, and are not at liberty to resign this trust. At this time, serving in parliament was seen much more as a duty than a privilege, so it was more necessary to 'force' MPs to serve.
However, under the 1701 Act of Settlement, no person who holds an Office of profit under the crown may continue to be an MP. Thus, any MP appointed to a position which entitles them to receive money from the Crown will cease to be an MP. In modern times, MPs use this loophole to resign, and MPs wishing to resign are appointed to either post of Steward of the Chiltern Hundreds or Steward of the Manor of Northstead. Appointments usually alternate between the two.
In terms of the actual 'salary' one receives for these roles, things are somewhat unclear. According to theEncyclopædia Britannica of 1911, "Up to the 19th century there was a nominal salary of 20S. attached to the post[of Steward of the Chiltern Hundreds]", but "There are no traces of any profits having ever been derived from the office [of Steward and Bailiff of the Manor of Northstead]". There's no mention of when in the C19th the salary was abolished.
The Mirror of Parliament, Volume 1 by John Henry Barrow (1839) says: "...a salary of 20l a year is attached to it, though the money is never received by any person accepting the office." (p.276). 20l (italic small-case "L") would be £20 in modern notation. Alansplodge (talk) 09:46, 28 October 2012 (UTC)[reply]
Isn't that 20/-? That is, 20 followed by a solidus, meaning 20 shillings or £1 in new money. See £sd. - Cucumber Mike (talk) 11:05, 28 October 2012 (UTC)[reply]
No, in the 19th century and before, pounds were signified by an italic "L" after the amount. See Old Bailey Proceedings Online; 4th February 1839. "...he had clothes which came to 2l. 4s. 6d., and 3l. 0s. 6d. in cash". Also Calendar of Treasury Books, Volume 8 - 1685-1689 "Petitioner has sworn before Baron Atkins that what has been recovered of the premises is only worth 5l. 10s. 0d. per an." There is some inconclusive discussion at Talk:Pound sign.Alansplodge (talk) 19:58, 28 October 2012 (UTC)[reply]
A new Steward of the Chiltern Hundreds used to cost £14 15/- in an inauguration fee, which at one point came from the profits on the land. There was also stamp duty of £2 on the writ of appointment. The sale of land led to the estates becoming much less valuable and the Government paid the fee (seeMagazine, January 1841 p. 43). There is a note in National Archives file E 197/1, the Crown book of appointments to the Stewardships from 1772 to 1847, which states "Fee of £16·15 on Appointment to the Stewardship of the Chiltern Hundreds discontinued according to Mr. Trevelyan's letter to me of the 22 Jan. 1840. R.B.A." The signature is presumably that of R.B. Adderley of the Exchequer Seal Office. Perhaps significantly, Trevelyan had only started work on the previous day. Sam Blacketer (talk) 11:53, 28 October 2012 (UTC)[reply]
According to the House of Commons Background Paper: Resignation from the House of Commons "Other such offices have been used for this purpose in the past, and some of them have carried duties and salaries: this is not the case today." This would suggest that the Stewards do not now receive any money following their appointments, nominal or otherwise.
I think the way this works is that by being appointed to one of these posts, MPs are entitled to receive the duties attached to the manors. The fact that, for at least a few hundred years, those duties have been £0.00 (or £0-0-0) is immaterial. It's the 'entitlement' to the profit that makes one inelegible to sit in the House. - Cucumber Mike (talk) 11:55, 27 October 2012 (UTC)[reply]
According to the Parliamentary fact sheet, "During the seventeenth century, a hundred years after any records of their actual administration cease, the office of Steward became divorced from any former actual duties, and ceased to enjoy any revenues from the area." So it's an "office for profit", but any profit has been stripped from it. It's the UK. It does not have to make sense ;-).--Stephan Schulz (talk) 12:58, 27 October 2012 (UTC)[reply]
On when a person elected becomes a Member of Parliament, elections are formally ordered by a Writ being sent from the Clerk of the Crown in Chancery to the Returning Officer in each Parliamentary constituency. The UK doesn't have any arrangement whereby absentee postal votes which arrive after polling stations close can be added to the count, so (unusually in a Western democracy) the count which takes place from 10 PM is the final result (save if it is challenged through an election petition). The official in charge of the election is called the 'Returning Officer' because the duty is that of making a Return to the Writ, which is to send back to the Clerk of the Crown the name and address of the person who has been duly elected. The Writ also formally tells the Returning Officer the date, time and place when the Parliament is summoned to meet. The Returns are all gathered together in a large white book and about ten minutes before the new Parliament meets, the book is formally handed over to the Clerk of the House of Commons. (SeeVotes and Proceedings for the record of this handover happening in the current Parliament.) Each person elected to Parliament then has to take the oath or affirm allegiance to the Crown in order to take their seat.
It is difficult to be precise about when a newly elected person has formally become a Member. It is certainly possible to argue that it is as soon as the Returning Officer has formally declared them elected. Against this it could be argued that no-one can be a member of a Parliament which has not yet been summoned, so it would be as soon as the Parliament had met. Members of Parliament are not paid unless they have taken the oath, but when they do take the oath their pay is backdated to the date their Return was received (see Briefing on Parliamentary Oath, p 8). However Members who have not taken the oath are still Members of Parliament and able to perform some functions. Sam Blacketer (talk) 13:23, 27 October 2012 (UTC)[reply]
I looked at two UK MPs chosen at random - Tony Blair and Patricia Hewitt. Blair was elected at the 1983 general election, held on 9 June. His own page tells me he was the Member for Sedgfield from, you guessed it, 9 June 1983. Hewitt was elected at at the1997 election, held on 1 May. Her page says she was the Member for Leicester West from 1 May 1997. It seems clear that UK MPs are members from election day or by-election day as the case may be. If that were not the case, we'd have some explaining to do about Blair, Hewitt et no doubt al. -- Jack of Oz [Talk] 21:02, 27 October 2012 (UTC)[reply]
That seems to be the answer to a subtly different question - "When should Wikipedia count a Parliamentarian's term as beginning?" It is custom and practice here as elsewhere to refer to the polling day in the general election. In fact only a very few fast-counting constituencies are able to produce a result in the two hours between the close of the poll and midnight. I have, though, undone an edit which used the fact that a byelection count had concluded after midnight as a reason for saying the successful candidate's term began on the following day. (There's more disagreement about when the term ends for an MP who is defeated: is it on polling day, or the day Parliament was dissolved? Practice seems to differ.) Sam Blacketer (talk) 23:52, 27 October 2012 (UTC)[reply]
The background paper downloadable from here talks about the Father of the House
  • The Father of the House heads the ‘seniority list’ of Members, a list of Members in order of their length of unbroken service. If two or more Members enter the House at the same election, each with unbroken service, their seniority is determined by the date and/or time they took the oath.
That is, for the purposes of determining who the Father of the House is, it may be necessary to look at when two or more members took the oath. But the implication is that, for general purposes, the term of service of an MP commences on their election day. -- Jack of Oz [Talk] 03:04, 28 October 2012 (UTC)[reply]
The disqualification attaching to offices of profit under the Crown is triggered if there is a possibility of payment through the office; it is not necessary for any actual payment to have been made. In 1955 there were a series of problems with Members of Parliament who had posts which were unpaid but where there was a theoretical entitlement to expenses; they were held to be disqualifying. As a result of the confusion the House of Commons Disqualification Act 1957 was passed which lists all the affected posts in a Schedule, so that there is no doubt. Sam Blacketer (talk) 13:23, 27 October 2012 (UTC)[reply]

diffrenciation and cost leadership

can a firm or business achieve diffrenciation and mantian a low cos leadership at the sametime? — Precedingunsigned comment added by 61.175.228.68 (talk) 01:19, 27 October 2012 (UTC)[reply]

Have a look at Differentiation (economics), Cost leadership and especially Porter generic strategies; does that help?184.147.123.169 (talk) 14:25, 27 October 2012 (UTC)[reply]
Generally, a differentiation strategy will employ advertising and other means, which will increase cost, over, say, generic alternatives. However, there may be exceptions. For example, once a company finds itself with a near monopoly, the economy of scale may allow them to advertise and still remain the lowest cost producer. StuRat (talk) 23:34, 27 October 2012 (UTC)[reply]

Hebrew english dictionary transliteration

Is there a website that is a Hebrew-english and English-Hebrew with English transliteration? I am trying to translate some English words into Hebrew like strong, might and union but I have hard time reading Hebrew that's why I need transliteration. — Precedingunsigned comment added by 70.31.22.30 (talk) 02:54, 27 October 2012 (UTC)[reply]

I google [hebrew transliteration online] and there seems to be a number of possibilities. ←Baseball Bugs What's up, Doc? carrots03:28, 27 October 2012 (UTC)[reply]
Have no idea about on-line, but in the dead-tree world, "Webster's New World Hebrew Dictionary" by Hayim Baltsan (ISBN 0-671-88991-5) is aimed at people like you... AnonMoos (talk) 05:59, 27 October 2012 (UTC)[reply]
WikiBooks has the beginnings of an elementary Hebrew course with transliteration here, but as of now it is woefully incomplete. The Foreign Services Institute has a very comprehensive course in Hebrew, with transliteration and accompanying recorded classes in mp3 format (around twenty hours of such material, by my tally) here. It's particularly useful in figuring out the particular Modern Israeli articulations of phonemes (especially vowels, which are by no means interchangeable with any of their English counterparts), something that is often neglected with the other gratis courses out there. Also, it's not exactly transliteration, but you might tryMilon and Foundation Stone (downloadable), both of which include audio files of Hebrew pronunciation. And if it's good-ol' Biblical Hebrew you're working with, most editions of Strong's I've seen have transliteration in one form or another. Evanh2008 (talk|contribs) 13:20, 27 October 2012 (UTC)[reply]
Those may be good resources, but if the original questioner wants to look up Hebrew words based on their modern Israeli pronunciations (ignoring the Hebrew alphabet), then the Baltsan book seems to be one of the few tools that allows this... AnonMoos (talk) 10:06, 28 October 2012 (UTC)[reply]
Google Translate is simple and easy if you want an ok translation. Otherwise, as someone noted above, just googling "Hebrew dictionary online" or "Hebrew translation online" will get tons of results. If you want specific words, feel free to ask me on my talkpage.
  • Strong = חזק (pronounced as chazak)
  • Might = עָצְמָה (pronounced as otz-mah. Note: if you're asking it as in "Might I go to the bathroom," it'd be האם, pronounced as ha-eem)
  • Union = איחוד (pronounced as ee-chood)
Hope it helps! --JethroB 00:37, 28 October 2012 (UTC)[reply]
To help a bit more, I'll indicate the stressed syllables in all caps and substitute letters I believe are closer to the Hebrew sounds: kha-ZAK, ots-MAH, ee-KHOOD - though in the _trade_ union context, I'd suggest ee-GOOD (rhymes with "see food"; Morfix online H>E dico gives "unification, merger, unity" foree-KHOOD). -- Deborahjay (talk) 18:44, 28 October 2012 (UTC)[reply]
איחוד is sometimes transliterated in writing as Ihud. --Soman (talk) 16:44, 29 October 2012 (UTC)[reply]

London geography - Primrose Hill

This question is related to some content from the Regent's Park page, but there is no content dispute, I am just curious about the facts. In fact, the reason I am asking here is to a large extent because it seems no one cares over at Talk:Regent's Park.

Primrose Hill is a hill in London. At some point content was introduced into the Regent's Park article to say:

"Primrose Hill is a Royal Park and belongs to the Sovereign along with all the other Royal Parks of the Crown Estate.

The supposition that Primrose Hill is owned and maintained by the Corporation of London is an error that has been the subject of successful Crown litigation in both in the High Court and Court of Appeal"

Now, I am wondering about the facts behind these statements because 1) Primrose Hill is not listed as a Royal Park in London, and 2) the second sentence seems a bit non sequitur - is there really a supposition that Primrose Hill is owned by the Corporation of London? And has the Crown really sued the City in the High Court and the Court of Appeal? --PalaceGuard008 (Talk) 15:01, 27 October 2012 (UTC)[reply]

I'll take the first bit: Whilst Primrose Hill isn't a separate Royal Park in and of itself, it is a part of Regent's Park. This is confirmed byCamden Borough Council: "Regent’s Park and Primrose Hill are operated by Royal Parks", bythis Royal Parks proposal for improvements to the summit and by the map of Regent's Park. I haven't found any evidence so far about the Corporation of London being involved, but these things sometimes get into the Wikipedia article by someone adding something they think they know, someone else reverting it, and the compromise being that the information is left in, qualified by 'it is often thought that...'. I'll keep looking. -Cucumber Mike (talk) 15:36, 27 October 2012 (UTC)[reply]
(EC) On 1), see THE REGENT’S PARK & PRIMROSE HILL OPERATIONS PLAN: January 2009 "The Regent’s Park and Primrose Hill are distinct but contiguous public parks jointly managed by The Royal Parks". However, on the Royal Parks website, there is no mention of Primrose Hill on the list on their Parkspage, however the accompanying map shows "The Regents Park and Primrose Hill" but when you click on the link, you only get details of Regent's Park.Alansplodge (talk) 15:42, 27 October 2012 (UTC)[reply]
Old and New London: Volume 5: CHAPTER XXII PRIMROSE HILL AND CHALK FARM (1878) says; "In 1827, the provost and fellows of Eton (who owned Primrose Hill at the time) began to see that their property would soon become valuable, and they obtained an Act of Parliament (7 Geo. IV., c. 25, private), enabling them to grant leases of lands in the parishes of Hampstead and Marylebone. Soon after the accession of Queen Victoria, endeavours were made to obtain Primrose Hill for the Crown, and a public act was passed (5 and 6 Vict., c. 78), for effecting an exchange between Her Majesty and the provost and college of Eton. By this act Eton College received certain property at Eton, and gave up all their rights in the Hill." Still searching for mention of litigation. Alansplodge (talk) 16:05, 27 October 2012 (UTC)[reply]
Regent's Park and Primrose Hill by Martin Sheppard (2010) says that Primrose Hill was acquired in 1841 because of a proposed scheme to turn it into a cemetery (p.97). No mention of any court case though, I suggest a "Citation needed" tag would be in order. Alansplodge (talk) 16:39, 27 October 2012 (UTC)[reply]
Thanks for the informative answers. On the first question then, it seems that Primrose Hill is managed as part of Regent's Park even if it geographically might not be within the Park's boundaries. Primrose Hill is not listed in our list in the Royal Parks of London article, but I am also wondering where the original author derived that list of the Royal Parks from - it cites section 22 of the Crown Lands Act 1851, but that statutory provision also includes many other pieces of land other than the ones listed in the Royal Parks of London article, including Primrose Hill. Perhaps someone could come up with a way to reconcile the two lists? --PalaceGuard008 (Talk) 15:07, 29 October 2012 (UTC)[reply]
Since I still haven't found the details of the alleged court case, I'll just mention that one way in which the Corporation of the City of London isinvolved with Primrose Hill is over the subject of the Protected views: nothing is allowed to be built such that it would block the view of the dome of St Paul's Cathedral, or of the Palace of Westminster, from the top of Primrose Hill. Consideration is also given to the backdrop: there are restrictions on what may be built behind the buildings mentioned as well.document has more details. - Cucumber Mike (talk) 19:58, 27 October 2012 (UTC)[reply]
Thanks for that interesting cite. Definitely a tag needed on the second sentence. --PalaceGuard008 (Talk) 15:07, 29 October 2012 (UTC)[reply]

U.S. election timings

Hey all. I'm looking for an updated version of http://news.bbc.co.uk/1/hi/world/americas/3760822.stm (for any timezone) but my google-fu just isn't up to strength. Does anyone know where I can find one? Or are the timings all going to be the same? Also, that page equivalates "polls close" and "result declared" --how good an approximation is that likely to be? I'm trying to organise a social event around the elections, you see :) Thanks, -Jarry1250 [Deliberation needed] 15:09, 27 October 2012 (UTC)[reply]

I would assume that the timings are going to be very close to the 2004 values (here's a less-readily-useful but updated2012 reference). As for "how good an approximation" -- no one can really say. Only a few swing states are really in play, so none of the west coast states in the late brackets are likely to matter (in terms of dramatic uncertainty). Nate Silver of fivethirtyeight currently estimates a 50/50 chance that Ohio will determine the election -- so on that basis, Ohio closing at 0030 UTC may be the highlight of the evening. If it's a landslide, that might be clear within 30 minutes (and good enough for pundits to declare X the all-but-inevitable winner). On the other hand, votes might be close enough to trigger a repeat of the 2000 election and take weeks to resolve, or any other timetable in between. — Lomn 18:44, 27 October 2012 (UTC)[reply]
(ec) [1] The results of many races will be able to be reliably predicted by news networks soon after the polls are closed (or even before the polls close, based on exit polling. In closer races, the information will take several hours as the different precincts report there numbers (maybe up to 12 hours, but most probably within 3-6 hours). However, most news networks will continue to give a blow-by-blow account as different results come in, so it could still be interesting (if you find that sort of thing interesting). If the race is close enough to trigger arecount, then the results could take a long time. If it's for president, as in the Florida election recount, they'll probably still try to get a president sworn in by January. If it's for a lower office, as in the United States Senate election in Minnesota, 2008, it could go significantly longer.Buddy431 (talk) 18:55, 27 October 2012 (UTC)[reply]
(ec) Yes, Buddy431's version of the link is substantially better. My advice, after the above caveats, is that 0000 - 0300 UTC is probably the sweet spot. If Romney has not made substantial progress in the Obama-leaning swing states of Virginia, Ohio, Wisconsin, and Colorado (all of which close by 0200), Obama probably wins, even with the remaining states yet to close (as those are the last of the large swing states). If Obama takes Florida (the largest Romney-leaning swing state) and Ohio, that's also probably good enough to call it his way. — Lomn 19:03, 27 October 2012 (UTC)[reply]
Thanks all. If I called it an evening at, say, 03:45 UTC, then, the general consensus is that I'd go to bed confident of the result? -Jarry1250 [Deliberation needed] 11:11, 28 October 2012 (UTC)[reply]
Not if it goes like it did in 2000. In fact, CBS projected Bush the winner in Florida, and then had to backtrack as the situation became clearer (or muddier). It was a good stretch of time before they got the Florida thing figured out. And it could happen again. ←Baseball BugsWhat's up, Doc? carrots16:59, 28 October 2012 (UTC)[reply]
Yes, barring a goes-to-recount result in a critical state in an otherwise-close election, I think the answer will be in hand by 0345. — Lomn 22:33, 28 October 2012 (UTC)[reply]
Fair enough. I guess you'd both agree that extending to (say) 04:30 UTC probably would not increase the chance of knowing greatly? Thanks! -Jarry1250 [Deliberation needed] 16:21, 30 October 2012 (UTC)[reply]

Freemasonry and homosexuality

My question is whether or not Freemasons accept homosexuals as member of their Lodges. Thank you. Iowafromiowa (talk) 19:02, 27 October 2012 (UTC)[reply]

A google search for "freemason homosexual membership" is instructive, provided you filter past the conspiracy-theorist results. The forums associated with AskAFreemason suggest "it depends on the lodge". This story from Lexington, KY, notes a vote to dismiss a proposal that would have banned gay members for that state. The presence of a vote at that level suggests to me that the initial link is on the right track: it depends on the lodge. — Lomn 19:08, 27 October 2012 (UTC)[reply]
Freemasonry, as an institution, accepts homosexuals. There is no Grand Lodge that has a ban on homosexuals being initiated. However, election to the fraternity takes place at the individual lodge level... so: "It depends on the individual lodge" is the correct answer. I personally know several several Freemasons who are openly homosexual. They joined lodges that either did not care about a candidate's sexual orientation, or actively sought out gay men. However, I also personally know several Freemasons who are openly homophobic. They tend to belong to lodges that consist of other homophobes... lodges that would reject an openly gay men (or even a man who the members suspect is gay).
It also can depend on what area of your state you are in. Freemasonry is a cross section of society in general. In a town or city where broader society is accepting of homosexuality, the local Masonic lodge will tend to reflect that societal acceptance. In a town or city where broader society is not accepting, the local Masonic lodge will reflect that lack of acceptance. Blueboar (talk) 20:46, 27 October 2012 (UTC)[reply]

Dividends and stock price

When people know upfront that some company is paying dividends, and I suppose it's almost always announced, how does that influence the price? I thought at first that the price would increase shortly afterbefore the payment day, and fall after that, but it seems that it follows a different logic. How does it work?Comploose (talk) 21:06, 27 October 2012 (UTC)[reply]

I'm not sure why you would think that. To me it seems more logical that the price should be highest shortly before the dividend payout, because there is going to be a very quick cash return. In most cases, though, the dividend is not a large enough fraction of the stock price to have a major short-term effect one way or the other. Looie496 (talk) 21:39, 27 October 2012 (UTC)[reply]
Sorry, I meant before. BTW, some dividends are higher than 5%, isn't that much? Comploose (talk) 22:49, 27 October 2012 (UTC)[reply]
Yes, but most dividends are paid quarterly, and the difference between getting 2% tomorrow and 2% three months from now is not all that much.Looie496 (talk) 03:40, 28 October 2012 (UTC)[reply]
The difference between getting the 2% tomorrow or in three months would be huge, if you could re-use your money for obtaining other 2% the day after tomorrow. But, things don't work like that. The expectation of getting 2% dividends for your stock is balanced by any other existent negative expectation. And these negative expectations exist if a company is paying dividends, since that's only due to the fact that there are good reasons to expect a negative outcome. That's way dividend paying stock is not getting up before they pay and down after. OsmanRF34 (talk) 17:52, 28 October 2012 (UTC)[reply]
The price will change on the announcement depending on how the announcement compares with market expectations. The price immediately before the announcement will be based on the expected dividend. If the actual dividend is larger, the price will go up. If it is smaller, the price will go down. (This is the same as you see following profit announcements - if profits are higher than expected, the price goes up, if they are lower, the price goes down - this is why it is common for a price to go up following news of a massive fall in profits, it's just that the fall wasn't quite as massive as people were expecting). The price will then drop by the amount of the dividend on the day the stock goes "ex-dividend", meaning the day it is determined who gets the dividend (if you sell your stock just before the ex-dividend date, they get the dividend, if you sell it just after, you get it - it will actually be paid a few days after that). It is clear that the price must drop, since the total amount of wealth of the shareholders can't change because of a purely financial action (nothing has happened that could actually create new wealth). --Tango (talk) 01:01, 29 October 2012 (UTC)[reply]

Write-in Candidates

One of the earlier threads about the upcoming election in the US made me think about write-in candidates. Being from a country where such a thing would never be allowed I'm just curious as to how that works. In particular I'd like to know how the people who count the votes determine who was meant by the people who wrote in their choice. It probably has no relevance in the real world but imagine this scenario: John Doe was never a candidate for any office but thousands of people write him in on the ballot (by the way, how do you write a name in on those ballots that are actually punchcards?). So John Doe gets the majority of the votes and thus becomes mayor-elect (or whatever office we're talking about). The John Doe everybody had in mind now says he doesn't really want to be mayor and declines the office. Since there is no shortage of John Does in the City, some other John Doe goes up there and says: "My name is John Doe and John Doe was elected mayor and I'd be happy to assume that office." How do you legally stop him becoming mayor?--Zoppp (talk) 22:23, 27 October 2012 (UTC)[reply]

See the second paragraph of our article on Write-in candidates. I've copied it below.-gadfium 23:06, 27 October 2012 (UTC)[reply]
"Some jurisdictions require write-in candidates be registered as official candidates before the election. This is standard in elections with a large pool of potential candidates, as there may be multiple candidates with the same name that could be written in."
But not all jurisdictions require that. As I noted in the other thread, Pennsylvania has no such requirement. And while the scenario that Zopppdescribed is unlikely in a city, in a small community it does happen. In my town a few years ago, a man was elected to borough council by write-in vote. No one was running for the office, but there were several dozen write-in votes. Each person written in had one vote, except for one man who had 3 votes. No one had ever even campaigned for the seat. But since he had the most votes, he was on council.    → Michael J    03:15, 28 October 2012 (UTC)[reply]
This issue doesn't just affect write-in candidates: [2] AlexTiefling (talk) 12:05, 29 October 2012 (UTC)[reply]



October 28

Did some of the world war 2, Korean War and Vietnam soldiers lie about their age and are any of soldiers who lied are alive today?

I'm wondering if some the ww2 , Korean war soldiers and Vietnam soldiers lied about their age and they were 17 or 16 they lied and they lied said they were 18. I could be wrong about this but back then they didn't have photo ID like today so it was easy to lie about their age. A birth certificate was probably needed, but if the person could pass for the required age, they could get in. I think also the country was eager to get any volunteers they could to protect and fight for their country. They did this in ww1 but what about ww2, Korean War and the Vietnam war and are any those soldiers who lied are alive today? 00:08, 28 October 2012 (UTC)

The Veterans of Underage Military Service says it has 29 active members who served in the US military or merchant marine at age 13, plus others older. Clarityfiend (talk) 00:47, 28 October 2012 (UTC)[reply]
This February 2012 article gives Walter Holy's account of his enlistment at the age of 15 and service with the paratroopers. Clarityfiend (talk) 01:00, 28 October 2012 (UTC)[reply]
You'll need a pretty blind recruiting sergeant to let a 13 years old pass as 18. OsmanRF34 (talk) 01:18, 28 October 2012 (UTC)[reply]
Well, yes, but then even today minimum enlistment age in the US is 17 (with parental consent), not 18. And boy seamen were allowed into the Royal Navy at age 14, though I don't know the equivalent in the US. --Stephan Schulz (talk) 01:31, 28 October 2012 (UTC)[reply]
There are some pretty big 13 year olds.[3] Clarityfiend (talk) 05:09, 28 October 2012 (UTC)[reply]
And some young-looking 18 year-olds. Alansplodge (talk) 09:02, 28 October 2012 (UTC)[reply]

Maybe he's lying today to you and he's not a vet of the Korean war OsmanRF34 (talk) 18:00, 28 October 2012 (UTC)[reply]

I wonder about how long ago the OP thinks the Vietnam War was! The youngest American veterans (18 in early 1975) are only 55 years old now. I doubt there are many reliable sources about the youngest Vietnamese veterans. Roger (talk) 18:18, 28 October 2012 (UTC)[reply]
The Korean War is not the Vietnam War. --140.180.252.244 (talk) 19:57, 28 October 2012 (UTC)[reply]
See Dan Bullock "...the youngest American serviceman killed in action during the Vietnam War." He joined the Marine Corps at 14 and was killed at An Hoa Combat Base aged 15. Alansplodge (talk) 19:34, 28 October 2012 (UTC)[reply]
See also My Father's Story by Crissandra L. Turner, which says of Edward Turner Jr. "In 1953 he sneaked off and joined the army Airborne using a cousin’s name at the age of 14, making him the youngest Korean War Veteran from the State of Missouri." Alansplodge (talk) 20:12, 28 October 2012 (UTC)[reply]
The movie Too Young the Hero is the true story of Calvin Graham, a 12-year-old who joined the Navy during World War II. 69.62.243.48 (talk) 00:26, 29 October 2012 (UTC)[reply]
My grandfather lied to get into WW2 at 17, but that was near the end of the war. My partner's grandfather joined at 16 for WW2 and for years never collected his military pension because he was afraid they'd find out & strip him of his rank. His family finally talked him into it, and the government didn't care one whit that he'd entered too young. — The Hand That Feeds You:Bite 22:32, 30 October 2012 (UTC)[reply]

how much cropland is lost to circle farming?

duplicate question, new answers should go at thescience desk
The following discussion has been closed. Please do not modify it.

how much cropland is lost to circle farming? Naively, we might assume that 2r*2r is the square equivalent versus pi * rsquared is the circle equivalent. But that would mean 4 rsquared versus 3.141 r squared - i.e. "almost a third" more area is actually available (1.27x more) if they farmed squares instad of farming circles. Is this right though? Because you don't need clearance of a full square around the circle, you can pack the next circle in slightly more closely. I'm asking about this.

http://www.google.com/search?q=circle+farming&tbm=isch

how much cropland is lost to circle famring instad of square farming? --89.132.116.35 (talk) 13:54, 28 October 2012 (UTC)[reply]

answers moved to science desk

Are there any black people in Greenland living there?

Are there any black people in Greenland living there? Neptunekh94 (talk) 19:03, 28 October 2012 (UTC)[reply]

I would assume that there are black people stationed at Thule Air Base. Although that may not be what you are talking about (as the service men living at the base there are not permanent residents). Blueboar (talk) 20:54, 28 October 2012 (UTC)[reply]
88% of Greenland's population is Inuit, and the Inuit are kind of black. Do they count? --140.180.252.244 (talk) 22:47, 28 October 2012 (UTC)[reply]
They're only black in the very vague and unhelpful sense of 'not European'. They're not (generally) part of the African Diaspora, although like Clarityfiend, I did immediately think of Matthew Henson's descendants, who are an exception to this. AlexTiefling (talk) 11:55, 29 October 2012 (UTC)[reply]
There are descendents of Matthew Henson, who are also Inuit.[4] Clarityfiend(talk) 22:52, 28 October 2012 (UTC)[reply]

Tete-Michel Kpomassie, 1981, An African in Greenland. — kwami (talk) 23:06, 28 October 2012 (UTC)[reply]

US capital gains tax

I'm curious about the definition of "capital gains tax" in the U.S. How does the IRS treat the income made by partners in a professional partnership like a law firm? Does it treat it like dividend income (and hence subject to a lower rate) or is it considered ordinary income like a salary? Thanks. — Preceding unsigned comment added by94.99.100.234 (talk) 20:10, 28 October 2012 (UTC)[reply]

In a partnership, income generally retains its character, and the distribution to partners (or failure to do so) is a non-event for tax purposes. This has led to controversy in the case of carried interest, when distributions to a hedge fund manager are treated as capital gains, even though the manager is receiving the distributions because of its provision of professional services to the partnership. John M Baker (talk) 21:02, 28 October 2012 (UTC)[reply]


October 29

Presidential candidates not elected by their home state

How many times has a presidential candidate not been elected by either their home state or the state they previously ran in as a senator/governor/representative? — Preceding unsigned comment added by 98.125.190.85 (talk) 02:13, 29 October 2012 (UTC)[reply]

Here you go:
That's the complete list I got by combing the full list of Presidential elections. --Jayron32 03:12, 29 October 2012 (UTC)[reply]
Al Gore lost his home state in 2000. It's not clear from the question if only eventual winners, or if the user is asking about all the candidates. RudolfRed (talk) 03:32, 29 October 2012 (UTC)[reply]
In that case, the list is likely a LOT longer. In several elections, major candidates came from the same state, so only one could win that state. If the OP is interested in expanding the list to include all of the significant candidates, even those that lost, they can do what I did: Start in 1787, check each page in turn, and create their own list. --Jayron32 03:48, 29 October 2012 (UTC)[reply]
John Kerry lost the state he was born in too. Hot Stop (Edits) 03:52, 29 October 2012 (UTC)[reply]
And, of course, Mitt Romney is likely to lose both his birth state of Michigan, due to his opposition to bailing out the auto companies while supporting Wall Street bailouts, and Massachusetts, due to his move from the center to the right (shown by his rejection of "Obamacare", even though his "Romneycare" was quite similar). See election prediction map here: [5]. StuRat (talk) 04:09, 29 October 2012 (UTC)[reply]
Of course General Motors was not bailed out, it was seized and its creditors not paid in abrogation of the bankruptcy laws. As for his "supporting" the Wall Street bailout, how did he vote on that in congress? μηδείς (talk) 16:18, 30 October 2012 (UTC)[reply]
In 1972 George McGovern lost his home state of South Dakota, winning only Massachusetts and the District of Columbia. Duoduoduo (talk) 15:55, 29 October 2012 (UTC)[reply]
Every "minor party" candidate for a long time now ... —Tamfang (talk) 00:53, 30 October 2012 (UTC)[reply]

Mass shootings and suicides

In several mass shootings, at the end, the perpetrators commit suicide (if they aren't killed by the police first). The question is, when planning the shootings, do such perpetrators already plan to make the day of the shooting their last day on Earth, or do they initially not want to kill themselves and only later do so out of remorse or so that they would not be caught? That is, do they plan to commit suicide after the shooting from the start, or do they only decide to do so when they're about to be caught? And in a related question, did the perpetrators of the Columbine High School and Virginia Tech massacres plan to commit suicide from the start or did they do it out of remorse or convenience? Narutolovehinata5 tccsdnew 03:29, 29 October 2012 (UTC)[reply]

I am not sure how many have given interviews, but one could write them and ask.... μηδείς (talk) 03:41, 29 October 2012 (UTC)[reply]
They could've kept diaries or documents where they said what they plan to do. --Jethro B 03:46, 29 October 2012 (UTC)[reply]
Yes, several have left web pages and such, which announced their intention to commit suicide after their killing spree. StuRat (talk) 04:11, 29 October 2012 (UTC)[reply]
And were their killing sprees becasue they hated life/depressed & thus the suicide, or was the suicide b/c they had no other option after the killing spree, other than being arrested? --Jethro B 06:21, 29 October 2012 (UTC)[reply]
For those who said they planned on committing suicide, I'd say most fall into the first category. After all, if they wanted to live, they would have planned a way to get away, as many murderers do. StuRat (talk) 06:27, 29 October 2012 (UTC)[reply]
You've basically got two cohorts, the in-cold-blood alienated children and loners who may have a manifesto, and the crime-of-passion adult who snaps due to a firing or a breakup or the like. I can't think of any instance among the latter group where a plan was found afterwards. μηδείς (talk) 16:14, 30 October 2012 (UTC)[reply]

Quinzaine Coloniale, 10 novembre, page 667

What source is this book referring to? "Voir la Quinzaine Coloniale, 10 novembre, page 667"--KAVEBEAR (talk) 03:40, 29 October 2012 (UTC)[reply]

La Quinzaine Coloniale is a periodical (ISSN: 11491833) published from 1897 to 1923 every two weeks by L'Union Coloniale Française. Some volumes are availables on the site of the BNF (Bibliothèque Nationale de France) here. Are you looking for this page (10 November 1902, page 667)? — AldoSyrt (talk) 12:03, 29 October 2012 (UTC)[reply]
Thanks.--KAVEBEAR (talk) 06:24, 4 November 2012 (UTC)[reply]

Survey revealed men have "a bit too much" sex

I'm looking for a very specific scientific article about this survey where a researcher found out based on his calculations that the men in a (tribe?) village are having proportionally a lot of sex per day. In reality the men of the village didn't know the guy was a researcher and so instead of giving accurate information they exaggerated their sexual activity. I can't remember the country, the name of the researcher or any dates, but I do know it's often cited as an example of a survey gone wrong. Can anyone fill me in with the details? --109.246.247.215 (talk) 08:29, 29 October 2012 (UTC)[reply]

While I didn't find details of this particular example, I would note the accuracy of what participants tell reseachers is usually a concern in any research which relies on such things and particularly so in cases involving intimate stuff like sexual activity. See [6] for example. Nil Einne (talk) 13:52, 29 October 2012 (UTC)[reply]
This question reminded me of Margaret Mead, who was accused by some of having been duped by some Samoan local's "jokes" into believing something different from the truth. But that wouldn't fit the OP's description of "his calculations". HiLo48 (talk) 16:44, 29 October 2012 (UTC)[reply]
Margaret Mead focused on the lives of teenage girls in Samoa anyway... AnonMoos (talk) 02:28, 30 October 2012 (UTC)[reply]
Note also somewhat reenforcing my earlier point, as highlighted in our articles covering the controversy, there's question of who was actually duped. Some suggest it was those who claimed to have found that the people she interviewed lied were in fact generally the ones duped. Nil Einne (talk) 06:03, 30 October 2012 (UTC)[reply]
In some past decades I was intermittently keeping track of the Derek Freeman controversy, and it seemed then that Freeman was fairly viciously anti-Mead, but hadn't been able to do much more than establish that by focusing on teenage girls, Mead had a somewhat narrow view of Samoan society as a whole (which is no great surprise -- though on the other hand, in anthropology before the 1970s, anthropologists were rarely considered to have a narrow view of a society when they talked almost exclusively with high-ranking adult males...). To judge from the Derek Freeman article, little has changed since then. AnonMoos (talk) 13:36, 30 October 2012 (UTC)[reply]

Is there a point in the Ocean which is the most remote place of all Oceans?

Thank you. Iowafromiowa (talk) 11:49, 29 October 2012 (UTC)[reply]

Yes - Point Nemo, or the oceanic pole of inaccessibility.
I believe it's possible to prove that on a finite globe with bounded coastlines, such a place must exist. AlexTiefling (talk) 11:54, 29 October 2012 (UTC)[reply]
Theoretically, if there were only two circular land masses on exactly opposite sides of the world, then there would be a ring of inaccessibility, not a point. You can also imagine scenarios with multiple points. StuRat (talk) 16:50, 29 October 2012 (UTC)[reply]
As far as human settlement goes, Edinburgh of the Seven Seas is apparently the most remote. 192.51.44.16 (talk) 01:58, 31 October 2012 (UTC)[reply]

Why do the Brits call soccer football?

Thank you. Iowafromiowa (talk) 11:54, 29 October 2012 (UTC)[reply]

A better question would be why American football is called "football" - the name implies a game primarily played with the feet. You'll find more at Names_for_association_football. --Dweller (talk) 12:00, 29 October 2012 (UTC)[reply]
Here's the short short version:
  • At first, there was just football, but there were hundreds of different varieties, whereby every village and town in the UK and US played different sports called football. The only commonality was that there was a ball that had to enter a goal area, and the game was played on foot (this was to distinguish it from games of similar rules played on horseback (c.f Polo). They all had a wide variation in rules. Some only allowed the ball to be kicked. Some allowed the use of hand, some not, some allowed the ball to be caught, some not, some allowed running, some not, some allowed throwing, some not, etc. etc. The first 1869 Princeton-Rutgers game was played under these conditions: the two teams used the set of rules played under Rutgers rules. A later game played at Princeton used their rules. Dartmouth had a game called Old division football. Over time, by the late 1800's, two distinct "codes" of football began to form: Association football, which did not allow the use of hands, and Rugby football, which did. This happened in the U.K. and Canada before it happened in the U.S. During the first few decades of organized football in the U.S. no uniform set of rules developed until the 1874 game between McGill and Harvard. Many of the other Northeastern American schools had settled on a code of football similar to soccer, but Harvard played a local game called the "Boston Game" that allowed players to handle the ball with their hands. Since the other schools didn't, Harvard had a hard time scheduling games against American teams, so they worked out a game against McGill University, a school from Montreal. Being Canadian, they had already adopted Rugby as their uniform code, so they taught the game to Harvard. Harvard fell in love with Rugby, and taught it to Yale. After working out some tweaks to the Rugby game (called the "Concessionary rules") Yale and Harvard would spread the Rugby-style game to other American schools. Over time, an annual rules convention in Springfield, Massachusetts developed to tweak the rules each year. These conventions (which eventually grew into the NCAA) would become dominated by Walter Camp who introduced a series of rules changes which changed the game from essentially Rugby football to a different sport entirely. The last major rule change to create American football was the 1905 change which allowed the forward pass. Canada, independent of the U.S., developed their own form of football which eventually adopted some similar rules.
So, there you go. American football and Canadian football both descend from rugby football, which was one of the two codes of football to become standardized in the nineteenth century. That's why they're all called football. --Jayron32 12:31, 29 October 2012 (UTC)[reply]
Rugby football has a fair amount of kicking in it, but also a lot of non-forward passing. American football likewise used to have more kicking in it, but the forward pass diminished the need for the kick as a game strategy. You still occasionally see a quick punt on situations other than 4th down, known as a "pooch punt". There's still plenty of kicking in American football, just not as much as there once was. And it's worth pointing out that the all-time scoring leaders in American football tend to be... quess what... kickers. ←Baseball Bugs What's up, Doc? carrots01:37, 30 October 2012 (UTC)[reply]
Yeah, but to be fair, kicking is not considered as vital a part of the game as other parts. Kickers are usually considered fairly interchangeable, and undervalued by those inside of football and those that cover the sport in the media and serious history. The Pro Football Hall of Fame has four players who played kicker for a significant portion of their career, but two played significant time at other positions (QB-K George Blanda and RB-K Paul Hornung) and were not cited specifically as kickers. Of the other two, only Jan Stenerud played kicker exclusively (Lou "the Toe" Groza also played at offensive tackle). There isn't a single punter in the pro football hall of fame, though several lists of "Who's not in the HOF but absolutely should be" have Ray Guy at the top of such a list. Kicking is an important part of American Football, but it isn't clear that kickers are. --Jayron32 01:47, 30 October 2012 (UTC)[reply]
"Football" is one of those words, like "corn", which is applied to the most common variety in a particular location. Where wheat is the most common grain, "corn" is wheat. Similarly for barley, oats, maize etc. Where soccer is the most popular game, people call it "football", and call other football games by other names. Similarly for American football, the various versions of rugby, Gaelic football, Australian rules football, and so on. The question realy ought to be, why do so many Americans seem think that what they call "corn" and "football" are the only things those words can mean? --Nicknack009 (talk) 13:15, 29 October 2012 (UTC)[reply]
Getting off-topic here, but the bit about corn is wrong. Americans don't call corn "corn" because it's the "most common grain". For all I know it may well be the grain that we grow the most of, by total tonnage, but we grow plenty of wheat as well, and if you ask an American to think of a grain, I would expect him to think of wheat, or maybe even rice, before corn. In American English, the word "corn" simply means what you call "maize". (The word "maize" here is usually understood to refer to "Indian corn" — brightly-colored hard-kernel corn used as a decoration around Thanksgiving.) --Trovatore (talk) 01:29, 31 October 2012 (UTC)[reply]
What we call "corn" was originally called "Indian corn", to distinguish it from other types of corn (maybe you've heard the term "barleycorn", for example). As "Indian corn" came to be commonly called just "corn" in America, the term "Indian corn" narrowed to the decorative corn you're referring to. ←Baseball Bugs What's up, Doc? carrots04:21, 31 October 2012 (UTC)[reply]
Yeah, that's history worth pointing out. I'm aware that corn in UK usage means, basically, "grain". But I think that's as specific as it gets. What Nicknack009 is missing is that that's not what it means in the States. In the UK, unless I'm mistaken, even if wheat is the most important local grain, corn still means "grain" and not specifically "wheat". (Here's a test: Potato bread, oat bread, then normal bread is ---- bread? I'm pretty sure the answer in the UK is not cornbread.)
In the US, on the other hand, corn means one thing specifically, not grain in general. This is a qualitative difference, and Nicknack009 is mistaken in his claim that the US usage is just an instance of a general pattern. --Trovatore (talk) 09:01, 31 October 2012 (UTC)[reply]
The real question is why do so many non-Americans seem to think that Americans don't know that words mean different things in different places. --Jayron32 13:57, 29 October 2012 (UTC)[reply]
Because of questions like the OP, and a 'World Series' in which a tiny fraction of the world competes? AlexTiefling (talk) 14:02, 29 October 2012 (UTC)[reply]
I think the real question is why all these non-Americans seem to think they're right. We invented this language! U-S-A! U-S-A!Lomn 14:32, 29 October 2012 (UTC)[reply]
What do you call an elevator with a group of intelligent, softly-spoken people inside? A lift. Ankh.Morpork 14:45, 29 October 2012 (UTC)[reply]
If American English was good enough for Jesus, it's good enough for me! —Tamfang (talk) 18:29, 31 October 2012 (UTC)[reply]
Since fewer than three in 10 think it important to know the locations of countries in the news and half don't know where India is?[7] Ankh.Morpork 14:38, 29 October 2012 (UTC)[reply]
Show me the equivalent version of that question for your country of preference. Can we quit all being so brilliantly baited by such obvious troll questions? Shadowjams (talk) 05:38, 31 October 2012 (UTC)[reply]

For some REAL football, see the Royal Shrovetide Football which seems to have been played in the town of Ashbourne since 1667. Here's an exciting moment from last year's game. Alansplodge (talk) 15:45, 29 October 2012 (UTC)[reply]

It's not just the Brits. It's a lot of Europe and even Asian countries. --Jethro B 16:01, 29 October 2012 (UTC)[reply]

Not to mention most of the countries on the American continent and Africa, all using their own native equivalent of "football". So it is basically most of the world. --Saddhiyama (talk) 16:17, 29 October 2012 (UTC)[reply]
However, the question was about the use of the word "football", hence the Anglo-centric nature of the discussion. Alansplodge (talk) 21:38, 29 October 2012 (UTC)[reply]

The article Names for association football highlights the fact that what Americans call soccer has many names around the world. In Australia, there are four different professional games called football by at least some of their fans - Australian Football, Rugby League, Rugby Union and Soccer. (They're in alphabetical order, in case anyone wants to take offence!) American Football and Gaelic Football are played at an amateur level. The fight by some Soccer fans to demand that their game be only called football is a very aggressive one. Those with a lot of time on their hands may be amused by Talk:Soccer in Australia#Requested move. Not one of the nicest nor rational debates ever on Wikipedia. HiLo48 (talk) 16:36, 29 October 2012 (UTC)[reply]

Why is the national team called "Socceroos" instead of "Footieroos" then? -- AnonMoos (talk) 02:14, 30 October 2012 (UTC)[reply]
As our article mentions until 2005 the organisation responsible was known as Soccer Australia or Australian Soccer Association. In 2005 it renamed itself to Football Federation Australia in line with general international usage of the word football. So the nickname arose at the time when the organisation itself used the word soccer, hence isn't particularly surprising. Changing it now is unlikely (plus to be blunt anything you can come up with foot and roo in it is likely to sound dumber then socceroos). Nil Einne (talk) 05:50, 30 October 2012 (UTC)[reply]
The name chosen for the national league operated by Football Federation Australia, the A League, avoids the issue of the name of the sport completely. HiLo48 (talk) 01:18, 31 October 2012 (UTC)[reply]

Presidential children and Proms

Did Susan Ford (who I think had her prom at the White House) or Caroline Kennedy wear dresses or female tuxedeos to their Proms. It was the mid-1970s, the height of Womens Liberation, so I'm just curious. Paul Austin (talk) 14:20, 29 October 2012 (UTC)[reply]

A simple search for 'Susan Ford prom' finds [8] which mentions her being in a jersey-gown. A simple image search for the same terms finds plenty of images like [9] showing her in her prom dress. Caroline Kennedy's prom presuming she had and attended one must have been a long time after her father's death given her age so I suspect was a more private affair (and searches aren't helped by various references to other stuff) Nil Einne (talk) 16:16, 29 October 2012 (UTC)[reply]
I doubt Caroline Kennedy would have worn a "female tuxedo". For one thing, the "female tuxedo" - known more accurately as Le Smoking - was ten years out of style by that time. For another, Kennedy attended Concord Academy, a very old-fashioned school, and it seems vanishingly unlikely bordering on unimaginable that school authorities would have allowed a girl to wear a suit like that. (Yes, "allowed" - schools had draconian dress codes back then.) Looking at the dates, by the way, it's very possible that she didn't even attend her school prom, as her stepfather died in March of that year. --NellieBly (talk) 05:09, 31 October 2012 (UTC)[reply]

Accelerated depreciation disincentive

From page 5 of [10] I have questions about this:

"Corporations complain that the corporate income tax discourages investment, but with interest deductible and with accelerated depreciation (relative to what economists call “true economic depreciation”11) it may be that the tax system actually is biased the other way.12"

The corresponding footnotes are:

"11. True economic depreciation would allow firms only to deduct the reduced value of the asset that results from usage, obsolescence and aging.
"12. That is, the cost of borrowing is reduced in the same proportion that returns are reduced."

My question is, can someone find a good source (preferably peer reviewed, secondary, or both) which elaborates this point about deductible interest and accelerated deduction? I can find many sources, but they aren't very scholarly. Thanks for any and all help with this. 64.134.60.65 (talk) 15:37, 29 October 2012 (UTC)[reply]

Biblical Inerrancy

How many people in the world believe that the Bible is inerrant or written without error? 140.254.226.217 (talk) 19:24, 29 October 2012 (UTC)[reply]

What type of errors do you mean? Are you referring to grammatical errors/cantillation errors as a result of passing on throughout the years, or errors since the inception? --Jethro B 19:28, 29 October 2012 (UTC)[reply]
A huge issue is if you take the Bible literally or figuratively. So, when it says the universe was created pretty much in it's current state in one week, does this literally mean seven 24-hour periods ? If so, this conflicts with just about every branch of science, so not many believe in that. However, if you take it figuratively, where a "day" can be billions of years, far more believe in that. StuRat (talk) 19:32, 29 October 2012 (UTC)[reply]
If you want to include fictional characters, we have Ned Flanders: "I literally believe in every bit of the Bible, even the parts that conflict with the other parts." :-) StuRat (talk) 19:34, 29 October 2012 (UTC) [reply]
All inconsistencies in holy scripture are only apparent. Explaining them away is intellectual entertainment for reasonably sophisticated believers. Unfortunately, these are increasingly rarer nowadays. --Stephan Schulz (talk) 19:40, 29 October 2012 (UTC)[reply]
Well in Judaism, these "inconsistencies" are determined to be consistent as analyzed in the Talmud and lengthy debates, but that's not relevant to Christianity or Islam (which are far more than 1% of the world's population). --Jethro B 20:08, 29 October 2012 (UTC)[reply]
Expanding on the distinction StuRat lays out, there's also "inerrant theology", which says little about the story aspect of the text. Specifically, in addition to "literal 7 day creation" and "figurative 'days' are 'billions' creation", there's also the "the point is, God saw that people were 'very good' creation". The literal interpretation is probably the easiest to define, but I'd be wary of assuming that arbitrary sources (like the one below) mean 'literal' when they say 'inerrant'. — Lomn 21:06, 29 October 2012 (UTC)[reply]
From [11], about 31% of US Americans. Don't know about other countries, but you can leave out the 2/3 of the world that are not Christian, and America is of course quite religious. I got this from googling, btw. IBE (talk) 19:48, 29 October 2012 (UTC)[reply]
That's hardly fair to the 1/6 of the world that's Muslim that believes in its literally inerrant scripture--far outnumbering such Christians. μηδείς (talk) 23:22, 29 October 2012 (UTC)[reply]
Do we know that all nominal Muslims so believe? —Tamfang (talk) 00:48, 30 October 2012 (UTC)[reply]
We can't make windows into men's souls, but we do know that the central message of Mohammed was that the prior books had become corrupted, and that he was personally given the literal word of Allah to correct the situation. There is certainly no waffling on that from the major sects like the Sunni and the Shia, and it's the reason for the uproar over The Satanic Verses. Catholicism, on the other hand is far from literalist, and literalism is a minority stance among Protestants, but it's growing. μηδείς (talk) 16:09, 30 October 2012 (UTC)[reply]
I don't think one needs to be a literalist to find the idea offensive that your prophet got revelation from the devil and couldn't tell it apart from divine revelation. (though of course you are correct about the mainstream view of scripture in Islam) - Lindert (talk) 19:14, 30 October 2012 (UTC)[reply]
(after ec) The original question was about the Bible, though. Mainstream Islamic belief is that the Hebrew scriptures and the New Testament have become corrupted over time, and do not represent the inerrant word of God. As to exactly how literally to take the Qu'ran - well, there's a diversity of opinion about that, too, although I agree that it tends more to the literalist end than the vast majority of Christians' or Jews' views of the Bible. AlexTiefling (talk) 00:53, 30 October 2012 (UTC)[reply]
But see Theistic evolution, the reconciling of evolution with Christianity. The leadership of the Roman Catholic Church do not subscribe to creationism. See Catholic Church and evolution. ""In his encyclical Humani Generis (1950), my predecessor Pius XII has already affirmed that there is no conflict between evolution and the doctrine of the faith... Today, more than a half-century after the appearance of that encyclical, some new findings lead us toward the recognition of evolution as more than a hypothesis." John Paul II, 1996. For Anglicans, the Archbishop of Canterbury recently said that ""creationism is, in a sense, a kind of category mistake..." (he likes to talk in riddles, but I think he's against it). In the UK, a recent poll showed that 17% of the population believed in "Young Earth Creationism" which I think is surprisingly high. I've only ever met one adult who believed this - the rest probably keep quiet about it. Alansplodge (talk) 21:31, 29 October 2012 (UTC)[reply]
I suspect that the survey results would greatly depend on how you pose the Q: "Do you literally believe everything in the Bible is true ?" ... "Sure !" ... "So, then you believe the universe was literally created in 7 days ?" ... "Heck no !". Thus, for many, their belief in Biblical inerrancy depends on their Biblical illiteracy. StuRat (talk) 21:36, 29 October 2012 (UTC) [reply]
Six days, heathen! —Tamfang (talk) 00:48, 30 October 2012 (UTC)[reply]
StuRat -- the many who have held to the "day-age theory" didn't think they were denying the truth of the Bible at all. AnonMoos (talk) 02:04, 30 October 2012 (UTC)[reply]
The key is the word "literal". If you believe a "day" is something other than 24 hours, then you believe in a figurative, not literal, interpretation of the Bible. StuRat (talk) 00:40, 1 November 2012 (UTC)[reply]
However, a significant number of day-age theorists did not consider themselves allegorical Euhemerists, but rather restorers of the original meaning of the text, in that they thought that the Hebrew word יום yom could have several meanings. It may seem logical to you that day-age theory is inconsistent with Biblical literalism, but some people have held to both... AnonMoos (talk) 02:12, 1 November 2012 (UTC)[reply]
So far this discussion has concentrated on Old Testament stories, but the Jesus bits require a bit of faith too. Apart from the basic stuff of Son of God and the resurrection, there's the loaves and fishes, walking on water, and healing the sick, etc. Mostly impossible from a scientific perspective, so belief in inerrancy would require some faith. HiLo48 (talk) 22:54, 29 October 2012 (UTC)[reply]
Well, faith is a fundamental requirement of Christianity, or indeed, any faith. That's why they're called "faiths" as well as religions etc. There's no point adhering to a religion if every single thing they teach is scientifically provable; if that were the case, it would just be some branch of history or science or anthropology. The very nature of these organisations puts them outside any of those mainstream disciplines. That in itself is theoretically OK, because people are not technically compelled to believe or follow a single thing they teach. There have been some rather notable exceptions to this theory, which has made for extremely muddy waters. -- Jack of Oz [Talk] 00:43, 30 October 2012 (UTC)[reply]
Religion would not be religion, if it is scientifically provable. Science is good, but there may be something more out there - something unquantifiable or immeasurable - that belongs in the realm of philosophical discussion or religious inquiry. Since religion caters to subjectivity, emotions, and experience, I am quite sure that they are not provable by science at all. We may study emotion, experience, and subjectivity, but we may also seek to value them as they have their own intrinsic worth. 140.254.226.198 (talk) 15:04, 30 October 2012 (UTC)[reply]
Within Anglicanism, there is a strand of liberal theology that suggests that literal belief in the all of the New Testament is not a requirement of Christianity. Its most famous proponent. Dr David Jenkins, formally the Bishop of Durham is quoted as saying; "To believe in a Christian way, you don't necessarily have to have a belief that Jesus was born from literally a virgin mother, nor a precise belief that the risen Jesus had a literally physical body,"[12] Of course, to many Christians, even Anglican ones, this is an anathema. Alansplodge (talk) 18:34, 30 October 2012 (UTC)[reply]
At the other extreme, there are millions of people who never give the scriptural nuts and bolts of Christianity any thought at all, but still consider themselves to be Christians because, to them, being a Christian means always being kind and courteous and decent and polite and "nice" and never swearing and never speaking out of turn and always paying your bills on time and never speeding. -- Jack of Oz [Talk] 19:48, 30 October 2012 (UTC)[reply]
I know someone who routinely speeds, and STILL consider himself a Christian! HiLo48 (talk) 21:42, 30 October 2012 (UTC)[reply]
In the letter to the Romans, Paul begins with a discussion of the state of humanity before the possibility of salvation through faith in Jesus. So, I suppose the "salvation by faith alone in Jesus and not by works to achieve salvation" thing is scriptural, and through that faith (which includes repenting old sinful ways) brings good works such as the neat stuff you mention. Sort of. 164.107.182.63 (talk) 21:12, 30 October 2012 (UTC)[reply]

North Korean official executed by mortar

Is Kim Il-ch'ŏl the same Kim Chol who was destroyed in spectacular fashion for not properly mourning the late Kim Jong-il? If so, he was pretty old. 205.156.136.229 (talk) 19:50, 29 October 2012 (UTC)[reply]

I don't think so. Kim Il-ch'ŏl's article says he was retired from all offices in 2010. On the other hand, he was born in 1928, 1933 and 1941 (until I found a reliable source), so who knows? Black is white, and up is down in that country. Difficult to say. An article says the executed man was a "vice minister of the army", while Kim Il-ch'ŏl, while "first deputy chief of the People’s Armed Forces Ministry", was later made head of the ministry, so you'd think he'd be described differently. However, Kim Il-ch'ŏl had previously displayed a drinking problem in public in 2000.[13] Clarityfiend (talk) 21:34, 29 October 2012 (UTC)[reply]
I'm reverting to my previous opinion. According to the Chosun Ilbo, Kim Chol was vice minister at the time of the alleged offense, and Kim Il-ch'ŏl was retired in 2010. Clarityfiend (talk) 04:51, 30 October 2012 (UTC)[reply]
There is also Kim Chol-man who's even older but from the sound of it also retired [14] Nil Einne (talk) 05:41, 30 October 2012 (UTC)[reply]
You probably know this already, but our Kim (Korean name) article says that 21.6% of Korean people have the surname Kim, Gim or Ghim. Alansplodge (talk) 19:01, 30 October 2012 (UTC)[reply]

Is Shinto animist or polytheist?

--168.7.230.21 (talk) 21:20, 29 October 2012 (UTC)[reply]

An examination of the animism and polytheism articles suggests that can be considered to be both. Regards, TransporterMan (TALK) 21:39, 29 October 2012 (UTC)[reply]
It's generally considered animist, although there may be people who consider animism to be polytheist. --Jethro B 21:46, 29 October 2012 (UTC)[reply]
In essence it`s animist, but there are a couple of more or less well defined deities as well so it has some polytheist tinge to it. 164.71.1.221 (talk) 02:52, 31 October 2012 (UTC)[reply]

Voter turnout based on polls

Has there been any research on how polls affect voter turnout? I'd think that if a likely voter hears that his party is way ahead, then it decreases the likelihood that he'll bother to vote. I'm interested in any data that demonstrates this, or refutes it. (I'm aware of the time zone problem the US had in the 1984 election and earlier with regard to exit polling, and I don't care about that for the purposes of this question.) Thank you! Tarcil (talk) 23:34, 29 October 2012 (UTC)[reply]

Do you mean polls on the day itself, or just a feeling ahead of time about how comfortable the result is likely to be? If you're excluding 1984, etc, I assume you mean the latter, so let's have a look at the voluminous literature on how perceptions of closeness affect turnout.
In support, Geys (2006) presents some evidence of a link, although it's not clear that he controlled for enough variables in his study. Likewise, one of the big things thought to indicate this effect is that PR systems (which have no "safe seats") seem to have higher turnout than majoritarian systems that do though it is not clear why (Blais 2008; Blais and Dobrynzynska 1998). On the other hand, it turns out to be really hard to pin this down. For example, in the 1997 British Electoral Survey, we see that seats with a 0-10%, 10-20% and 20-30% margin of victory all had similar levels of turnout, mostly because Conservative voters in the UK seem to turn out regardless of how safe a Conservative seat it is they're voting in (there's a better correlation among Labour voters). My lecturer adds that the "relationship between district marginality and turnout is even weaker in the US and Canada" though he doesn't provide a citation for that.
One might ask: if the closeness of the election does not affect turnout, why do we see higher turnout in the elections presumed to be close like the 2010 British General Election? The answer is probably because voters perceive greater difference between the parties in these close-run elections and/or become better informed about those differences, both of which have been statistically linked with higher turnout (see BES 1997 for example). You might argue that the publication of polls showing a close result contributes to those two effects and thereby boosts turnout. Anyway, I hope that gives you some pointers about the variables involved here, if I did indeed interpret your question correctly. - Jarry1250 [Deliberation needed] 00:27, 30 October 2012 (UTC)[reply]
Great answer and thank you! Tarcil (talk) 23:21, 30 October 2012 (UTC)[reply]
A poll released shortly ahead of the European Parliament election, 2004 (Sweden) gave the June List slightly above four-percent threshold for the first time in a national poll. In the election they won almost 14.5% of the vote. In this case the tripling between the poll and the subsequent election was presumably a consequence of people adjusting their behaviour based on the poll. Gabbe (talk) 07:25, 30 October 2012 (UTC)[reply]
Why wouldn't it be just as likely that it's an inaccurate poll? There is a lot of noise in polling. Tarcil (talk) 23:21, 30 October 2012 (UTC)[reply]
In electoral systems with an election threshold, you basically throw your vote away if you vote for a party that receives fewer votes than the threshold. The June List had been formed four months ahead of the election, and all polls ahead of the election indicated that they would get fewer votes than the threshold. That is, until a poll released a few days before the election, in which they received more than 5%.[15] According to this report, 69% of the June List's voters made their decision to vote for the party in the final week ahead of the election. Gabbe (talk) 08:07, 1 November 2012 (UTC)[reply]

October 30

When did the US and the UK become allies?

Comploose (talk) 00:17, 30 October 2012 (UTC)[reply]

See United Kingdom–United States relations. A common interpretation is that they were neutral from 1815 to the 1880s, and became "allies" sometime between then and the First World War (depending on your definition of allies). HTH, - Jarry1250 [Deliberation needed] 00:29, 30 October 2012 (UTC)[reply]
According to chapter 5 of The Rise and Fall of the Great Powers by Paul Kennedy, the U.K. offered several concessions to the U.S. ca. the early 1900s (on the Alaska boundary dispute, fisheries and the Panama Canal) as part of a general policy of reducing colonial frictions to allow for greater diplomatic freedom of action in Europe. The U.S. and U.K. weren't militarily allied until 1917... AnonMoos (talk) 01:47, 30 October 2012 (UTC)[reply]
Note that after World War I, although there were close economic ties, the US was careful to keep itself aloof from any military alliances. The US didn't join the League of Nations, which was dominated by Britain and France. US military planners even had plans for war with Britain, see War Plan Red. Although Britain and France were favoured over the Axis at the start of World War II, they still made us pay top dollar for the arms they were sending us (I believe that we had to pay up front in gold bullion). See British Purchasing Commission. Later on there was Lend Lease and help with convoy escorts, but the US didn't enter the war against Germany until Hitler declared war. Alansplodge (talk) 10:56, 30 October 2012 (UTC)[reply]
Well that's being a bit technical. Franklin Roosevelt had a bit of a problem with the U.S. Congress (and public too) at the time, but Pearl Harbor changed everything. Although it made the case for war in Europe much stronger when Hitler, somewhat inexplicably, declared war on the U.S. after it declared war on Japan. But don't mistake American isolationism post-WWI for hostility towards Britain. Lend Lease can hardly be seen as anything except the U.S. helping the Allied powers. Nevermind the next ~6 years. Shadowjams (talk) 05:29, 31 October 2012 (UTC)[reply]
Also note that the US opposed the UK (and France and Israel) during the Suez Crisis. StuRat (talk) 02:55, 31 October 2012 (UTC)[reply]
It is worth noting that the definition of "ally" is not "blindly endorses everything the other does". The U.S. - U.K. alliance does not preclude them disagreeing on major issues. --Jayron32 05:31, 31 October 2012 (UTC)[reply]
(ec)The U.S. opposes All three of those countries quite often on lots of things. That's hardly here-nor-there when answering that question. But maybe the better question is when's the last time the U.S. and Britain have directly been in conflict, and/or indirectly (diplomatic maneuvering hardly qualifies). Yes, what jayron says exactly. Shadowjams (talk) 05:32, 31 October 2012 (UTC)[reply]
I believe that the last time that the U.K. and U.S. fought a pitched battle was the Battle of New Orleans, though Britain did economically support the South in the Civil War, see Cotton diplomacy, and Union blockade which notes the unofficial support of the South by the British in defiance of the embargo. There were probably some naval skirmishes between privately-manned British ships and Union ships during the Civil war. --Jayron32 05:44, 31 October 2012 (UTC)[reply]
Britain hasn't fought Russia since the Crimean War, but it doesn't mean that we're allies. Alansplodge (talk) 10:37, 31 October 2012 (UTC)[reply]
Agreed. Being allies is more than not being at war with each other. The US actively opposed the UK during the Suez crisis, so they were not allies during this period. StuRat (talk) 19:36, 31 October 2012 (UTC)[reply]
I disagree with that final statement. Allies don't cease being allies just because of a different point of view over a major issue. If the UK had not supported the invasions of Iraq or Afghanistan, that would not have meant they were no longer allies of the USA. -- Jack of Oz [Talk] 19:46, 31 October 2012 (UTC)[reply]
I agree with what you said, but "not supporting" is not the same as "actively opposing". StuRat (talk) 20:24, 31 October 2012 (UTC)[reply]
The UK and US were both signed the North Atlantic Treaty on 4 April 1949, which bound us to come to each other's aid if either one were attacked, whether there was a diplomatic dispute in progress or not. So in strictly military terms, the answer to the question is:-
1) 1917 to 1919
2) 1941 to 1945
3) 1949 to the present
Any other definition of being "allies" is, as can be seen above, rather debatable. More recent sicking sticking-points in US / UK relationships can be found at Special Relationship Alansplodge (talk) 21:23, 31 October 2012 (UTC)[reply]
Sicking-points = vomitoria?  :) -- Jack of Oz [Talk] 22:01, 31 October 2012 (UTC) [reply]
Doh! Alansplodge (talk) 17:37, 1 November 2012 (UTC)[reply]

Two questions about the Japanese military

1. Does the Japan Maritime Self-Defense Force use metric or imperial units for navigation?

2. In WWII, do IJN pilots carry swords during sorties? A8875 (talk) 00:24, 30 October 2012 (UTC)[reply]

For Q.1, I presume they follow the standards here. Otherwise, Japan appears to be metrified. Mingmingla (talk) 00:57, 30 October 2012 (UTC)[reply]
Then what about outside of navigation? For short distances would they use yards or meters? A8875 (talk) 01:32, 30 October 2012 (UTC)[reply]
They would use metres, as yards are a completely alien concept and they would have no reason to adopt British measuring systems. They currently use the metric system. KägeTorä - (影虎) (TALK) 07:44, 30 October 2012 (UTC)[reply]
Do you have a reference for that? A8875 (talk) 10:41, 30 October 2012 (UTC)[reply]
1957 KägeTorä - (影虎) (TALK) 13:06, 30 October 2012 (UTC)[reply]
See Japanese units of measurement; "From 1924, the shakkanhō system was replaced by the metric system..." So it seems that they went straight from their own indigenous measures (of Chinese origin), to the metric system, without ever using Imperial or US measures. However, the International Nautical mile of 1,852 metres was adopted as an international standard in 1929, so I imagine that the Japanese naval forces navigate in nautical miles and metres. I can't find a direct reference for that at the moment. Alansplodge (talk) 13:03, 30 October 2012 (UTC)[reply]
Thanks, you two, but I'm asking about the JMSDF in particular. I am well aware that Japan is metricized but that has little to no bearing on what a specialized field use in specialized situations; consider how altitude is still measured in feet in most metricized countries. I'm looking for references specific to the JMSDF.A8875 (talk) 13:30, 30 October 2012 (UTC)[reply]
Why would altitude 'still be measured in feet' in a country which has never used feet as a measuring system? It's measured in metres. The Japanese Wikipedia page on Mt. Fuji gives the height of the mountain in metres. This is what we are trying to tell you. Japan is completely metricised, apart from measuring rooms in apartments by tsubo. KägeTorä - (影虎) (TALK) 14:34, 30 October 2012 (UTC)[reply]
KageTora, there's a bit of confusion between height and altitude. In aviation altitude is given in feet, see flight level. So Mt. Fuji would be in metres but Haneda Airport is at an elevation of 21ft. CambridgeBayWeather (talk) 04:52, 31 October 2012 (UTC)[reply]
Seems like the JSDF is exempt from the metrication laws[16][17]. In practice seems like most naval and aviation equipment uses US customary units[18]. Your contributions, however erroneous, are much appropriated, but please try to provide references next time. A8875 (talk) 16:24, 30 October 2012 (UTC)[reply]
However, our article, International Convention for the Safety of Life at Sea says; "In 1975 the assembly of the International Maritime Organization decided that the 1974 convention should in future use SI units only." Although warships are specifically exempted from these regulations, they would have to follow them when communicating with civilian vessels. This wouldn't rule out the use of nautical miles or knots, as these have been approved for use with the SI system. The links supporting the use of US Customary units quoted by A8875 above, appear to refer to the maintenance and calibration of US made equipment, rather than navigational use, which was the original question (I only found a machine translation of the third link, which talks about the size of American built helicopter rotors among other things). Please correct me if I've got the wrong end of the stick. Alansplodge (talk) 17:52, 30 October 2012 (UTC)[reply]
Japanese Naval Aviation Uniforms and Equipment 1937-45 (p. 56) says, "The only known exceptions to naval fighter pilots carrying a sword in their aircraft during flight would have been between late October 1944 and the end of the war, if a pilot were assigned to a kamikaze suicide attack mission." According to The Feel of Steel, "... in World War II, Japanese kamikaze pilots took their samurai swords into their cockpits in order to go to their deaths with honour." Clarityfiend (talk) 04:09, 30 October 2012 (UTC)[reply]
Aren't those two statements mutually contradictory? Unless "exceptions" is being used in an unusual sense? --PalaceGuard008 (Talk) 08:59, 30 October 2012 (UTC)[reply]
The first statement could be interpreted to mean they were only allowed to kamikazes. It sounds a bit odd to say that only kamikazes couldn't take swords. Unfortunately, my preview doesn't show the previous page, which might have cleared things up. Clarityfiend (talk) 10:07, 30 October 2012 (UTC)[reply]
I found a view of the previous page (55) which says; "Army officer pilots sometimes carried swords into their aircraft, as a symbol of authority... Navy pilots on the other hand, did not follow this tradition, and thus did not carry their swords in their aircraft for two specific reasons." (My summary:) 1) Not enough space in the cockpit of naval aircraft 2) The large piece of steel might affect the compass. Alansplodge (talk) 13:15, 30 October 2012 (UTC)[reply]

Support for Romney's healthcare plan

Can someone provide references that show thinktanks, organizations, fact-checkers, etc, that support elements (or all) of Romney's healthcare plan?

Thanks. --Jethro B 01:04, 30 October 2012 (UTC)[reply]

Do you want Republican party backed thinktanks, organizations, etc. or Democratic party backed thinktanks, organizations, etc.? --Jayron32 01:21, 30 October 2012 (UTC)[reply]
I'd prefer neutral organizations, but as long as it's reputable, I'm fine with that. --Jethro B 19:19, 30 October 2012 (UTC)[reply]
Do you mean the plan he passed as Massachusetts governor, or his current positions on federal government involvement in healthcare (which don't seem to add up to much of a coherent "plan")? -- AnonMoos (talk) 01:34, 30 October 2012 (UTC)[reply]
I'm referring to the policies he wants to implement as president, if he's elected. --Jethro B 19:19, 30 October 2012 (UTC)[reply]
If you ever find any specifics, the rest of the country would like to know as well. He's been very vague on how he plans to accomplish any of his goals (repeal the parts of Obamacare he doesn't like, "fix" Medicare, remove the deficit, lower taxes, etc.). — The Hand That Feeds You:Bite 22:42, 30 October 2012 (UTC)[reply]
I know, and that's why I turned here. But he's made claims like Obamacare removes $716 billion from Medicare and he'd put the $716 billion back in. So if there's some group that says "Our factcheck backs this up" or "our research backs this up," I'd be interested in that. That's just an example. I understand how tough this can be, normally I'd just google it myself but most of it seems to be negative (and I need positive in order to play devil's advocate). --Jethro B 22:52, 30 October 2012 (UTC)[reply]
Funny thing how Paul Ryan had the same $716 billion maneuver in his proposed budget, but that didn't keep Romney from choosing him as VP candidate. Maybe Romney should run anti-Ryan ads... AnonMoos (talk) 01:08, 31 October 2012 (UTC)[reply]
Undent. Kaiser is a reputable source on most things healthcare, but for our daily consumption Politifact gave Romney's $716 billion a "Mostly false". Politifact also told me Romney has said his plan would 'look most like' his VP's. It has a few points only, such as Medicare for younger-than-55s. Politifact ruled it untrue that, as advertised, most of those individuals would enjoy as good healthcare as members of Congress. Politifact ruled it true that the expenses would go up on average (I believe double), citing Kaiser which who said it was "because private plans have higher administrative costs and typically pay higher fees to providers than Medicare." [19] 83.108.141.146 (talk) 01:58, 31 October 2012 (UTC)[reply]
Thank you, but my question asked for support, not opposition. --Jethro B 03:51, 31 October 2012 (UTC)[reply]
This is politics. Romney himself has said that he will repeal Obamacare in its entirety, then on the very next day said he will keep children under their parents coverage til 26 and the ban on denying coverage of pre-existing conditions. Under no circumstances would it benefit a politician who wants to get elected to announce a specific plan--it would give his opponents something concrete to attack and would in Romney's case, if it were anything but the promised full repeal, confirm to the justifiedly skeptical base that he's a socialist squish. This is just like Obamacare, you are going to "have to pass the bill so that you can find out what's in it" in January. μηδείς (talk) 19:52, 31 October 2012 (UTC)[reply]
Even before the 2012 presidential election loomed, congressional Republicans have never come out with anything which could remotely be called a "plan" (i.e. specific policy proposals with accompanying numbers which add up) that would allow pre-existing conditions restrictions to be eliminated. Any such plan, if it wasn't "single payer", and retained a role for private insurance companies, would inevitably start looking a lot like Obama's bill. Certainly establishing individual health savings accounts as the main way of paying for insurance won't do anything about pre-existing conditions by itself (and in fact it could put people even more at the mercy of insurance company profit motivations -- not to mention stock-market fluctuations -- than today, unless accompanied by stringent government-imposed safeguards). Unfortunately, one prominent feature of Mitt Romney's campaign -- ever since Obama's "If we keep talking about the economy, we're going to lose" quote was maliciously taken out of context in November 2011 -- has been blatant out-and-out lying which goes significantly beyond ordinary garden-variety self-serving political spin: The Onion even wrote an spoof article about how Romney's strategy for turning around the campaign was lying more. If you feel comfortable that Romney will find a way to preserve the elimination of pre-existing conditions restrictions in that context, then by all means go ahead and vote for him... AnonMoos (talk) 21:00, 31 October 2012 (UTC)[reply]

Winner-Take-All - Part 2

I obviously spawned a lively discussion which was good by the way. I learned a lot which is what intended. But I am still curious about the second part of my question I asked above. What is the history of this winner-take-all policy enacted by most states? Has it been enacted since the beginning of the electoral college (seems unlikely to me but you never know)? Did it happen later? When did it happen? Did all of the states adopt it simultaneously or was it slowly one by one by one?70.58.0.141 (talk) 05:28, 30 October 2012 (UTC)[reply]

See Electoral vote changes between United States presidential elections and in particular the footnotes of the tables for info on states not giving all their electoral votes to one candidate, all the way back to 1816. These footnotes give the impression that splitting a state's electoral votes was the exception rather than the rule even back then. Duoduoduo (talk) 00:07, 31 October 2012 (UTC)[reply]

Postponing elections?

Given the damage being wrought by Hurricane Sandy in the USA at the moment, does provision exist for the Presidential election to be postponed to help with the clear-up? --TammyMoet (talk) 10:46, 30 October 2012 (UTC)[reply]

I don't know, but how would postponing it help with the clear-up? --Viennese Waltz 10:51, 30 October 2012 (UTC)[reply]
Postponing because many New Yorkers won't be able to vote next Tuesday might be more meaningful, specially for Obama, since these are millions of Democratic votes. OsmanRF34 (talk) 12:20, 30 October 2012 (UTC)[reply]
Voters from both sides will be equally inconvenienced by the storm. I don't see how it could disproportionally affect Democrats. Whether Obama wins NY by 1 vote or 1 million votes doesn't matter, since it's still 29 electors in the end. A8875 (talk) 14:10, 30 October 2012 (UTC)[reply]
They won't necessarily be equally inconvenienced. Generally there's a big urban/rural split in party affiliation, and in New York, urban/rural follows the Upstate New York/NYC split. Note the location of red versus blue in File:New York Presidential Election Results by County, 2008a.svg. Upstate, being further from the coast, (as well as things like generally less reliant on subway travel, etc.) will be less inconvenienced by the storm than city dwellers. Will that make enough of a difference to tip the results? Who knows, but probably not, as New York state polls solidly Obama, and even a moderate inconvenience to Democrats is unlikely to change that. - I'll also link the well-followed FiveThirtyEight blog on this topic [20] [21]-- 20:51, 30 October 2012 (UTC) — Preceding unsigned comment added by 205.175.124.30 (talk)
Unsurprisingly, this is a much discussed topic. A simple search for something like 'us postpone election sandy' will find plenty of results (some are indirect links) like [22] [23] [24] [25] [26] [27]. From those results, some states have existing provisions for postponing their elections, others would need new legislation. However the date for presidential elections are set by the US Congress to the Tuesday after the first Monday of November. Theoretically Congress could legislate for a new date but there isn't any real existing provision for postponement, in fact Congress has previously passed a resolution against postponing elections in the event of a terrorist attack. Note if the election dates are postponed this doesn't change the date when electors have to meet meaning you'll get less time for possible recounts etc. The general consensus is a postponement is unlikely although some measures like extended voting hours may be used. Funnily enough as per previous discussions there has been a big push for early voting, it seems that Sandy may be another push for future elections at the very least. Nil Einne (talk) 12:44, 30 October 2012 (UTC)[reply]
It's basically a case of "too bad". How elections are run is a state-by-state affair. You can bet there will be no support in congress for making any changes or allowances. New York may hold off on certifying its results for about a month as did Florida in 2000, but the votes have to be cast by election day. As said above, all voters of every party affiliation are equally inconvenienced in a district. And there is the time honored solution of cheating. μηδείς (talk) 16:01, 30 October 2012 (UTC)[reply]
"all voters of every party affiliation are equally inconvenienced in a district"- that's not true at all. Transportation and power are the issues. People with their own transportation and power generators (i.e. rich people) are much less inconvenienced. Poor people are more inconvenienced, and this affects one party more than the other. Staecker (talk) 17:33, 30 October 2012 (UTC)[reply]
Guess you've never heard of limousine liberals and have no familiarity whatsoever with NYC politics where the working class votes Republican, not the rich. In any case, getting yourself to the polls is your responsibility, no one else's. Michelle Obama herself said it:"You wake up on Election Day -- you might have a cold, babysitter gets sick, it's raining, the car broke down, I could go on -- toilet overflowing. There are so many ways to mess up a day when you don't have a lot of time," she said to laughter. The notion that the world is unfair because enough Democrats haven't voted yet is yet another leftist delusion.]

See also [28] , which in particular says

Federal law says that if a state fails to conduct an election for federal races on the day Congress chooses, the state legislature can pick a later date. But state and federal laws don't always jive perfectly. Virginia Gov. Bob McDonnell has said his state's laws don't grant him authority to reschedule the presidential election.

and

changing the date would wreak havoc for state and local elections also scheduled for Nov. 6. States might have to hold two separate days of voting, which could bust state budgets. Duoduoduo (talk) 21:05, 30 October 2012 (UTC)[reply]
It will be possible to hold elections virtually everywhere on Election Day. However, areas without power won't be able to use means of voting that require electricity. In these areas, photocopied paper ballots could be distributed, marked, collected, and counted manually, as was done in times past. Polling places are generally distributed densely enough in urban areas that (able-bodied) people can walk to them, and even in less urban areas it is possible to drive to one's polling place without crossing large bodies of water, so flooded tunnels shouldn't be an impediment. The only impediment would be rigidity and incompetence on the part of state and local authorities, and of course those could come into play. Marco polo (talk) 18:55, 31 October 2012 (UTC)[reply]
The US Constitution authorizes the Congress to set a uniform date for national elections. In an emergency, presumable the Congress could grant special dispensation to a state that might be unable to conduct its election on the normal day. ←Baseball Bugs What's up, Doc? carrots11:14, 1 November 2012 (UTC)[reply]

can the same person be chairman and president with a different ceo?

can the same person be chairman and president with a different ceo? Meaning, that this person actually operationally runs the show, answering to the CEO.

But the CEO, quarterly or once a year, actually answers to this person. Can this be done? — Preceding unsigned comment added by 91.120.48.242 (talk) 11:36, 30 October 2012 (UTC)[reply]

Absolutely The chief executive officer and chairman of the board and company president (often "chief operating officer") are separate positions within the usual corporate governance structure, though in many companies they are held by the same person. The board of directors is technically the group elected by the shareholders of the company to run the company in their interest, while the so-called "C-suite" (CEO, COO/President, CAO, CIO, etc.) is the group hired by the board of directors to actually manage the day-to-day operations of the company. So it works like this: The shareholders elect a board of directors, who hire the CEO and President and other C-level executives, and THOSE people then handle the operation of the company. There's nothing to prevent the board from hiring themselves into the management positions of a company (and many do so), but there's also usually nothing requiring it. Indeed, this is one of the difference between American-style corporate governance and that found in other parts of the world: In America, there is often considerable overlap between the board of directors and the C-level management of a company, in other places there are two distinct groups of people with non-overlapping membership: the supervisory board (which represents the shareholders) has a distinct membership from the executive board (which manages the operations of the company). In the U.S. it is usual (though not universal) that the same people serve both roles (The same person is both CEO and Chairman and sometimes President/COO as well). But they don't have to be. --Jayron32 11:55, 30 October 2012 (UTC)[reply]
Maybe I missed it, but I don't think that really answers the OP's question: The question presumes the rank ordering chairperson (1) > CEO (2) > president (3), and asks whether positions 1 and 3 can be held by one person (A) while position 2 is held by another person (B), so that B reports to A (CEO reports to chairperson) and yet A reports to B (president reports to CEO). Duoduoduo (talk) 17:06, 30 October 2012 (UTC)[reply]
The current Chairman and CEO of Coca-Cola, Muhtar Kent previously served as Chairman and President simultaneously, with a different CEO. So for at least a time, Coca Cola had the "1 and 3" positions held by the same person, with the "2" being a different person. See [29]. I have no idea what this meant for the practical aspect of corporate governance, but it has happened. --Jayron32 20:02, 30 October 2012 (UTC)[reply]

Questions about Ted Bundy

Is it true that he was a psychologist? and my second question is whether or not would it be possible for another Ted Bundy to exist now with all the technology in the criminal field. Thank you. Iowafromiowa (talk) 13:21, 30 October 2012 (UTC)[reply]

Marc Dutroux existed in the mid-1990s... -- AnonMoos (talk) 13:27, 30 October 2012 (UTC)[reply]
Robert Pickton's case is even more recent. AlexTiefling (talk) 13:43, 30 October 2012 (UTC)[reply]
The answer to your first question can be found from a careful reading of the early part of our Ted Bundy article; psychology was the only one of his several university courses he completed. He graduated in the subject from the University of Washington. But he never worked in the field - his work was as a political activist, and his continuing studies were in Law. AlexTiefling (talk) 13:53, 30 October 2012 (UTC)[reply]

And technology is no protection against a charming, trustworthy psychopath. alteripse (talk) 14:36, 1 November 2012 (UTC)[reply]

Journal Officiel des Établissement Français de l'Océanie

Where can I find "Journal Officiel des Établissement Français de l'Océanie" of August 30 to September 6, 1900? "Journal Officiel des Établissement Français de l'Océanie" of September 5, 1901? Besides this and this? Can somebody ask this question on the fr:Wikipédia:Oracle in French? --KAVEBEAR (talk) 13:22, 30 October 2012 (UTC)[reply]

Feel free to ask a question in English at the Oracle. Just say hello first and apologize for not being able to post in French. Lots of regulars there speak English. --Xuxl (talk) 15:26, 30 October 2012 (UTC)[reply]
Not easy to acess. For Journal Officiel des Établissement Français de l'Océanie of September 5, 1901, here is the link to the first page: http://www.archives.gov.pf/afficher_pdf.php?id_doc=/srv/www/htdocs/etatcivil/donnes/jopf//1901/JOPF_1901_page_00315.pdf, you have to change the URL for the next pages: JOPF_1901_page_00316.pdf, JOPF_1901_page_00317.pdf, etc. — AldoSyrt (talk) 18:28, 30 October 2012 (UTC)[reply]
Better, try (and retry if necessary) this link http://www.archives.gov.pf/jopf_liste_repertoire.php?dir=%2Fsrv%2Fwww%2Fhtdocs%2Fetatcivil%2Fdonnes%2Fjopf%2F%2F1901&order=name&asc=a. — AldoSyrt (talk) 18:44, 30 October 2012 (UTC)[reply]
Unfortunatly year 1900 is not archived on line on this site. — AldoSyrt (talk) 18:46, 30 October 2012 (UTC)[reply]
Wow that is amazing! Thanks. I will look over them.--KAVEBEAR (talk) 20:45, 30 October 2012 (UTC)[reply]
Where else could you find this? Is it in libraries in the US?--KAVEBEAR (talk) 20:59, 30 October 2012 (UTC)[reply]
Do you mean what Worldcat calls the Journal officiel with an author of "Etablissement français de l'Oceanie"? If so, the only US library owning it might be the New York Public Library. Go to Document Delivery, although it might help if you tried the Ask a Librarian feature first; your library may subscribe to some online database that includes it. Nyttend (talk) 01:25, 3 November 2012 (UTC)[reply]

Austro-Hungarian newspapers

As a RefDesk regular, I feel a bit weird being the one to ask the question. However, my historical researches have led me to Austria-Hungary, and in particular to Mureck, in Styria. I am interested in tracking down online versions of any Styrian, Imperial Austrian or (post-1867) cis-Leithanian newspapers. At one stage Google Books offered me what seemed to be a large bound collection of the Grätzer Zeitung, but now I can't find them again. Searchable resources would be especially valuable. Can anyone help, please? Many thanks. AlexTiefling (talk) 14:14, 30 October 2012 (UTC)[reply]

Here you can select Austrian newspapers online by name or by date, e.g. Klagenfurter Zeitung or 1-Jan-1867. --Pp.paul.4 (talk) 23:16, 30 October 2012 (UTC)[reply]

Work week in Hong Kong

Which of these two unsourced versions is correct? Paum89 (talk) 14:15, 30 October 2012 (UTC)[reply]

The pre-revision version talks about when the typical person (not everyone) works, and about what is open on weekends. This coincides with the section on the US, which says The standard workweek in the United States begins on Monday and ends on Friday, 40 hours per week, with Saturday and Sunday being weekend days. Most stores are open for business on Saturday, and may be open a full or half-day on Sunday as well (except where prohibited by law, which is called the Blue law). In contrast, the new version retains some of this information, but muddies the issue by starting out with In Hong Kong, the working week begins on Monday and ends on Sunday even though presumably few people actually work all seven days. It also says Normally, the Chinese consider the week beginning with Monday and ending with Sunday, which is irrelevant in that it refers to the week and when it starts and ends, not the the workweek which is what the article is about. So I suggest you revert the edit, although you may want to retain the bits about telecomunications and manufacturing. Duoduoduo (talk) 17:23, 30 October 2012 (UTC)[reply]

The first (left) version is better. While some office are open on Saturday morning, this is a fading practice. DOR (HK) (talk) 09:04, 1 November 2012 (UTC)[reply]

Structure of Petition

Does anybody know how the structure of this petition from rulers of Rimatara and Rurutu to the British government would have been like? It is all jumbled up in the sources: [30], [31], [32]

November 27, 1888: Petition from the King of Rurutu and the Queen of Rimatara and their nobles to Her Most Gracious Majesty Queen Victoria, and to the Prime Minister of the United Kingdom. May you have good health. We, Teuruarii, King of Rurutu and Te Maere, Queen of Rimatara and our nobles, ask for the Prime Minister to place our islands and our ships under the protection of the British flag. These are the islands, namely, Rurutu, Rimatara, and Marià, and there are the names of the ships: Faaito and Ronui, and the masters of the same are natives. This is our word to you: Do not forsake us; we are your children; you taught us the word of God, and that has led us in the path of civilisation; therefore we know that you are a good parent to us. The thoughts of the children cling fondly to their good parent; they do not wish to be separated from their good parent. If the parent forsake the children, the children will seek the parent; so do we; we are like those children, and we ask that you will give us your flag to protect us. We have heard that you have taken Rarotonga and the neighbouring islands under your protection, but we remain without anyone to protect us. When we received the news that Rarotonga and the neighbouring islands were placed under your protection, we wept aloud because we were forsaken by you; we were afraid lest we should be adopted by another paretn. The strange parent we mean is the French. They did not feed us with the milk of the gospel, but you did. O Great Britain; you fed us with that milk which has given life to us. This is our last word to you; we do no wish for French annexation or protection, not al all, but we wish you to be our parent, O Great Britain. We pray you now to accede to this our request. This letter was written in the house of Queen Pa. -TEURUARII, TE MAERE ARII.

It seems to be quoted in full in the first source you gave (and you seem to have posted the full petition here). It looks like it is already properly structured. Adam Bishop (talk) 21:29, 30 October 2012 (UTC)[reply]
A petition or any formal diplomatic paper does not look like. I am talking about structure not content. It is there a break after the date? After the title? Is there an indent in the body paragraph? Are the sentences broken into different paragraphs? Is the last part about Queen Pa's house aligned to the left like a letter or is there a break from the last part. Are the names at the end signature? All these question can't be answered by the sources because it is all jumbled up. Even with content each source differ a little on the wording, like one says "a Prime Minister" vs. "the Prime Minister", one uses neighbouring and the other use "neighboring", one uses a semicolon where the other a new sentence is used, etc.

signature.--KAVEBEAR (talk) 00:59, 31 October 2012 (UTC)[reply]

Have you read many 19th-century British formal legal documents? I've seen a ca. 1830s apprenticeship indenture which was also not very well-endowed with paragraph breaks or sentence-internal punctuation. AnonMoos (talk) 06:21, 31 October 2012 (UTC)[reply]

Where would the British government even keep the original or did they discard it?--KAVEBEAR (talk) 14:17, 30 October 2012 (UTC)[reply]

Have you tried the National Archives? --TammyMoet (talk) 14:40, 30 October 2012 (UTC)[reply]
Can't find anything searching for Rimatara or Ruturu?--KAVEBEAR (talk) 20:41, 30 October 2012 (UTC)[reply]
You may need to approach them directly for help. --TammyMoet (talk) 09:53, 31 October 2012 (UTC)[reply]
I've done a bit of a search and it looks like they may be kept at the Institute of Historical Research, University of London. Try and contact them to see (a) whether they do indeed have this document, and if so, how to go about viewing it. I suspect it will have to be done in person. --TammyMoet (talk) 09:57, 31 October 2012 (UTC)[reply]

Of Mice and Men lonliness

Can anyone give me 3 examples of lonliness in the novel "Of Mice and Men"? Exx8 (talk) 14:47, 30 October 2012 (UTC)[reply]

Please see the If your question is homework requirement at the top of this page. Regards, TransporterMan (TALK) 14:52, 30 October 2012 (UTC)[reply]
No, it isn't. thank you.Exx8 (talk) 17:21, 30 October 2012 (UTC)[reply]
Then why do you need to find three examples? And why can't you go find those yourself? - Lindert (talk) 17:24, 30 October 2012 (UTC)[reply]
Yes, ditto. Also, if it is homework, we can certainly guide you with references and suggestions and the like, we just can't answer the question. We are like a library ref. desk, which will certainly help you with your homework, it just won't do it for you. Show us you've attempted to answer the question, and we can give general advice (eg. clarify x/y/z) or offer reading suggestions (eg. Cliff's notes, although I'm sure someone can do better). IBE (talk) 17:29, 30 October 2012 (UTC)[reply]
It really isn't a long book. If you've read it, what's the problem with picking three situations in which people are lonely? AlexTiefling (talk) 17:33, 30 October 2012 (UTC)[reply]
I believe that only George's and Lennie's position is isolated, so I would like to get some help without being attacked.

Exx8 (talk) 17:37, 30 October 2012 (UTC)[reply]

A quick search online for "loneliness in of mice and men" on google hits you with a first result of the BBCs Bitesize website. This page is their section of the bitesize website in relation to the book of Mice and Men http://www.bbc.co.uk/schools/gcsebitesize/english_literature/prosemicemen/3prose_mice_men_themerev1.shtml ny156uk (talk) 18:58, 30 October 2012 (UTC)[reply]

George is lonely after he shoots lennieGeeBIGS (talk) 02:13, 1 November 2012 (UTC)[reply]
There's loneliness throughout. But read the scene in Crooks' quarters to find a couple. Also read the scene involving Candy's dog. Antandrus (talk) 03:08, 1 November 2012 (UTC)[reply]

Russian law about computer crimes

Hi,
does anybody have the Russian law about comptuer crimes?
If you have the translation too, I'll be greatful.
Thank you. Exx8 (talk) 20:00, 30 October 2012 (UTC)[reply]

CREATING A MEDIEVAL BARONY

I'm creating a barony for fiction and I could need some feedback. Both positive and negative feedback is welcome. I'm far from finished creating the barony, but here's what I have got so far.

It's the 1100's and a small barony 4000 acres/4x4 square km large consists of:

  • 640 people, included the baron's family
  • 40 men-at-arms (1/16 of the entire population)
  • 2 villages in which the peasant-families live
  • 1 church, a dairy, a tannery, a cartwright's workshop and a smithy in each of the 2 villages
  • one of the villages have a watermill and a fresh-water river running through it, as well as a lumber-mill/sawmill which can use the river for transportation of timber. The other have a horsemill and a nearby quarry.
  • the land is rich and arable, well-suited for agriculture, and it largely conists of fields and pastures. As such, the barony is hugely self-sufficient when it comes to covering their own need for food. Surplus food is a major source of income from the local market in the city.
  • the barony lies at the coast, and have been granted permanent fishing rights in the rich coastal waters, providing even more food.
  • the barony lies close to the nearest city, allowing them to frequently bring fresh, surplus wares (fish, grain and stone) into the city-market.
  • it holds 1000 acres of forest for timber.
  • Due to barony's smallness, hunting is limited. Hunting can be done in the nearby king's forest however, with a hunting licence.
  • the barony have to import metal and cloth. It does produce some wool of its own from their sheep but linen and a very limited amount of silk must be imported.
  • the hides from slaughtered cattle and other livestock animals is obviously sent to the tannery for making leather.

Do these numbers and facts seem realistic?? If anything seem out place or outright wrong/unlikely to you I would appreciate to read your opinions, which hopefully also comes with reasons for why you think the things you do. Do you see anything that needs altering? Then please say so :) F.ex. do you think a barony this size could hold either more or less people than the 320 people I have? Or do you think there are too many men-at-arms in such a small barony? I'd also appreciate any tips or ideas for things to add, even small things that I might not have thought of.

Krikkert7 (talk) 20:47, 30 October 2012 (UTC)[reply]

I was going to refer you to a previous thread; Several questions about how things worked in a medieval barony, but I see that you wrote that question too! Alansplodge (talk) 21:05, 30 October 2012 (UTC)[reply]
So was I! For this list, 1/8 of the population seems like a lot for men-at-arms, but does that mean anyone capable of being conscripted? That is, all the people the local baron would call up to bring with him - all the teenage/adult men, anyway (no one too old or too infirm). In a population this small, the baron himself would probably be the only knight, who could afford armour and a horse and proper weapons and all that (or maybe one or two others in his family). I'm sure there are lots of academic works about small villages like this, especially in England, we'll have to see if we can find some. I know a list of villages in crusader Jerusalem that also says how many knights each village owed to the king, but that doesn't quite match what we're looking for here. Adam Bishop (talk) 21:21, 30 October 2012 (UTC)[reply]
Economy of England in the Middle Ages Fifelfoo (talk) 21:19, 30 October 2012 (UTC)[reply]
"...does not have much forest, and thus it lacks hunting and access to timber." This is a bit of a problem, since woodland was a fundamental element of a medieval manor. A wood, managed as a coppice or sometimes pollard was an essential. You need; firewood (cooking and heating), charcoal (for the blacksmith's forge), fencing, building materiel (sticks for wattle and daub as well as large timber for joists and planks), wood for tool handles, furniture and crafts, acorns and beech mast to feed pigs. If you have to buy all this in, it not only costs you a lot of money but gives you a huge transport problem. You need to start planting trees now. Alansplodge (talk) 23:02, 30 October 2012 (UTC)[reply]
Is this supposed to be in England? Looie496 (talk) 23:24, 30 October 2012 (UTC)[reply]

A permanent fighting unit of 40 men at arms? You might need that many if you're a robber baron, I suppose, but you'd need to be pulling in a lot of loot to justify the expense. Don't forget, your call-upon fighting force is most of your adult men (and some of what we'd consider adolescents these days, too). You could add a small castle to your list, and one of the villages (the larger one) would be nearby, with the main market, too. Given it's twelfth century, and I too assume you're talking about England, you could have one of those newfangled shell keeps sitting around your ageing motte. Checking out genuine entries in Domesday book, which doesn't precede this period by much, might help with other ideas. --Dweller (talk) 23:55, 30 October 2012 (UTC)[reply]

NB our article on Shell keep is appalling. This site is pretty reliable, although it looks awful. Click the numbered links near the top for more info. Sorry, I'm too knackered to look at it properly now myself - off to bed, methinks. --Dweller (talk) 00:02, 31 October 2012 (UTC)[reply]
Why not obtain a copy of Ken Folloett's excellent and well-researched novel set in a 12th-century English village. Although it's a work of fiction you could get some ideas as to what a medieval barony comprised.--Jeanne Boleyn (talk) 07:56, 31 October 2012 (UTC)[reply]
You could check out Michael Wood's The Story of England , which traces the story by examining one village (Kibworth Beauchamp) from prehistoric times to today. The BBC TV series had at least 2 episodes on the medieval period and I suspect the book will cover it too. It also gives sources for you to follow. --TammyMoet (talk) 09:51, 31 October 2012 (UTC)[reply]

Hmmm. Metal, stone and timber are all expensive things to import, and as for the high amount of men-at-arms, I need the barony to have just that, many men-at-arms, and most, if not all need to be well-trained and well-equipped. I can also see the problem with having no timber of their own as Alansplodge says. The barony needs to be relatively small, but there is nothing in the way for me to double the size of the barony to 4000 acres (4x4 square km). 4000 acres is still relatively small I'd think. With double the size of the barony I could have twice as many inhabitants and thus the high number (40+) of men-at-arms would make more sense. With 4000 acres I would also be able to include, say 1000 acres of forest? I am however unclear on how much timber can be gained from 1000 acres of forest and how long it takes for a forest to regrow once cut down for timber. I've also been considering adding a quarry which would be another source of income. I need enough income to be able to equip my many men-at-arms. Horses, armour, weapons and maintenance are expensive. With double the size of the land I'd also like to have two churches, one in each village. I'd also like to have two dairies. And yes, the manor-house will be a mini-castle with walls up on a small height. I'd also need a large stable (or possibly more than one) for all the horses, for as I said, I need my men-at-arms to be well-equipped and have horses, at least most of them. And yes, a local market in the largest of the two villages is important to have. Of course :)

At the end of the day, the barony need not be rich, but obviously it needs to be in econimical balance, and to be able to defend itself. It is one of several baronies that lies on the border of the kingdom, with the enemy just across the sea to the north - this is the reason for its many men-at-arms.

So the barony can feed itself due its agriculture, and it has 1000 acres of forest, but looking at income vs. expenses I am currently at this:

INCOME:

  • Surplus fish
  • Surplus grain
  • Stone from quarry

EXPENSES:

  • Cloth
  • Men-at-arms (maintenance, weapons & armor)
  • Metal for tools and other small things

It really helps with a quarry for income, as well as for personal use.

Once again, both positive and negative feedback is very welcome

Krikkert7 (talk) 10:11, 31 October 2012 (UTC)[reply]

It would be really useful if you could tell us where in the world this is supposed to be. Looie496 already asked if this is supposed to be England. The climate and the geography make a big difference to the economic and political situation. AlexTiefling (talk) 10:24, 31 October 2012 (UTC)[reply]
Maintaining enough horses, weapons, and armour for 40 men out of a population of 320 would be incredibly expensive, probably enough to totally impoverish the rest of the barony. A horse is like a luxury car, or in this context like a tank. Most importantly it's a status symbol for a knight, the ability to maintain one is part of their higher social status. In Romance languages and German, for example, the word for "knight" is basically "a guy on a horse". They might even have two or three horses if they could afford it. Forty knights with armour, weapons, and horses in a little barony would be very unusual. Forty foot-soldiers in the retinue of the baron would make sense...at least in real life. For the purposes of your story it's not impossible, but I can imagine that situation raising the suspicions of whoever is in charge of the nearby big city! Maybe they're trying to start a rebellion out there...I wonder, though, when exactly in the 1100s this is? Perhaps the barony is centred around the priory of a military order, like the Hospitallers or Templars? They would have access to more horses. Adam Bishop (talk) 10:44, 31 October 2012 (UTC)[reply]
Also, 16 sq km (that's what 4kmx4km comes to) seems very small for a whole barony. I'd expect a baron's domain to run to between 1/4 and 1/2 of a county, and thus to be several hundred square kilometres. 16 sq km is the size of the Isles of Scilly, the smallest rural local authority in England. AlexTiefling (talk) 11:03, 31 October 2012 (UTC)[reply]

To answer ur question whether this is England, no. It is a fictional world and a fictional kingdom which have been long in the making. This allows for a little bit of freedom. However, the kingdom in question is supposed to be very similar to France. The kingdoms are based on real-world kingdoms and the economical and cultural similiraties are definitely there. My version of France is known for its heavy cavalries, its knights, its nobility and as being a leading agricultural kingdom. My version of England is known for their longbownmen and for being leading in the wool-trade, my version of Flanders is a centre of trade and it's weavers are heavy importers of wool from England etc etc. There are many similarites, so you might say the kingdom in question is France. Krikkert7 (talk) 12:08, 31 October 2012 (UTC)[reply]

I'm not sure that twelfth century kings of England and France would have had standing forces of men at arms as large as your very small barony. Look, several of us have said the same thing and we can't make you listen to us, but it does seem odd that you'll ask for advice but not take it. --Dweller (talk) 12:24, 31 October 2012 (UTC)[reply]

Dweller..? I'm taking advice aplenty (!) and thinking all the feedback of great help ! It helps me to work my way towards my goal - in this case to make a barony that both is what I want it to be and at the same time a realistic one. I knew even before I made my initial post here yesterday that I would have to alter things to succeed in that, being well aware that I had far from all angles covered, but its not done in the blink of an eye. Changing one thing leads to me having to change 1 and 2 and 3 other things and so on, and I'm taking feedback to heart all along. Once I have bled this forum dry of suggestions and feedback (from those willing and able to give any) I will read it over several times and go from there. Not sure why you believe I don't listen. Krikkert7 (talk) 13:02, 31 October 2012 (UTC)[reply]

You're right, I apologise - there was no call for me to speak like that. I'll strike the words. --Dweller (talk) 13:16, 31 October 2012 (UTC)[reply]

Some further help for you: from our Longbow article: "Yew is the only widespread European timber that will make good self longbows, and has been the main wood used in European bows since Neolithic times.", so make sure you have a decent Yew forest to hand, if you're a warlike baron. --Dweller (talk) 13:20, 31 October 2012 (UTC)[reply]

(after ec) Dweller, our Man-at-arms article suggests that in 1363, France was supposed to have a standing army of 6,000 men-at-arms, but that the real number was probably about half that. I think it's unrealistic to suggest that an entire kingdom would have as few as 40 men-at-arms even 200 years earlier. Our examples of feudalism article mentions that in 12th century England, a barony might consist of as many as 60 knights' fees - each of which should be able to furnish a knight-bachelor, his esquires, their mounts, steeds, armour and equipment. So a barony might in principle have able 3-400 men under arms if needed. However, those were not permanent, but only for 40 days each year. It's doubtful whether anyone was equipped for war 24/7/365, unless the baron or one of his leading tenants paid for a bodyguard out of their own purse. It's not unlikely that a 16 sq km territory like the one you're describing might be contained in just one or two manors, and be a single knight's fee. If you want to describe a single manor, that's one thing; if you want a functioning barony, you're looking at a larger scale - not just two villages with their churches, but market towns, abbeys, highways, game forests, and so on. AlexTiefling (talk) 13:28, 31 October 2012 (UTC)[reply]
That's my very point. It's entirely reasonable that a baron would be able to call on 40 armed men if he needed them, but for the vast majority their day jobs would not be military. --Dweller (talk) 13:49, 31 October 2012 (UTC)[reply]
A few thoughts on this:
  • This is a rather small domain for a baron. I would expect an area this small would have only a baronet.
  • It's also a rather densely populated area, considering that it's a rural agricultural district, although the doubling of the size makes it somewhat less implausible.
  • You're not going to be able to have any significant number of sheep; they require too much land. There would probably be a few cows for milk purposes, but not enough to export milk or beef unless the local people go without.
  • How important are the fishing rights? If they're important enough that you export fish, as you suggest, then you must have full-time fishermen, and you've given no indication of them. If you don't have full-time fishermen, fishing could still be an important source of protein for locals based upon part-time fishing, but it would not be a significant source of exports.
  • I echo the thoughts of others on fighting men. You essentially have every able-bodied man a fighter. This is realistic for militia defense, but it is not realistic as their full-time occupation.
  • One of your settlements will be smaller than the other. I would characterize the smaller settlement as a hamlet, rather than a village.
  • An area this small would probably not have a proper castle. People would probably go to the church for refuge. There might conceivably be a fortification if it's on the border, as you indicate.
  • This is going to be a poor area, though not unsustainably so. People who are able to work will have food to eat, but no luxuries. People who are not strong enough to work in the fields will be dependent on the charity of others. The family of the baron(et) will have some luxuries, but will long for the greater luxuries that they see in other landed families.
  • There will be very few horses. Horses need a lot of food, and you don't have space for pasturage for them. Other than a small number of horses owned by the family of the baron(et), all of the horses will be needed for farmwork, although they will be multifunctional and also available for other uses. John M Baker (talk) 14:39, 31 October 2012 (UTC)[reply]
Baronets were only invented by James I and VI in the early 17th century - do you mean banneret? A baron was and is the lowest level of peerage; baronets were introduced as a way to raise money (by selling titles) to pay for the hired troops that were needed after the collapse of bastard feudalism.
If you're interested in the use of fortified churches for refuge, there are some good examples in the French Theirache region. But I agree that 16 sq km is much too small for a real castle. A properly-sized barony (say 600 sq km) might have one decently-sized castle, a handful of free-standing keeps, and a number of tower houses, fortified churches and other fortifications. AlexTiefling (talk) 14:53, 31 October 2012 (UTC)[reply]
Do you mean Thiérache? (French WP: Fortified Churches in Thiérache)— AldoSyrt (talk) 08:18, 3 November 2012 (UTC).[reply]
Perhaps what we're talking about here is a Lord of the Manor; "A manorial lordship is not an aristocratic title...". Alansplodge (talk) 17:38, 31 October 2012 (UTC)[reply]
Alex and Alan: I stand corrected. John M Baker (talk) 18:44, 31 October 2012 (UTC)[reply]

Alex and Dweller - surely you agree that even well-trained men-at-arms needed a place to live (on the baron's land) and likely even fields etc to work on. So no, they wouldn't be running around armed to their teeth 24/7, but they would be expected to answer their baron's call when called upon. And they would need to stay fit and ready for military service - the rest of the time they would be working the land or whatever needed be worked on much like peasants and serfs. They would also have families of their own, being fathers and husbands. Some of them might even be 'freemen' who wasn't directly tied to their landlord's land like a villein or serf, instead renting a very small patch of land from the baron for only personal use. And a way of paying that rent could f.ex. be military service rather than paying with coin, or maybe both. At least that makes sense to me. If they were free to work only their own patch of land and did not owe the baron labour-time like serfs they would also find it easier to train and stay sharp. Krikkert7 (talk) 14:25, 31 October 2012 (UTC)[reply]

With "surely you agree...", it sounds like you're trying to convince me that your existing plan works. I'd earnestly encourage you to look again at what I've said about just how big a barony is, and recalculate on those grounds. If you get answers that are on the same order of magnitude as real-life manors (or real-life kingdoms), then your calculations just aren't feasible. Please read our articles on manor, knight's fee, barony, hundred examples of feudalism, and so on. There's a lot there about how the various feudal levels fit together. Your knights are generally going to be lords of the manor, and if you have a knight banneret, he may well also be a tenant-in-chief. All of these people will have their own peasants (eg villeins) to farm their estates. Our modern idea of rent for land post-dates Quia Emptores; the receipt of land for service was the norm before that, and lesser landowners held their land from more senior ones through subinfeudation. AlexTiefling (talk) 14:36, 31 October 2012 (UTC)[reply]

hehe, I was merely talking about men-at-arms and their role outside military tasks ;) But your words are noted, Alex. I eventually have to sit down and really look over everything - more than once too. Krikkert7 (talk) 14:46, 31 October 2012 (UTC)[reply]

John M. Baker. you make many good points and I'll be sure to delve into these matters and do even more research. Krikkert7 (talk) 14:59, 31 October 2012 (UTC)[reply]

In addition to Economy of England in the Middle Ages, try England in the Middle Ages, which may help with some of this. Even England varied considerably, region to region, of course. One general comment I'd make is to be cautious about what one might term environmental determinism, e.g. "the land is rich and arable, well-suited for agriculture, and it largely consists of fields and pastures. As such, the barony is hugely self-sufficient when it comes to covering their own need for food". A region can be rich and arable and many people can still be poor and ill-fed, if - for example - it is supporting a large population and individual land holdings are small, or perhaps if rents are high. You'd probably enjoy Oliver Rackham's "The History of the Countryside", by the way. If you want any advice on literature for particular areas of England (e.g. you specifically fancy the barony being a bit like Suffolk or Yorkshire, etc.) do drop me a line and I may be able to recommend particular books/articles. Hchc2009 (talk) 20:43, 31 October 2012 (UTC)[reply]

October 31

Mizrahi president of Israel

Is Moshe Katsav the only president of Israel to be from a Mizrahi background? — Preceding unsigned comment added by 65.92.149.143 (talk) 00:23, 31 October 2012 (UTC)[reply]

Wikipedia has an article titled List of Presidents of Israel. There are only 9 in history, so it shouldn't take you too long to read each article and find out. --Jayron32 00:26, 31 October 2012 (UTC)[reply]
Yitzhak Navon was born in Israel but came from a Sephardi/Mizrahi family. --Jethro B 03:49, 31 October 2012 (UTC)[reply]
According to the WP entry, Yitzhak Navon's paternal ancestry is solidly Sephardic, having settled in Turkey after the Spanish Expulsion and arriving in Jerusalem in 1670 [!]. His maternal ancestors arrived in Jerusalem from Morocco in 1884; their earlier origins aren't indicated but certainly not among the Mizrachi of the mid/late 20th C. mass immigration waves from North Africa and the Arab Middle East. -- Deborahjay (talk) 13:19, 31 October 2012 (UTC)[reply]

1912 franc

Was one franc worth a lot in 1912? Was it extremely rare for the non-French natives of the colonies? In this, it gives people from Tahiti donating 5 to half a franc to the French army. --KAVEBEAR (talk) 01:59, 31 October 2012 (UTC)[reply]

According to the Stella_(United_States_coin) article, $4 = 20 francs before WW1, so it can't have been extremely valuable. Have no idea what it meant to an inhabitant of Tahiti at that time... AnonMoos (talk) 02:21, 31 October 2012 (UTC)[reply]

"On the Reasons of Jewish Noxiousness"

Both the Polish and English Wikipedia entries on Stanisław Staszic list O przyczynach szkodliwości Żydów (1818) among his best-known works. Neither page provides any information on what this highly regarded philosopher, a "leading figure in the Polish Enlightenment" - albeit a Catholic priest - had to say on the subject, and how influential his position may have been in shaping the hearts and minds of his fellow Polish gentiles and perhaps public policy as well. I'd appreciate help in finding reputable content in English before I turn to Polish sources (and WP:PL editors). -- Deborahjay (talk) 13:11, 31 October 2012 (UTC)[reply]

This book contains a description of his views on the Jews, which I would presume is taken from that work. Another (short) description here. --Saddhiyama (talk) 15:04, 31 October 2012 (UTC)[reply]
I'm sure a Jew could write something "On the Reasons of Goy Noxiousness", but most of them are above that sort of thing. ←Baseball Bugs What's up, Doc? carrots11:12, 1 November 2012 (UTC)[reply]

Why were Indira Gandhi and her son Rajiv so unpopular among the Sikhs?

Both assassinated by them. Iowafromiowa (talk) 16:02, 31 October 2012 (UTC)[reply]

Both deserved their fate, they murdered thousand of Sikhs in the storming of the shrine. — Preceding unsigned comment added by 190.178.162.64 (talk) 16:31, 31 October 2012 (UTC)[reply]
That's a pretty obnoxious response, but it basically answers the question. Looie496 (talk) 16:37, 31 October 2012 (UTC)[reply]
One storming or two? They could hardly both be responsible for the same storming, as (iirc) Rajiv kept out of politics while Indira was alive. —Tamfang (talk) 18:35, 31 October 2012 (UTC)[reply]
I thought Rajiv Gandhi was murdered by Tamils... AnonMoos (talk) 16:43, 31 October 2012 (UTC)[reply]
I think that the events immediately following Indira Gandhi's killing also affects Sikh feelings towards the Gandhis and the Congress Party. Riots broke out in many areas of northern India, where Sikhs were targeted. It is generally accepted that the Congress Party played a major role in these pogroms, and in difference to the storming of the Golden Temple these killings affected Sikhs regardless of political or religious affiliation. And AnonMoos is of course right, LTTE and not Sikhs killed Rajiv... --Soman (talk) 17:18, 31 October 2012 (UTC)[reply]
Indifference even. StuRat (talk) 19:23, 31 October 2012 (UTC) [reply]
For those not familiar with Indian politics, LTTE = Liberation Tigers of Tamil Eelam. Alansplodge (talk) 17:27, 31 October 2012 (UTC)[reply]
Whilst, to my shame, I don't know much about either of the Gandhis, note that it only takes one person (or, at most, a small group) to carry out an assassination. Yitzhak Rabin was generally extremely popular with Israeli Jews, yet was killed by one. History is filled with other examples of people killed by extremist members of normally supportive groups. - Cucumber Mike (talk) 19:12, 31 October 2012 (UTC)[reply]
  • Religious self defense is a central tenet of Sikhism. They are commanded to carry a ritual dagger (kirpan) with that as a possible use. I once discussed the matter with a Sikh taxi-driver, being surprised her own guard had killed her. He expressed the opinion that she should have expected it. μηδείς (talk) 19:40, 31 October 2012 (UTC)[reply]
The thing is that Sikh-state relations were very complex at the time. Sikhs have consistently been overrepresented in the Indian army, since colonial times, and the notion that Sikhs as a collective were loyal to the Republic was very strong. The Sikh extremists on the other hand had remained an extremely marginal force. The storming of the temple disrupted balance of power in Sikh politics temporarily, and was probably difficult to predict its consequences. --Soman (talk) 21:25, 31 October 2012 (UTC)[reply]
I'm not saying that she should have fired all the Sikhs in the military, just found a few Hindu bodyguards to take over. This brings up an interesting Q, does the US Secret Service employ Muslims to guard the President ? Presumably most would do an admirable job, but there's always the risk of an extremist. StuRat (talk) 22:12, 31 October 2012 (UTC)[reply]
That question could be posed from various angles, e.g. "Does the US Secret Service employ Christians to guard the President ? Presumably most would do an admirable job, but there's always the risk of an extremist." -- Jack of Oz [Talk] 23:46, 31 October 2012 (UTC)[reply]
Why would Christian fundamentalist kill a Christian President ? Or are you one of those who believes that Obama is a Muslim ? StuRat (talk) 23:58, 31 October 2012 (UTC)[reply]
I am fairly certain, StuRat, that Jesus commanded his followers something like, "Kill them all, you em-effers! Kill them all!" μηδείς (talk) 00:42, 1 November 2012 (UTC)[reply]
That 2nd quote is bad news for those who enjoy fencing. StuRat (talk) 00:46, 1 November 2012 (UTC) [reply]
Caedite eos. Novit enim Dominus qui sunt eius. ("Kill them all. For the Lord knows those who are His.") Clarityfiend (talk) 06:53, 1 November 2012 (UTC)[reply]
Crusader makes a joke 800 years ago and Jesus gets the blame? μηδείς (talk) 17:04, 1 November 2012 (UTC)[reply]
A fundamentalist Christian can kill the president if he thinks the president isn't being "Christian" enough, if he is siding with Muslims too much or something, or if he appear too "anti-Christian". Plenty of Muslims have killed other Muslims and plenty of Christians have killed other Christians. And people can change minds too. A person might be okay, a decent Muslim with a clean record so he gets hired as a body guard. Then the president does something extremely "anti-Muslim" which flips him or he meets someone who convinces/brainwashes/blackmails him and then he assasinated the president.70.58.0.141 (talk) 02:00, 1 November 2012 (UTC)[reply]
You are just making that up off the top of your head, and rather poorly. I am an atheist, but even I have enough respect for Jesus not to make up such nonsense. μηδείς (talk) 02:08, 1 November 2012 (UTC)[reply]
No, Stu, I'm not one who believes Obama is a Muslim (not that there's anything wrong with that). But I equally do not believe that, just because someone is a Muslim, there's automatically more of a "risk" factor than if they were a Christian or a Buddhist or a Jew or an atheist or anything else. I know it's anachronistic to make this comparison, because the rise of terrorism postdated them, but I'll say it anyway: None of the assassins of US Presidents have been Muslims. -- Jack of Oz [Talk] 09:58, 1 November 2012 (UTC)[reply]
It seems quite obvious that there are Muslim fundamentalists who would like to assassinate any US President. It's not at all obvious in the case of Christian fundamentalists, although I could possibly imagine them assassinating a liberal President, utilizing similar logic to that used to justify killing abortion providers. StuRat (talk) 10:06, 1 November 2012 (UTC)[reply]
It appears that sentiment is still there. See SGPC pays homage to Indira's assassin. 61.16.182.2 (talk) 04:05, 1 November 2012 (UTC)[reply]

Freemasonry and youth

I'm 77 and wanting to enter the Iowa Grand Lodge, once inside, can I invite my 20-year-old nephew to enter Freemasonry too? He is supported by his mother. Iowafromiowa (talk) 16:23, 31 October 2012 (UTC)[reply]

Maybe. See our article on Freemasonry#Membership requirements for the details. Looie496 (talk) 16:36, 31 October 2012 (UTC)[reply]
Some jurisdictions require apprentices to be 21 years old. Roger (talk) 16:38, 31 October 2012 (UTC)[reply]
Technically, the answer is "No, you can not invite him to join"... Masons are not supposed to invite potential members to join... potential members are supposed to initiate the process by asking a mason to propose him. Of course, once you are a member, you can let him know of this tradition... and inform him that, should he ask to join, you would be more than willing to act as his proposer. (ie you can ask him to ask you).
That said... My advice is don't rush into things... first go through your own initiation, passing and raising. Get a sense of what Freemasonry is all about. In a year or two, you will have a much better concept of the sort of men your lodge is looking for, and whether your nephew is one of them. Blueboar (talk) 20:47, 31 October 2012 (UTC)[reply]

"The New Evangelization"

So, how is the Roman Catholic Church going to evangelize Europe? Send missionaries all across Europe and invigorate the Catholic-Christian faith? What happens if an irreligious European becomes a Protestant instead? Does that still count, or does the Roman Catholic Church only recognize Catholics as true Christians? Is there any benefit in being a Catholic rather than an irreligious person? I attended a Wednesday Mass today, and this priest mentioned something about the New Evangelization, and even though the New Evangelization is "great" and all that, he claimed that Catholics should be respectful and loving and kind and generous and (ideally) allow those good characteristics to bring new Christians in. How is this "evangelization" carried out? 140.254.121.34 (talk) 20:05, 31 October 2012 (UTC)[reply]

This is partly commentary, and partly a question. As per the question, the Catholic Church does missionary work across Europe but more low-key through its own example (like running schools, charities, etc.) than the active door-to-door knocking preaching of Jehova's Witness and some protestant groups. A fair number of Protestants are converting to Catholicism every now and then (imho seemingly interested by the mysticism of the Catholic church, absent in many Protestant denominations), so the Protestant/Catholic divide is somewhat blurred as compared to past centuries. --Soman (talk) 21:31, 31 October 2012 (UTC)[reply]
The main objective of "New Evangelization" is to convince Catholics who have stopped practicing to do so again. There is no thought of trying to convert Lutherans or Calvinists or whatever. --Xuxl (talk) 09:31, 1 November 2012 (UTC)[reply]
When I was in France, I was amused to see Catholic tracts "accidentally" left in convenient places (public toilets, on the Métro, etc). In North America, that sort of thing is associated with evangelical Protestants, like Chick Tracts. Adam Bishop (talk) 10:44, 1 November 2012 (UTC)[reply]
There are some thoughts on the matter of Evangelization within the Catholic Churches of Europe. (And not only within those specifically. Biblical literacy is pretty low in many areas.) Youcat and | NightFever being some - specifically Catholic - of them.
On the recognition issue: That is a fairly complex thing, which, in its entitiy, is well deserving its own article. Generally speaking, Protestant Churches -- so long they have Bishops -- are recognized as being 'a' Church (though not 'the' church) and their Baptisms as valid - so long as the ritual involving water and a trinitarian formula is being adhered to. Valid Baptisms again are generally speaking considered necessary for salvation in Catholic Theology. (Which is why there is such a thing as a Emergency Baptism.) Churches with invalid Baptisms are, again very generally, regarded as not being Christian. (Yes, the Latter-Day Saints are, according to Catholics, neither a Church nor Christian.)
On the matter of benefits: Theologically, from a Catholic perspective, there are some of course. But then again benefits in religion are scarcely objective in an economical sense. Though objectively you can communicate within the church only when youre Catholic. (That disambiguation page needs a section on Theology, but Im too new here to work on that.) I doubt a Priest would try to subsume the issue under the guise of 'benefits'. --Abracus (talk) 12:37, 2 November 2012 (UTC)[reply]
Are all Trinitarian churches with bishops really considered good enough from a Catholic point of view? I was under the impression that Methodists didn't have apostolic succession in Catholic teaching, and I can't imagine the Pope recognising the Church of God in Christ as having it. Nyttend (talk) 01:15, 3 November 2012 (UTC)[reply]

Economic impact of natural disasters

After a catastrophic natural disaster with significant damage such as the 2011 Super Outbreak, 1999 Sydney hailstorm, or say the recent Hurricane Sandy, how much of the economic loss caused by the storm is recuperated by the subsequent rebuilding and related processes? (The lines I'm thinking along are basically "If a tornado destroys a $150,000 house and it costs $300,000 to build a new one (between parts, labor, etc) does this mean that the tornado actually boosted the economy by $150,000?") Ks0stm (TCGE) 22:22, 31 October 2012 (UTC)[reply]

Some economists seem to think that way. I've always been one to wonder where the $300,000 actually comes from. HiLo48 (talk) 22:50, 31 October 2012 (UTC)[reply]
Broken window fallacy. Loosely related: Lump of labour fallacy. —Tamfang (talk) 22:51, 31 October 2012 (UTC)[reply]
Imagine that instead of you hiring other people, it was all you. You would consider that time rebuilding wasted. Larger, more complicated economies are no different. You're thinking about the flow of money/value through an economy, which is a natural way to think about things because that's how governments tax, but if you want to disabuse someone of the notion that "flow" is equal to "value" then consider if you bought and sold the same item back and forth for exactly the same price. You'd generate a lot of "flow" but no change in value. Shadowjams (talk) 23:03, 31 October 2012 (UTC)[reply]
The short answer is, none of it is recouped, per the above fallacies, also known as the difference between what is seen and what is not seen.. μηδείς (talk) 23:05, 31 October 2012 (UTC)[reply]
I believe that the flow of money can lead to increased wealth. The reason is that unemployed people, who were not contributing to the economy, are then employed to do repairs, and thus contribute. If they get enough on-the-job training to remain permanently employed, all the better. There's also an opportunity to rebuild destroyed slums as something more valuable. StuRat (talk) 00:01, 1 November 2012 (UTC)[reply]
The money that is wasted paying the unemployed to repair what otherwise would already have been in good condition, were it not for the hurricane, would otherwise have been spent or invested in other activities that would also have employed them, but without the cost of the hurricane's destruction. That's the whole point of the fallacies mentioned above, StuRat. If destruction were good for the economy, the best thing would be to burn absolutely everything. The problem is that you see rebuilding to the same state we had before, and think economic activity. But what you don't see is the economic activity that will never happen because people have to spend money getting back where they were in the first place. For a downloadable PDF that covers this in detail, see economist George Reisman's college textbook, Capitalism. μηδείς (talk) 00:29, 1 November 2012 (UTC)[reply]
Re: "would otherwise have been spent or invested in other activities". Not necessarily. If government does the repairs, and is already spending beyond their income, this means more money will likely be borrowed to pay for employing those people. This is a classic economic stimulus plan. Of course, some day we will have to pay the piper when the debt comes due, but that's another discussion. StuRat (talk) 00:35, 1 November 2012 (UTC)[reply]
Not a very good mathematician if you don't think equations balance whether we want them to or not. Read Reisman. He's also quite hilarious. μηδείς (talk) 00:45, 1 November 2012 (UTC)[reply]
Not sure what your argument is. Does the government borrowing money to pay people to go to work stimulate the economy or not ? Is your argument that no matter what happens or what anybody does, it has no net effect on the economy ? If not, then how do you distinguish between things which do and things which don't ? StuRat (talk) 01:20, 1 November 2012 (UTC)[reply]
The paying of the piper (or the popping of the bubble) would thus be the "what is not seen" part. —Tamfang (talk) 20:41, 1 November 2012 (UTC)[reply]
If, StuRat, you didn't have to pay to fix your roof, wouldn't you either have spent that money on some other item which would have kept someone else employed or got the a job, or saved it, meaning it would be available as capital to a borrower to start a new business? μηδείς (talk) 00:31, 1 November 2012 (UTC)[reply]
Saved it in my sock drawer. :-) StuRat (talk) 00:35, 1 November 2012 (UTC)[reply]
That would cause deflation, increasing the value other other's assets, still serving the same purpose, but not profitting you by the interest you forewent. μηδείς (talk) 00:43, 1 November 2012 (UTC)[reply]
I'm guessing you mean it would decrease inflation, as causing deflation would be a bad thing. StuRat (talk) 01:18, 1 November 2012 (UTC)[reply]
Your putting your money in a sock is a local irrational choice that will at worst contribute to disinflation or a minor deflation, both of which are good. It cannot cause a disinflationary spiral unless, as they say, everyone is batshit, in which case we are all, as the Viennese Economists say, "fucked". μηδείς (talk) 02:01, 1 November 2012 (UTC)[reply]
I don't think it is irrational, considering the risk versus reward of having it in a bank. Interest rates are truly pathetic now, such that monthly fees and hidden bank fees can quickly overwhelm them. Then there's the possibility of your money becoming unavailable when you need it. A friend had his account locked down due to "irregularities in his account" (suddenly taking the max out at ATMs several days in a row), and had to meet with the manager and present his birth certificate and several forms of photo ID to convince them he was really the owner. Given these risks, it makes sense to keep some cash at home. StuRat (talk) 07:18, 1 November 2012 (UTC)[reply]
That sounds like a bank that's taking precautions to protect its customers. Supposing he had been called in and it turned out it wasn't him that had been pulling the money out? There's another factor to consider: Spending is what fuels the economy. The more money banks have, the more they can loan out and stimulate business. Every dollar stashed in a sock is a dollar not being spent, and a small chip taken out of the economy. See Paradox of saving. ←Baseball Bugs What's up, Doc? carrots11:24, 1 November 2012 (UTC)[reply]
Building war equipment spent the US out of the Great Depression. Cities and factories in Germany and Japan were badly damaged in WW2, but the rebuilding seemed to give their economies quite a boost. One financial person on TV today said that fixing the damage from the storm would be a boost to the US economy (in a way that gets around Congress's refusal to pass any infrastructure programs proposed by the President). Edison (talk) 04:08, 1 November 2012 (UTC)[reply]
As usual in economics, the answer is "it depends". If before the disaster the economy is wisely and fully employing all of its scarce resources, above all people, then μηδείς is absolutely right. Destruction is destruction. Real wealth is going to be destroyed. It has very little to do with money. People are going to be taken from other tasks to rebuild. No recouping. But continuing to follow her for one more sentence before violently disagreeing, in the world we live in everyone is batshit, and we are all, "fucked". Because the assumption that Bastiat and μηδείς make of full employment is rarely satisfied. Because modern economies rarely are sensibly run at full employment, except when disasters have occurred, in particular the disaster of war. If the disaster kills everyone or the nation is conquered in war, then she is still right of course. But if we have the conditions of the USA before WWII - still high, though decreasing, unemployment, then the war-disaster can stimulate the economy enormously. Of course it would have been better to not have the war or disaster and for the government to spend the same amount on peaceful pursuits if there is mass unemployment - then the economy would have grown even faster. But the constant disaster of forcing people not to work during the usual periods of mass unemployment in peacetime capitalist economies with no disaster is an unseen destruction that dwarfs many, many broken windows. As Keynes said "Pyramid-building, earthquakes, even wars may serve to increase wealth, if the education of our statesmen on the principles of the classical economics stands in the way of anything better."[33] The disaster can be a helpful kick in the ass.John Z (talk) 10:18, 1 November 2012 (UTC)[reply]
Well said. Note that government spending to increase employment must not compete with private industry, or it will just displace those workers. So, this means they need to do something which private industry won't do. Hence the building pyramids bit. I'd personally like to see more employment in the field of space exploration. For example, the Mars Rovers seem to be quite successful, so why not set up an assembly line, build thousands of them, and explore every inch of Mars, the Moon, and whatever other rocks we can land them on  ? (This may not make sense in the current debt situation, but is a general observation on stimulus programs.) StuRat (talk) 10:40, 1 November 2012 (UTC)[reply]
I think it might be efficient - even desirable - for the government to compete with private industry if private industry's existing provision is significantly inefficient, or acts contrary to public interests which the state might reasonably be expected to serve. To take a no-brainer: the existence of mercenaries clearly demonstrates that the provision of armies is something which the private sector can and will do; and the state could, in theory, just hire mercenaries when it needs troops. But the risks to the state in such an arrangement (bidding wars immediately before or during shooting wars, unilateral terrorism as reprisal for non-payment, leaks of state secrets) mean that most states since 1648 have wanted their own standing armies instead. More contentiously, it could be argued that beyond a critical point, the provision of railways solely by private enterprise leads to highly inefficient use of land that is in demand by other sectors - and therefore at least the infrastructure ought to be masterplanned and built by the state, even if the resulting traffic is privately-controlled. AlexTiefling (talk) 13:46, 1 November 2012 (UTC)[reply]
The US got a boost in WW2 from being the last major industrial power not at war; the stimulus money came from outside. After the war, Germany and Japan had the benefit of not having to pay off the old regime's war bonds out of new taxes. —Tamfang (talk) 20:41, 1 November 2012 (UTC)[reply]

Nobody has mentioned yet that the bulk of cash for reconstruction will not be coming from the US taxpayers, but from the world-wide insurance market. Estimates for the total loss for this "catastrophe" (the term used by insurers for very large losses) vary from USD 20 billion[34] to 60 billion[35]. Insurers obviously know all about hurricanes and use sophisticated software to be certain that their exposure to any one event will not be more than the company can cope with. The system of reinsurance ensures that the burden of the loss will be spread around hundreds of companies worldwide, much of it via Lloyds of London and the associated companies that make up the London insurance and reinsurance market. As with the 9/11 losses, I suspect that the business interruption claims will outstrip all the personal and property losses. As to the overall effect on the US economy, the "Time Business and Money" link above gives an opinion that "In the long run, the devastation the storm inflicted on New York City and other parts of the Northeast will barely nick the U.S. economy. That’s the view of economists who say a slightly slower economy in coming weeks will likely be matched by reconstruction and repairs that will contribute to growth over time." Alansplodge (talk) 15:51, 1 November 2012 (UTC)[reply]

You all (except maybe for Medis) are bouncing around to avoid the central point that wanton destruction doesn't help civilizations by and large (if someone brings up the black death then this conversation may take a decided turn). You guys keep referring to all the rebuilding efforts, but the simple AND OMG A 1ST GRADER WOULD REALIZE THIS, fact is that if I unnecessarily destroy something that gave value, the rebuilding of it will not somehow magically give more value. If you believe otherwise, I'm available for hire. I supply my own sledgehammer. Shadowjams (talk) 04:32, 3 November 2012 (UTC)[reply]

Parenthetical parentheses

If I want to cite a sentence with parenthetical parentheses, but I want to use two citations, would I put both citations in two seperate parentheses at the end of the sentence? For example:

I like to eat cake (Krugman, The New York Times) (Pear, The New York Times).

Thanks. --Jethro B 22:42, 31 October 2012 (UTC)[reply]

Why not (Krugman, The New York Times; Pear, The New York Times)? Clarityfiend (talk) 23:43, 31 October 2012 (UTC)[reply]
Is that how it's supposed to be done? I just don't know the correct way to do it, so I'm asking. --Jethro B 00:40, 1 November 2012 (UTC)[reply]
In what context? On Wikipedia, or under the regime of some academic style guide (and if so, which one)? -- Finlay McWalterTalk 01:51, 1 November 2012 (UTC)[reply]
Not Wikipedia. It's not really for academic peer review or anything, I'm not publishing it, but I guess we could follow those guidelines. Anyway, now this is more of a hypothetical question, as I've avoided the issue by simply splitting the material in each reference to two different sentences and then using only one citation per sentence. --Jethro B 03:56, 1 November 2012 (UTC)[reply]
Citation styles differ widely, depending on who the publisher is. Also see Parenthetical referencing. Nowadays, you will nearly always use software to manage your references - if you are lucky, you use BibTeX, but see Reference management software. Most scientific publishers will tell you which style they prefer, and with reasonably software this is just a global setting (e.g. \bibliographystyle{plain}) to change this in the finished paper. --Stephan Schulz (talk) 09:05, 1 November 2012 (UTC)[reply]
The most common style is to use a semicolon, as in (Krugman, New York Times; Pear, New York Times). Looie496 (talk) 15:06, 1 November 2012 (UTC)[reply]

NYPD firearms and bullets

What kind of bullets do the NYPD officially supply their officers with? I know the official weapons are largely Glocks 19s (which are all 9mm I think?), but this New York Times article lists full metal jacket, but flat fronted .40 S&W. I don't live in NYC but in the times I've been there I've overwhelmingly seen officers carrying service sized Glocks, but I can't really distinguish the caliber at a glance. Does anyone have any additional knowledge? Shadowjams (talk) 22:53, 31 October 2012 (UTC)[reply]

It was quite a matter of controversy whaen they switched over to hollow points in the late 90's. You can search google getting these results with this at NYT being an example. μηδείς (talk) 23:03, 31 October 2012 (UTC)[reply]

November 1

unattractive women

I know some young women who are nice and have attractive qualities but who aren't much to look at. One of them, for example, has (her words) "terribly unattractive breasts". They are single and have found a certain logic that keeps them that way: they say, "If I loved a man enough to want to be with him long term/marry him, then I would love him enough to not want him to be stuck with someone as unattractive as I am."

The difficulty for me is that I found this hard to argue with. What advice should I give them, when it seems that perhaps more people should be self-aware/selfless like that? Are they so wrong that they need correcting? 24.101.18.83 (talk) 01:02, 1 November 2012 (UTC)[reply]

The problem is not that they are selfless, but that they don't love themselves. Someguy1221 (talk) 01:09, 1 November 2012 (UTC)[reply]
That's the same thing. See The Virtue of Selfishness. μηδείς (talk) 01:40, 1 November 2012 (UTC)[reply]
That's very similar to the logic: "I'd never want to join a club with such low standards as to let somebody like me in". And how can a young woman's breasts be unattractive ? Varicose veins ? Different sizes ? StuRat (talk) 01:16, 1 November 2012 (UTC)[reply]
That was one of the questions that passed through my mind as well, Stu. I know one woman who is completely flat chested. Which in today's society is generally not something that is lauded or desired. She is very happily married to what I would guess is a very attractive man who is successful and is very attentive to her. She's also, IMO, very pretty. So there's that also to consider but my point is, breasts aren't everything. As for the OP, I can't remember where I saw it. Some documentary I found on Netflix one night, that much I know. Anyway, the documentary was about people's attractiveness. And in it there was a study where people were asked to rate those that they would date and those that they thought were "out of their league" and such. Time and again people were matching each other up with someone who was of similar relative attractiveness. I know that probably won't go a long way to console some unattractive woman, as the OP refers to them, but it's worth keeping in mind. Dismas|(talk) 01:23, 1 November 2012 (UTC)[reply]
Small breasts are not universally considered unattractive. And they are also easy to "fix", between push-up bras, falsies, and implants. StuRat (talk) 01:31, 1 November 2012 (UTC)[reply]
(e/c) I believe that she used to be very overweight (well into the obese category) and lost it all to now be considered thin/slim. While that is very laudable, it's left her 'droopy', I believe considerably so. She is very distressed that almost all young women have nice breasts apart from her, and that even her post-menopausal mother has more conventionally attractive breasts. She sees her breasts (and other 'unattractive' features) as deal-breakers, and as there is such a culture of attractiveness, I don't see how I can dissuade her, as it is something that men actually do care about. I tried to tell her that not all men care about such things, and she said that it would mean being with someone who therefore wasn't interested in sex, and that she'd rather be single (with the possibility of having one-night stands) than in a passion-less relationship. 24.101.18.83 (talk) 01:39, 1 November 2012 (UTC)[reply]
Physical infatuation lasts for the first 30 days of a sexual relationship. After that, if you are two mature people with shared values an adult relationship is possible, assuming you have been brought up to value respect and affection. See oxytocin. μηδείς (talk) 01:48, 1 November 2012 (UTC)[reply]
Your "friend" has a very sad view of herself and of others that I think is well beyond the capability of the respondents on the Ref Desk to address. How she gets from "don't care" about conventional measures of attractiveness to "uninterested in sex", I don't know. I am having difficulty in seeing this as a serious question. (And I am curious as to where Medeis's "30 days" comes from. My own experience says it can be longer or shorter, and by quite a large margin.) Bielle (talk) 01:53, 1 November 2012 (UTC)[reply]
Men are not traditionally known for their ability to look past physical appearances. I don't think it's at all unreasonable for a young and inexperienced woman to think that either a man cares about physical appearances and therefore about sex, or he doesn't care about either. Is there much evidence out there to the contrary? Not really. And why does she have such a sad view of herself? Isn't she . . . well, kind of right? Put a nude picture of her on the web, for example, and she'll be harshly criticised in the crudest of terms by any man passing by. Is she supposed to delude herself into thinking she's okay? 24.101.18.83 (talk) 02:01, 1 November 2012 (UTC)[reply]
la nuit, tous les chats sont gris. --Trovatore (talk) 01:32, 2 November 2012 (UTC)[reply]
Sure men can look past physical appearances:
"By then her once smooth skin was wrinkled and her former auburn hair had turned grey, but fortunately I was able to look past all that ... and see her pretty daughter standing behind her." - Montgomery Burns. StuRat (talk) 02:18, 1 November 2012 (UTC)[reply]
Your own response, Bielle, indicates that you see it as an a average, which is obviously what I meant. On the close mean order of a month. μηδείς (talk)
If we were all looking for the same things in our partners, there would be one very long line up, and almost all of us would be doing without. Take a look around at the real world. We don't live in advertisements or magazines or TV ads. Bielle (talk) 02:42, 1 November 2012 (UTC)[reply]
No, but men wish that they did. 24.101.18.83 (talk) 02:54, 1 November 2012 (UTC)[reply]
Not in my world. I think yours must be very young and inexperienced. Bielle (talk) 03:23, 1 November 2012 (UTC)[reply]
Bielle -- "Almost all of us would be doing without" only if no-one was willing to "settle". See Assortative mating, Matching hypothesis... AnonMoos (talk) 15:04, 1 November 2012 (UTC)[reply]
Nothing in either of your links, AnonMoos, speaks to "settling"; quite the opposite, in fact. They speak rather to individuals being attracted to (not "settling for", whatever that may mean) others at their same level of attractiveness. (I am still not certain how those levels are determined or by whom.) We aren't all attracted to the same characteristics no matter what advertisers (and inexperienced young men and women say) say and do and not all of attractiveness is about physical appearance. Bielle (talk) 18:59, 1 November 2012 (UTC)[reply]
Shallow people can quickly size up how "hot" someone was, as in Hot or Not. I once encountered 3 girls walking down a sidewalk at a college, and each would utter a number from 1 to 10 as they passed a guy walking the other way, as if they were judges at a diving competition. I resented their "negative" comments. Edison (talk) 04:04, 1 November 2012 (UTC)[reply]
To the OP: why do you think your friend's view is a problem? Is she obviously unhappy about herself, and does she obviously want to marry? In any case, your friend's logic is fallacious because men do not all fall into the cookie-cutter stereotype she seems to have in her mind. I'm only slightly older than the average age of a first date, and people of my age are stereotypically portrayed as much shallower than adults. Even so, I know boys who prefer flat chests. I know boys who prefer droopy breasts, or disgustingly gigantic breasts (for my taste), or the type of breasts you see in magazines. I also know boys who like nerdy girls, almost regardless of her looks. Even more oddly, I know boys who like tomboys, in the sense of physically and emotionally tough girls. My point is that human sexuality, just like most other human preferences, is bizarre, diverse, deviant, and abnormal in countless ways, and none of those words is a judgement. --140.180.252.244 (talk) 05:33, 1 November 2012 (UTC)[reply]
I find it hard to believe that you know this. Young men who sit around discussing sex don't talk about how they like droopy boobs! Most guys would be ashamed of that. And they wouldn't say they like 'em flat, either. It's cool to like the perfect ones in magazines, or the fake ones in porn. That's why a lot of girls are in a hopeless situation, because men are being fed a warped perception of female beauty that exists in not so many girls. I think you're guessing. 24.101.18.83 (talk) 18:38, 1 November 2012 (UTC)[reply]
You accuse me of guessing, yet your entire post is one enormous guess about what young men I know, what young men do, and what young men are ashamed of. You also use the phrase "most guys" numerous times, whereas the entire point of my post is that not all guys act in the same way that "most guys" do. Your claim is similar to saying "most guys aren't smart enough to invent relativity. Therefore, everyone who says Einstein invented relativity is guessing." --140.180.252.244 (talk) 20:53, 1 November 2012 (UTC)[reply]
Speaking solely and exclusively for myself, I like well-proportioned breasts -- and I really do not find breasts grotesquely disproportionate to the body to be attractive. There are also changing cultural ideals -- the 1920s favored a somewhat small-breasted ideal, while 1950s media depictions played up large breasts. Many 1950s sex-symbol models and actresses would be considered "fat" today... AnonMoos (talk) 01:11, 2 November 2012 (UTC)[reply]
Regardless, what makes a woman beautiful is primarily her face (and I know I speak for a lot of guys saying that). Sure, men like breasts, but they are of lesser importance, especially when it comes to long-term relationships. (see e.g. [36] and [37]) - Lindert (talk) 19:47, 1 November 2012 (UTC)[reply]
I would tell them that in the long term, character is much more important for a happy relationship than looks. I would be far less afraid to be 'stuck' with a less attractive wife, than with a gorgeous but nasty wife. As Solomon said: "Better to live on a corner of the roof than share a house with a quarrelsome wife." (Proverbs 21:9) - Lindert (talk) 09:13, 1 November 2012 (UTC)[reply]
I think you're missing the point. The unfortunate looking girls might be nice and easy to get along with, but they know that there are plenty of nice, easy to get along with girls who are also good-looking. That leaves them at the end of the queue, and probably better off single. And even then, men feel entitled (by virtue of their biology) to stray. A happy home and a mistress in a hotel, I guess that's what men want. 24.101.18.83 (talk) 18:38, 1 November 2012 (UTC)[reply]
The 'plenty' part is simply inaccurate. There are just as many, sometimes even more men than there are women. You may be right when it comes to men who are rich, successful, handsome etc, but just like with women, there are plenty of guys that fall short in one category or the other. For them, choices are much more limited. What it comes down to is that women who are not 'perfect' in every way will just have to settle for a guy who is also less than perfect. It works both ways. And about men being unfaithful, maybe you are being too pessimistic. There are decent guys left and I know plenty of them. (And it's not like there aren't any women who cheat either.) Try to view each person individually and don't judge them before you know them. - Lindert (talk) 19:28, 1 November 2012 (UTC)[reply]
Beauty between the ears trumps most everything else. ←Baseball Bugs What's up, Doc? carrots11:20, 1 November 2012 (UTC)[reply]
Someone once wrote a song about that... AnonMoos (talk) 15:04, 1 November 2012 (UTC)[reply]
Yes. And a few years later, Flip Wilson made this to-the-point comment: "You marry a beautiful woman, after a while she turns ugly. Marry an ugly woman, get a few drinks in you, and she starts lookin' good!" ←Baseball Bugs What's up, Doc? carrots22:51, 1 November 2012 (UTC)[reply]
Body Dysmorphic Disorder. --TammyMoet (talk) 18:32, 1 November 2012 (UTC)[reply]

One does not simply walk into Mordor!

Is there an original Tolkien quote on which this dictum of Boromir in the movie is based on? If yes, what is the wording? --KnightMove (talk) 07:35, 1 November 2012 (UTC)[reply]

I don't think so. In fact during the Council of Elrond—when Boromir says this in the movie—in the book Aragorn mentions ([38]) walking within sight of the Black Gate and in Morgul Vale, neither of which is technically in Mordor but are close. And a page later Gandalf says Gollum "went to Mordor", although he was caught. I suspect this little speech of Boromir's, which goes on to mention the unsleeping Eye, etc, was put into the movie in order to summarize quickly what takes a lot longer to make clear in the books—not only the "folly" of trying to sneak into Mordor but also Boromir in particular rejecting this plan, or accepting it only grudgingly. This goes with his idea that the Ring should go to Gondor as a weapon of war, and/or his rapid falling prey to the lure of the Ring. In any case, the Council of Elrond chapter was much rewritten for the movie, unsurprisingly. Pfly (talk) 08:09, 1 November 2012 (UTC)[reply]
The closest thing I could find from Boromir is "The only plan that is proposed to us is that a halfling should walk blindly into Mordor and offer the Enemy every chance of recapturing it for himself. Folly!" (Said shortly before he tries to take the ring from Frodo). Later, Gollum puts it this way: "It's not sense to try and get into Mordor at all." - Lindert (talk) 08:39, 1 November 2012 (UTC)[reply]
"One does not simply walk into Mordor." ... "Quite right, but with proper assistance, two should have no trouble at all." StuRat (talk) 08:54, 1 November 2012 (UTC) [reply]
My favorite variation is http://www.ooblick.com/text/tomordor/ (though some of the roads mentioned seem to have been in extreme disrepair at the end of the third age)... AnonMoos (talk) 14:57, 1 November 2012 (UTC)[reply]
Tsk, tsk. The lengths people will go to to avoid baggage fees for magical items. Clarityfiend (talk) 23:01, 1 November 2012 (UTC)[reply]

Satire artwork interpretation

What does the last one mean[39]? (The one with the sandwich covering a calendar.)24.246.85.20 (talk) 13:00, 1 November 2012 (UTC)[reply]

Given the general drift of the others: that the single sandwich is the only food for that month, and thus is being served as one small piece per day. It does not work as a stand-alone carton since it lacks context. --Tagishsimon (talk) 13:21, 1 November 2012 (UTC)[reply]
Also note there is only butter in the weekend, and only cold cuts on Sunday. That could be interpreted in different ways but is certainly different from the expectation in rich countries. I don't know whether there is any significance to the calendar being from March 2005. PrimeHunter (talk) 13:41, 1 November 2012 (UTC)[reply]

Location of street address in California and attributes

In Los Alamitos Unified School District I'm trying to determine if "THE BUNGALOWS - LONG BEACH 90815 Claremore Lane 2900 - 2972 (even #’s only)" is within the Long Beach City Limited and/or the Los Angeles County City Limits. Long Beach is supposed to be entirely within Los Angeles County (but I'm trying to see if 2900-2972 Claremore is in Orange County)

Thanks, WhisperToMe (talk) 20:13, 1 November 2012 (UTC)[reply]

It appears from a topo map that the county line cuts through at least some of the buildings on the south side of Claremore Lane. —Tamfang (talk) 20:35, 1 November 2012 (UTC)[reply]
It would be interesting to see how the buildings are classified, if they vote in LA County elections but get services from Orange County or if they vote for LBUSD schools but can send kids to Los Alamitos schools... WhisperToMe (talk) 20:40, 1 November 2012 (UTC)[reply]
In some similar cases, cross-boundary buildings pay taxes and receive services from the jurisdiction in which their main entrance is located (have no idea if that would apply here)... AnonMoos (talk) 09:47, 2 November 2012 (UTC)[reply]
One has heard of doorways being moved in Baarle for that reason. —Tamfang (talk) 05:00, 3 November 2012 (UTC)[reply]

November 2

How much did Thomas Peterffy pay for his seat on the AMEX in the 70s?

How much did Thomas Peterffy pay in "70's" dollars at the time for a seat on the AMEX? Was this something only already extremely wealthy people could do, roughly speaking? According to his article, he got to the US in 1965, so had only been there 15 years by the end of the 70s, and his jobs there were first draftsman for highway projects, and then computer programmer at the same engineering firm. 67.163.109.173 (talk) 00:51, 2 November 2012 (UTC)[reply]

I can't answer your question but this was the sort of thing that got printed in the Guinness Book of World Records. I seem to remember it was in the mid-six figures, with the NYSE more expensive than the AMEX. I suppose not necessarily extremely wealthy, possibly you could swing a loan, but you'd have to be successful.--Wehwalt (talk) 01:00, 2 November 2012 (UTC)[reply]
No, you do not need to be extremely wealthy to buy a seat on the Amex. You just have to pay the asking price. μηδείς (talk) 17:26, 2 November 2012 (UTC)[reply]
Which was in the ballpark of _____ in the 70s (just suppose 1975 if that's too wide an interval)? 20.137.2.50 (talk) 17:34, 2 November 2012 (UTC)[reply]
This said that a seat sold in December 1974 for $72,000. this (scroll to bottom, not a free article, I'm afraid) says that one sold for $60,000 on January 29, 1975 and one for $50,000 on February 13, 1975. this (also) says that a seat sold for $44,500 on August 7, 1975 and another for $52,000 on September 8. This said one sold for $35,000 on March 24, 1975. [40] This (same comment)] says on March 2, 1976 a seat sold for $64,000 and one had sold for $63,000 on March 1. That should give you enough to get going. Buyers and sellers are not identified in the sources. All of these except one are NY Times articles which has a pay archive, so you most likely would have to pay for full text. Other articles show the market peaked early in 1970 at $185,000 which was not approached again until late 1980. I would guess that the instability in prices was caused by the recession which was then going, and which hit New York City particularly hard, that was how Ford to City: Drop Dead got into the lexicon. But I suspect that 1975 was the bottom of the market. If he bought then, he showed his potential.--Wehwalt (talk) 17:57, 2 November 2012 (UTC)[reply]

Punjabi Sindhi Pashto Baloch Universities in Pakistan

Which universities of Pakistan offer lectures or courses in Punjabi only?; Which universities of Pakistan offer lectures or courses in Sindhi only?; Which universities of Pakistan offer lectures or courses in Pashto only?; Which universities of Pakistan offer lectures or courses in Baloch only? --Donmust90 (talk) 01:48, 2 November 2012 (UTC)[reply]

HA, you ask too much of Pakistanis. Yes a very obnoxious answer I know but Pakistanis respect the crown too much and are too impressed by it despite being enslaved by it. The English left but left English behind. So since its independence English (both in Pakistan and India) has always been considered the language of the elite. You are only taken seriously, you are only considered educated, and worthy of respect if you speak English (yes, I am exaggerating but very little). People are impressed very quickly even if you throw random English phrases here and there in a conversation without any hint of their true meaning. Hence, if an institution of any kind (especially educational or a business) wants to be taken seriously, it has to work in English. It may not be 100% English (English medium) but English must be there. The irony is that there are school in which you are actually punished for NOT speaking English. Even if someone heard you having a conversation in Urdu with your friend in the library, you get in trouble. If there was a university which taught only in Punjabi/Sindhi/Balochi/Pashto in Pakistan, people will laugh at it and no one would enroll in it. You are asking Pakistanis to love their culture and embrace their roots too much. But I will check just to make sure.70.58.0.141 (talk) 07:51, 2 November 2012 (UTC)[reply]

bengali punjabi gujarati Marathi Assamese Telugu Tamil Kannada Malayalam Oriya India Universities

Which universities of India offer lectures or courses in Bengali only? Which universities of India offer lectures or courses in Assamese only? Which universities of India offer lectures or courses in Oriya only? Which universities of India offer lectures or courses in Gujarati only? Which universities of India offer lectures or courses in Marathi only? Which universities of India offer lectures or courses in Punjabi only? Which universities of India offer lectures or courses in Telugu only? Which universities of India offer lectures or courses in Kannada only? Which universities of India offer lectures or courses in Tamil only? Which universities of India offer lectures or courses in Malayalam only? --Donmust90 (talk) 01:53, 2 November 2012 (UTC)[reply]

I would be extremely surprised to find a university in India that does not use English together with the local language(s) in its area. Roger (talk) 07:30, 2 November 2012 (UTC)[reply]

FEMA centralized displacement city

Why hasn't the govt ever developed a centralized ghost city to be used for disasters where all people can go until their hometowns are restored or at least safe to inhabit again?. It seems like the real issues with natural disasters is the weeks to months after with no food energy shelter in that location. In other words simple logistics. Other places have ample and even excess of those necessities but it is getting the two (people and necessities) together that is the real problem. Instead of trying to force the necessities into the ravaged locations why not evac the people to the necessities in essence waiting for them?68.83.98.40 (talk) 02:03, 2 November 2012 (UTC)[reply]

Have you considered how much such facilities would cost vs. how seldom they would be used? ←Baseball Bugs What's up, Doc? carrots02:07, 2 November 2012 (UTC)[reply]

The initial cost would be great but it could be "turned on/ off" when not in use. It could also use all the new tech with green energy and buildings. Prisoners could maintain it when not in use. Old cargo containers stacked. Seldom? It seems like once a year these days! Just another ounce of prevention vs pound of cure argument if you ask me. I think the cost of how it's currently done Is greater.68.83.98.40 (talk) 02:15, 2 November 2012 (UTC)[reply]

Where would you build it? ←Baseball Bugs What's up, Doc? carrots02:18, 2 November 2012 (UTC)[reply]

Well, without actually researching ....in general some place in the middle of us like Missouri Iowa Illinois ?68.83.98.40 (talk) 02:24, 2 November 2012 (UTC)[reply]

How much population capacity would it need? And how would you transport the people there? ←Baseball Bugs What's up, Doc? carrots02:26, 2 November 2012 (UTC)[reply]

More unresearched off the cuff answers but....1...million sounds like a good # to start or start with 1/4 of mil to start then add more throughout the years to better fit w budget. Plane, train automobile limo taxi helicopter.68.83.98.40 (talk) 02:31, 2 November 2012 (UTC)[reply]

What if it's something midwestern that needs escape from? Such as a massive tornado outbreak? Would you build the homes tornado-proof? ←Baseball Bugs What's up, Doc? carrots02:42, 2 November 2012 (UTC)[reply]

Im sure i would. It would only be logical.68.83.98.40 (talk) 02:45, 2 November 2012 (UTC)[reply]

Or do some research and find the least prone location to natural disasters, boom.68.83.98.40 (talk) 02:48, 2 November 2012 (UTC) Here is a map . I would say Wyoming Nebraska area. This map says PAC NW. But I'd rather stay a little farther away from the mt St. Helens super caldron.GeeBIGS (talk) 03:03, 2 November 2012 (UTC)[reply]

Mt St Helens is a mere pimple compared to the mega-volcano that underlies Yellowstone. Of course, if that blows up, we're all dead anyway. ←Baseball Bugs What's up, Doc? carrots04:03, 2 November 2012 (UTC)[reply]
Portable shelter and facilities (clinics, water purification, electricity generators, etc), are much more cost effective. Roger (talk) 07:34, 2 November 2012 (UTC)[reply]
Unihabited buildings very quickly become decrepit. Also the town would have no fresh food, no supermarket, not gaz station etc. as no one would want to keep running these services in a ghost town with no customers. --Lgriot (talk) 12:32, 2 November 2012 (UTC)[reply]
Wow. Such narrow-minded knit-picking negativity. So pick a better location(s)!Stacked Cargo ships dont get decrepit that quickly if properly sealed. Yeah, Roger, lets keep doing it like we are: bottlenecking rescue efforts and supply shipments into dangerous areas where people are disgruntled hungry tired and cold. Did you notice the lines for gaz? generators only work with gas in them! Hmm? Where should we put the mobile clinic? right between the downed power line and the leaky gas main.165.212.189.187 (talk) 12:56, 2 November 2012 (UTC)[reply]
It's pretty foolish to criticise realistic objections as 'nit-picking negativity'. Simply being phrased negatively doesn't make a thing bad - that's a clear fallacy. The simple truth is that moving people is more costly than moving other things, because people need a range of life-support while on the move. This is precisely because of the point you make about the durability of preserved food. So deporting up to a million people some 2-3 days journey across country would be much less efficient than moving them a minimum safe distance from the disaster area, and then bringing resources to them. And the point that Lgriot makes about how rapidly buildings decay is of crucial importance here: the towns of Pripyat, Varosha, Tomioka and Centralia show what becomes of cities that are not maintained. And the main way cities are maintained is by people living in them. Otherwise, wild animals find a way in sooner or later, the weather takes its toll, and a city becomes a ruin. In a country where many thousands are homeless day by day, and there is no effective federal assistance for them, why would the federal government blow billions of dollars on building an empty city that will begin rotting the day it is finished? AlexTiefling (talk) 13:11, 2 November 2012 (UTC)[reply]
Not sure about the others, but Centralia didn't decay by natural causes. The buildings have been very eagerly demolished, in part because there's a large company with mineral rights to whatever coal is left under the town, which can be accessed economically by strip mining if no residents remain. Wnt (talk) 21:04, 2 November 2012 (UTC)[reply]

Well, when you put it that way it sound exactly like something the govt would do!165.212.189.187 (talk) 19:59, 2 November 2012 (UTC)[reply]

I agree, it is not realistic. And second, if the government were to build such a city, how could it justify it remaining empty while people are homeless? (also how you empty it after something like Katrina in prep for the next one)--Wehwalt (talk) 20:29, 2 November 2012 (UTC)[reply]
Only an idiot would build a town like that, but idiots seem to have all the money nowadays. Of course, the unwashed refugees from some disaster would not be welcome. Wnt (talk) 21:07, 2 November 2012 (UTC)[reply]
That proposed fake city at least had some specific purpose in mind. I've been trying to hint to the OP that such a concept would be prohibitively expensive. It makes sense to keep the shelters as local as possible, to keep various costs minimal. ←Baseball Bugs What's up, Doc? carrots05:39, 3 November 2012 (UTC)[reply]

Continuation of the National Emergency with Respect to Weapons of Mass Destruction

http://www.whitehouse.gov/the-press-office/2012/11/01/letter-continuation-national-emergency-respect-weapons-mass-destruction

How many national emergencies are we in right now? Hcobb (talk) 02:22, 2 November 2012 (UTC)[reply]

Voting in hurricane ravaged areas

Why haven't we heard more about postponing the election? The arguments that I've heard say its not fair to one party or another, but what about it not being fair to those that can't vote, regardless of who they would vote for. I think it would add extreme insult to injury not to postpone the election. I mean gov christie was able to reschedule Halloween so that all the kids got their candy but te US can't reschedule election so that all citizens can exercise their right as an American? A travesty if you ask me.GeeBIGS (talk) 03:15, 2 November 2012 (UTC)[reply]

There is no real provision for moving election day. Congress would have to pass a new law and force it on the states and much lawsuit-ing would ensue.[41] Rmhermen (talk) 03:21, 2 November 2012 (UTC)[reply]
It would actually require a Constitutional amendment. Also, because of the Electoral College system where the votes are tallied separately in each state, if some states postpone their vote unilaterally, the election might well be decided without them. 24.23.196.85 (talk) 05:42, 2 November 2012 (UTC)[reply]
I think you're mistaken. When this was discussed a few days ago, none of the source suggest a constitutional amendment was likely to be needed. Nor does the source in what you're replying to. The constitution gives Congress the ability to set the date which they have done, in law, so they will need to pass a law to change the dates but a constitutional amendment would not be necessary. In fact even the date of electors meeting is only fixed in United States Code. However the date the president is to take office is fixed in the constitution so everything including the meeting of electors would need to be completed before then. You're right it's likely to be controversial given the problems and issues that would result and could lead to lawsuits if any of the dates are changed even if technically it could be done without a constitutional amendment. (Definitely the date for the electors to meet should be changed before the elections.) In any case, as all the sources suggest, the chance this is going to happen is close to zero so it's a moot point. (And the close it gets without something happening the less likely it is to happen.) Nil Einne (talk) 06:47, 2 November 2012 (UTC)[reply]

NJ and ny probably will postpone their presidential elections. GeeBIGS (talk) 03:23, 2 November 2012 (UTC)[reply]

Do you have a source for that? Given the answers to this question, and to an earlier one on a similar theme, it seems highly unlikely. AlexTiefling (talk) 12:55, 2 November 2012 (UTC)[reply]
According to this source, a postponement in any state is highly unlikely. Probably, the worst case is that the polling place for battered waterfront precincts is moved to less damaged places nearby and, in places without electricity, paper ballots are substituted for electronic ballots. Counting ballots might take longer in seriously affected places, but the places where the storm had its greatest impact are in securely Democratic states and thus unlikely to affect the electoral vote anyway. Marco polo (talk) 15:12, 2 November 2012 (UTC)[reply]

St. Patrick's Church, County Down

Where is the St. Patrick's Church, County Down exactly? Does it still exist? Is the Norwegian King Magnus Barefoot still buried there?

It seems to be the one in Downpatrick. Adam Bishop (talk) 10:53, 2 November 2012 (UTC)[reply]
Although St Patricks Church, Downpatrick was built in 1868, replacing an earlier church of the same name built in 1787[42]. It is the main Roman Catholic church of the city. Down Cathedral (Church of Ireland) is dedicated to the Holy Trinity and is on an ancient religious site reputed to be the burial place of St Patrick, so probably this is the church referred to. However, the Cathedral's website history page doesn't mention Magnus Barefoot. One of the other proposed burial sites of King Magnus is reputed to be under a mound outside the city; a memorial stone was unveiled there in 2003.[43]
This scholarly article; MEETING IN NORWAY: NORSE-GAELIC RELATIONS IN THE KINGDOM OF MAN AND THE ISLES, 1090-1270 by ROSEMARY POWER (p.18) says; "Magnusís name survived, among the Ulaid who buried him at the ancient monastic site of Downpatrick according to the Chronicle, and also in two Gaelic ballads (Christiansen 1931, 131-71, 401-06). The place where he died is described in Morkinskinna, Fagrskinna and more fully by Snorri, indicating a landscape of dykes and ditches, of scrubby copses on low hills, of freshwater and saltwater wetlands. It seems likely that this was not the landscape of Magnusís own day, but that developed later through land reclamation by the monks of the Cistercian abbey of Inch founded on the banks of the River Quoile in the 1180s, and described to the Norse writers by contemporaries who had seen it (Power 1994, 219-21)." Alansplodge (talk) 16:56, 2 November 2012 (UTC)[reply]

Liechtenstein

How many legitimate male-line descendants of Gundakar, Prince of Liechtenstein are there?--The Emperor's New Spy (talk) 05:03, 2 November 2012 (UTC)[reply]

According to the English Wikipedia article, he had two male sons that themselves were married. However, the German Wikipedia article only lists one son. So, I don't even know which is correct, but you have those two as leads. --Jayron32 13:46, 2 November 2012 (UTC)[reply]
Gundakar's article leads to [44] and [45], apparently the work of an amateur genealogist but highly detailed; if this can be trusted, I count 318 in all, 121 living, 66 male. —Tamfang (talk) 05:57, 3 November 2012 (UTC)[reply]

Non-denominational Christians and Bible Studies/Prayer Meetings

Here in the Philippines, we have several so-called "Born Again" churches (Non-denominational Christian churches), which frequently advertise a Bible study or a Prayer meeting. While Catholicism also has Bible studies and Prayer meetings, they aren't as common as they used to, and they aren't mandatory (although Bible studies at least are encouraged). I'm not saying that non-denominational churches give emphasis to such programs (being a Catholic, I'm not sure if members of such churches are required to attend such meetings), but it makes me wonder: why do non-Catholic churches seem to give a lot of importance to such meetings? Narutolovehinata5 tccsdnew 05:37, 2 November 2012 (UTC)[reply]

Because historically, protestantism was a reaction against the centralised, hierarchical, ceremony-driven approach of the Catholic church, and instead emphasised direct Bible study (see sola scriptura) and personal salvation (see five solae). And, from a memetic point of view, regular meetings are good for forming a sense of community and spreading the meme. --Stephan Schulz (talk) 07:36, 2 November 2012 (UTC)[reply]
The pattern holds true in the U.S. as well. In fact, "High Church" Protestants are seem likely to have such than Evangelical and Pentacostal churches. Rmhermen (talk) 13:01, 2 November 2012 (UTC)[reply]

Axis natural resources

During World War 2, what resources did Japan have that Germany didn't? 24.23.196.85 (talk) 05:49, 2 November 2012 (UTC)[reply]

Rubber for one. Clarityfiend (talk) 07:43, 2 November 2012 (UTC)[reply]
Silk; Germany's advanced chemical industry made parachutes from rayon and an early form of nylon because they couldn't get silk from Asia, although Italy had a modest silk industry. Alansplodge (talk) 18:54, 2 November 2012 (UTC)[reply]
And, if you don't mean to limit "resources" to natural resources, they also had a more substantial surface fleet, at least at the start. On the human resources front, they had a lot more people willing to die for the cause. StuRat (talk) 23:10, 2 November 2012 (UTC)[reply]
Thanks everyone, I was asking specifically about raw materials. I'm researching for a World War 2 historical/military thriller, and I'm trying to come up with some ideas for raw materials that could be used for a chemical weapon, for a secret "conventional" superweapon and/or for the German nuclear program, but would have to be imported from Japan (that's how the good guys will find out what exactly the Germans are making at their secret underground factory in the Hurtgen Forest -- they will have known that a U-boat has docked in St. Nazaire, and after having ambushed an eastbound train from that base, they will discover crates marked in Japanese and filled with the material). 24.23.196.85 (talk) 01:23, 3 November 2012 (UTC)[reply]
The Japanese experimented extensively with biological weapons (mainly on Chinese civilians), so your plot could be them having weaponized a superbug, and shipped it to Germany, where it's to be put on board V-2 rockets and launched against England. The Japanese characters could spell out "Unit 731". StuRat (talk) 06:51, 3 November 2012 (UTC)[reply]

Language attitudes in Islam

I know clearly that there is a consensus in mainstream Islam about non-Arabic languages. Simply speaking it is forbidden to pray aloud in non-Arabic and any translations of Quran are treated as corrupted interpretations.

But what are attitudes to non-Arabic prayer, Quran translations and to the religious use of of non-Arabic languages generally in Reform/Liberal and Quranic Islam?--Lüboslóv Yęzýkin (talk) 06:16, 2 November 2012 (UTC)[reply]

First let me just explain a few things. Prayers (if you want them to count I mean) must be said in Arabic yes but use of non-Arabic languages is not as strict as it may seem. Most Muslims don't have Arabic as their first language. But of course if you want to read the Quran, you need to know how to read Arabic so non-Arabic speakers learn arabic. That does something interesting. People learn how to "read" and pronounce arabic but they don't know what it means. So they can "read" the Quran but they have no idea what they are reading. So Qurans do include translations in whatever the native language may be. The point is to read the Quran and understand it but learning another language is hard but then you have to "read" it in Arabic for it to count. So people read it in Arabic, don't understand it, and then read the translation right underneath it.
The same thing happens for sermons (khutba) for the Friday prayers. The sermon is supposed to be in Arabic too. But chances are that the attendees don't know Arabic. So the speaker gives first a longer sermon in the native language, then quickly say it in Arabic at the end. In addition, if the congregation is mixed then just use whatever language is most common. So in the USA for example, the sermon is always in English followed by quick one in Arabic at the end right before the prayer begins.
The translations are considered corrupt only in the sense that a translation should not be regarded as the Quran itself. And this was done to prevent what had happened to other books where translations were presented as genuine word of God. And religious authorities just kept changing it to suit their needs and presenting it to others. I mean just try to count the different version of the Bible out there today accepted as the GENUINE word of God and not just a translation or a variation. In Islam it is perfectly okay to read a translation of the Quran and believe it. But then that also depends on the authority of the translator. The more well-known & educated he is, the more people believe his translation to be accurate. Nowadays, there are dedicated governing bodies (some are gigantic, even across international borders) who standardize the translation (and publish copies with their seals). So if you are a Sunni Muslim regardless of where you live on the planet, just walk into your local Saudi embassy and pick up a copy of the Quran (with Arabic and whatever language you can read). If you are Shia then you might have a problem with that translation so you would go elsewhere.
And now to actually answer your question, I haven't heard anything like what you are asking about. So I don't think that this is on the reform/liberal agenda. As far as I know, they all do the same and pray/read Quran in Arabic. I guess they think they have more important issues and then this kind of (minutae?) stuff can be taken care of IF anyone has a problem with it.70.58.0.141 (talk) 07:28, 2 November 2012 (UTC)[reply]
The exact same question was asked on the Language desk, please respond to that thread instead of starting a seperate discussion here - Lindert (talk) 08:46, 2 November 2012 (UTC)[reply]
These two desks have slightly different subjects and are followed by different people. So I do not see any problems here in two parallel themes.--Lüboslóv Yęzýkin (talk) 10:41, 2 November 2012 (UTC)[reply]
Many questions to the reference desk are applicable to multiple RD sections but questioners still properly follow the no-double-posting rule. You and your question are not entitled to special privileges to break that rule.A8875 (talk) 11:02, 2 November 2012 (UTC)[reply]

Thanks, 70.58.0.141, I have already known about these theological issues. But you still cannot loudly pray in your native language (let's say it's English). You cannot loudly pronounce all the time (not only at the begining of your conversion) an English translation of al-Fatiha while doing the salah. If you do you will be blamed as a heretic if not sent to prison (an example).

I do not consider the Quran being God's words, sorry. This is one of the greatest books, but it was not send from heaven, it was written down by people like all other books. I do not consider its outer Arabic form and the Arabic language itself as the "holly cow". Its translations are equal to the original in the sense of its meaning (like an English translation of "Iliad" are equal to the Ancient Greek original). And the Bible is not direct words of God, it is was only inspired by God, this is why it can be scientifically and theologically criticized without being persecuted by any law (like Shariah) or simply public opinion.

I only want to know who from modern liberal Islamic scholars or what Islamic movements think the same.--Lüboslóv Yęzýkin (talk) 11:45, 2 November 2012 (UTC)[reply]

I have no respect for either the Quran or the Bible, but very few people would consider English translations of the Iliad to be equivalent to the original. In particular, the Iliad is written in dactylic hexameter, and it's very difficult to write even one line of English poetry in that complex of a meter. It's also very difficult to translate Homer's meaning without losing the beauty of his poetry, changing the tone, or being incredibly hard to understand. I've yet to see a translation that reasonably balances all of these criteria. --140.180.252.244 (talk) 13:21, 2 November 2012 (UTC)[reply]
It was just a rough example which came first to my mind, you can substitute it for any other. Do Christians (except for obscurant medieval ones, maybe) think that singing in the church the Lord's prayer or Psalms in English, French etc. rather than in Hebrew, Greek or Latin is blasphemous? I believe they do not. I do not defend Christianity, I just want to know what Islamic movements do like Christianity in this aspect. --Lüboslóv Yęzýkin (talk) 21:10, 2 November 2012 (UTC)[reply]
In general the Western and Catholic position is that there is a sense behind the words which is what is their essence, separate from the form of the words, which is an accident. That is, the message outweighs the language in which it is conveyed. Understanding that requires you study Aristotle or Aristotelian philosophy, also known as scholasticism. μηδείς (talk) 22:19, 2 November 2012 (UTC)[reply]
Do you mean that modern Islam lacks such philosophy and is bound to outer formalism?--Lüboslóv Yęzýkin (talk) 09:44, 3 November 2012 (UTC)[reply]

Penal Servitude in the UK

I'm puzzled by the concept of penal servitude as it existed in England and Wales (and probably other parts of the UK before 1948). It was clearly somewhat similar to imprisonment with hard labour, but the two penalties were clearly not the same; for instance in the Criminal Justice Act 1948 section 1 (1) abolished penal servitude and section 1 (2) abolished imprisonment with had labour. Moreover in various Acts of Parliament the two punishments had been shown as alternatives. For instance the Offences Against the Person Act 1861 specified the penalty for Bigamy as "to be kept in Penal Servitude for any Term not exceeding Seven Years and not less than Three Years,—or to be imprisoned for any Term not exceeding Two Years, with or without Hard Labour." So could anyone please explain the differnece betweemn penal servitude and imprisonment with hard labour, as online sources (including our own articles) seem to imply that they were one and the same. rossb (talk) 09:27, 2 November 2012 (UTC)[reply]

Penal servitute is penal transportation to penal colonies for penal labour.
Sleigh (talk) 11:36, 2 November 2012 (UTC)[reply]
I don't think that's correct - see Penal_servitude#British_Empire, which says "The Penal Servitude Act 1853 ... substituted penal servitude for transportation". AndrewWTaylor (talk) 12:06, 2 November 2012 (UTC)[reply]
[ec] 'Hard labour' involved hard physical work; penal servitude did not necessarily (non-authoritative source: http://archiver.rootsweb.ancestry.com/th/read/DERBYSGEN/2010-05/1275317467 ). Also penal servitude did not require imprisonment as such; those performing penal servitude after transportation were as I recall often not imprisoned (having nowhere to run). HenryFlower 13:04, 2 November 2012 (UTC)[reply]
From the article, it looks like penal servitude was an umbrella term encompassing both hard labour and Non-punitive prison labour. Rojomoke (talk) 13:02, 2 November 2012 (UTC)[reply]

A few quotations

I saw these "inspirational" quotes in a trending post on Google Plus, and some of them sound anachronistic or stylistically wrong to me. Does anyone know if they are accurate or are misattributed or have been paraphrased? If they are accurate, what are the respective sources?--

  1. Shakespeare: "Never play with the feelings of others because you may win the game but the risk is that you will surely lose the person for a life time."
  2. Napoleon: "The world suffers a lot./ Not because of the violence of bad people./ But because of the silence of good people."
  3. Einstein: "I am thankful to all those who said NO to me / It's because of them I did it myself."
  4. Abraham Lincoln: "If friendship is your weakest point then you are the strongest person in the world."

PalaceGuard008 (Talk) 11:07, 2 November 2012 (UTC)[reply]

Number 2 is a bit like the famous saying 'All that is necessary for the triumph of evil is that good men do nothing', dubiously attributed to Edmund Burke. There's also 'Not only will we have to repent for the sins of bad people; but we also will have to repent for the appalling silence of good people', attributed to Martin Luther King. I've taken the liberty of moving your signature to the end of the list of quotations.AndrewWTaylor (talk) 12:00, 2 November 2012 (UTC)[reply]
Thanks Andrew, that's interesting. Both of these seem to be likely sources of inspiration for #2. --PalaceGuard008 (Talk) 11:19, 3 November 2012 (UTC)[reply]
I would suggest that all of these quotes are recent - probably less than five years old - and that none of them can be reliably attributed to anyone famous. All Google searches for them seem to turn up nothing but copies of this same email. I'm wondering who would be foolish enough to believe these attributions. AlexTiefling (talk) 12:39, 2 November 2012 (UTC)[reply]
Many people on Google+ at least, it seems. Pretty sad?
The other two quotes in the same post were from Mahatma Gandhi and A. P. J. Abdul Kalam whom, I admit, I had never heard of until I searched for him just now. So I'm guessing that this particular compilation at least originated in India. --PalaceGuard008 (Talk) 12:49, 2 November 2012 (UTC)[reply]
As Einstein once said: "be a skeptic, do no believe anything you read on the internet." OsmanRF34 (talk) 13:04, 2 November 2012 (UTC)[reply]
"Everything on quotes is legit" -Henry XII.203.112.82.2 (talk) 14:42, 2 November 2012 (UTC)[reply]
Einstein? Wasn't it Confucius who said that? -- Jack of Oz [Talk] 19:53, 2 November 2012 (UTC)[reply]
You are confuciued. OsmanRF34 (talk) 20:59, 2 November 2012 (UTC)[reply]
No, I'm just a visitor from the Forth Dementian.  :) -- Jack of Oz [Talk] 21:56, 2 November 2012 (UTC)[reply]
One of the purposes of a good liberal education is to teach why and how those supposed quotes are ridiculously bad English and obvious recent illiterate forgeries. μηδείς (talk) 22:15, 2 November 2012 (UTC)[reply]
Verily. One does not need to believe that "Shakespeare" was Shakespeare to know that quote #1 was not from either of their pens. It may perhaps be a modern day paraphrase of something one of them wrote, but then, it's no longer Shakespeare. Or even "Shakespeare". -- Jack of Oz [Talk] 22:51, 2 November 2012 (UTC)[reply]
The one allegedly from Napoleon is about as likely as this one:
Violence is justified in the service of mankind. -- Attila the Hun
Baseball Bugs What's up, Doc? carrots22:26, 2 November 2012 (UTC)[reply]
Or "I am totally committed to peace, and I'll kill anyone who stands in my way of achieving it". -- Jack of Oz [Talk] 22:53, 2 November 2012 (UTC)[reply]
Obviously they're quotes from Fred Shakespeare, Alphonse Napoleon, "Jersey Joe" Einstein (the "physicist of fisticuffs" and "professor of pugilism") and Abraham Lincoln Continental. What, no bluelinks? Clarityfiend (talk) 23:14, 2 November 2012 (UTC)[reply]

It's easy to dismiss them as made up, but I was hoping for anyone who has any thoughts on whether these might have been paraphrased or synthesised from real quotes - even better if by those people, along the lines of AndrewWTaylor posted above. (Though I doubt Napoleon would ever express a sentiment like that.) But - any thoughts? --PalaceGuard008 (Talk) 11:19, 3 November 2012 (UTC)[reply]

Shortest branch line

Today I had the pleasure of riding what is widely claimed to be the shortest branch line in Europe at 0.8 miles (1.3km). This raises the question of what is the shortest branch line in the world? All my Googling just comes back home to Stourbridge. I'm talking main line scheduled services only.--Shantavira|feed me 17:05, 2 November 2012 (UTC)[reply]

There's the 42nd Street Shuttle in Manhattan, which is the same length. It is one of the least pleasant train rides available. Much, much nicer is the Dinky, but it's a bit longer. μηδείς (talk) 17:17, 2 November 2012 (UTC)[reply]
That's a subway line, which doesn't meet Shantavira's criterion of a "main line" service. I suspected Japan might be able to rival the Stourbridge line, but the shortest branch line I could find was the Miyazaki Kūkō Line, which is marginally longer. Marco polo (talk) 18:22, 2 November 2012 (UTC)[reply]
Could you explain the difference? The princeton Junction line is a train but it requires one transfer from one train to the other at the station. That seems no different from a subway. μηδείς (talk) 18:57, 2 November 2012 (UTC)[reply]
The concept is really British, so I'm not 100% sure I get it. Maybe Shantavira can chime in. However, I think the main difference is that a main line branch needs to be physically connected to a national passenger rail system so that rolling stock can move from the branch line to the actual main line. That is the case for the Princeton Branch, which is connected by a switch to the main Northeast Corridor branch, even if passengers have to change trains at Princeton Junction. I don't think that the New York City subway system is integrated in the same way with the national rail system. Marco polo (talk) 20:13, 2 November 2012 (UTC)[reply]
Well, maybe that's the case for the Princeton Junction connection, but it abuts a concrete platform at the actual station, with no physical track connection there. Maybe there is a track connection elsewhere, but I don't know, having only ridden that side line once, and transferred by foot. μηδείς (talk) 22:09, 2 November 2012 (UTC)[reply]

Where is the free will?

Pick two imaginary women, who are identical and give birth at the same time. The first, motivated by oxytocin and other hormones, get attached to her baby and develops maternal instincts, the second, also motivated by hormones drops the baby in a the first garbage can. Should be punish the second and praise the first? What they are doing is just part of a kind of "biological program." How do modern day philosophers deal with this? Even just 100 years ago, philosophers were not aware of the existence of hormones and similar stuff, so I suppose the concept of "free will" didn't take enough bashing as it should. But nowadays, isn't it a discredited concept among educated researchers? Philoknow (talk) 18:16, 2 November 2012 (UTC)[reply]

Free will is a huge topic. I even have my own goofy ideas I've advanced at a previous discussion, but of course there's a wide range of viewpoints. [46][47] Wnt (talk) 18:21, 2 November 2012 (UTC)[reply]
OK, I don't expect to get an answer about what it free will and what it is not, but what about how academic philosophers integrated the discovery of hormones and neurotransmitters into their philosophy. It looks like more and more things are being explained in plain mechanical terms. How does philosophy deal with that? Philoknow (talk) 18:58, 2 November 2012 (UTC)[reply]
Don't know (or really care) how philosophers deal with your example... but I know how the police would deal with it. In most countries the woman who tossed her baby in the garbage would be arrested for child endangerment. Blueboar (talk) 19:05, 2 November 2012 (UTC)[reply]
That's true, but it says nothing about moral, just about how the law codified the norms. Philoknow (talk) 19:31, 2 November 2012 (UTC)[reply]
There is no such thing as free will. Our bodies just do things according to their nature, and we punish others' bodies when they do bad things. Lucky for us we are not the same thing as our bodies, so we can give our minds credit for things the body hasn't done, and when our bodies are punished it doesn't affect our minds at all.</irony> μηδείς (talk) 19:45, 2 November 2012 (UTC)[reply]
Of course there is. You chose to write the above statement, just as I am choosing to write this one. No one is compelling us to do so. ←Baseball Bugs What's up, Doc? carrots22:29, 2 November 2012 (UTC)[reply]
Is that just your private opinion? Where should this </irony> tag start? I wanted to know specifically how academic philosophy deal with this discovery of the physical mechanism of decisions. Philoknow (talk) 20:42, 2 November 2012 (UTC)[reply]
I don't know how philosophers would deal with it, but scientists would say that there isn't any reliable means of mind control. A person hyped up on methamphetamine or PCP or Hunter Thompson's legendary adrenochrome doesn't have to commit any crime (apart from drug law technicalities...). The most that a hormone or genetic factor can do is impart a predisposition - and usually, a very small one. Wnt (talk) 21:00, 2 November 2012 (UTC)[reply]
There isn't a reliable means of mind control right now, but that doesn't mean it's impossible. Ophiocordyceps unilateralis, a fungus, can mind control ants and make them climb up a plant before they die. You'll have a hard time convincing many scientists that there's a fundamental difference between the brain of humans and our animal relatives that makes the latter mind-controllable, but not the former. --140.180.252.244 (talk) 22:21, 2 November 2012 (UTC)[reply]
I think there's a difference between simple behavior and moral choices. You can douse someone in sneezing powder and make them sneeze or itching powder and make them scratch, but is that the same as denying them free will? Wnt (talk) 00:35, 3 November 2012 (UTC)[reply]
@ Philoknow. Basically, you are your body, and your body is able to act in complex ways that aren't strictly determined in a yes or no fashion by the immediate inputs of the environment in the way that plants and bacteria are. People who claim that they don't have free will because their bodies have forced them to do things are claiming that they are ghosts imprisoned in a body, which is a rather silly, if common delusion. See the links Wnt gave in the second post above, which give my arguments at some length and links to previous discussions. (Basically my point is, yes, you have free will and responsibility for your own actions, but most people define free will in a rather ridiculous fashion.) μηδείς (talk) 22:03, 2 November 2012 (UTC)[reply]
Considering any possible mechanism allowing free will has never been discovered, it makes little sense to dismiss the idea there is no free will. Also to claim that certain organisms' actions are determined by their environment yet humans are not makes little sense without going into religious beliefs like souls or other ideas that somehow separate humans from other organisms.
That said, to answer the OP, see Hard determinism#Implications for ethics. There are some philosophers, who, like you, question whether one should be held responsible for their actions in a world without free will. In a determined world without free will a person is born with as much choice for their future actions as they do for their skin colour, none. The real world consequences are complicated, but I guess we would/should continue to punish immoral behaviour (even if any punishment is immoral) just to lower the total number of immoral actions - punishment deters immoral behaviour.50.101.137.171 (talk) 00:54, 3 November 2012 (UTC)[reply]
You'll have to define wht you mean by free will before you can complain there's no mechanism for it. We do know people make choices and can act independent of outside inputs. Nothing makes you get out of the bed in the morning other than you. μηδείς (talk) 04:15, 3 November 2012 (UTC)[reply]
The question is not really about free will, it is about the morality of punishment. They are often taken to be the same thing, but that is a conclusion that needs to be demonstrated, not just taken as obvious. Looie496 (talk) 22:30, 2 November 2012 (UTC)[reply]
No, the question is about free will. That's why it's "where is the free will?" Where is the fair punishment is just a logical secondary question to the first one. Philoknow (talk) 23:46, 2 November 2012 (UTC)[reply]
In your example, the first woman chose to be a loving mother and the second one chose not to be. It ain't rocket science. As to why they chose to do things, there's a lot more to it than hormones. There's also upbringing and consequence development of a good conscience. ←Baseball Bugs What's up, Doc? carrots05:34, 3 November 2012 (UTC)[reply]
Here are the results of plugging the search string +"free will" +hormones into google scholar - this should give you some idea of how current philosophy approaches the problem. Taknaran (talk) 00:28, 3 November 2012 (UTC)[reply]
Neuroscience of free will refers to a few contemporary philosophers dealing with some of the questions of compatibility too. ---Sluzzelin talk 00:58, 3 November 2012 (UTC)[reply]
Free will occurred for the first ten minutes following the big bang, but you are still experiencing it.[48] Paum89 (talk) 07:56, 3 November 2012 (UTC)[reply]

Are there any European Far-Right Parties that support Iran?

Are there any European Far-Right Political Parties that support Iran? --Gary123 (talk) 19:51, 2 November 2012 (UTC)[reply]

Yes, the Austrian Freiheitliche Partei Österreichs (FPÖ) and the German NPD. Both are also against Islam in Europe, but recognize Iran as a force against Israel and America. Philoknow (talk) 19:59, 2 November 2012 (UTC)[reply]
References that support those claims would be helpful (and funny). μηδείς (talk) 21:56, 2 November 2012 (UTC)[reply]
For a link in German: [[49]].Philoknow (talk) 22:30, 2 November 2012 (UTC)[reply]
As it concerns the FPÖ this is definitely wrong. To the disgust of many of its voters, the party's current Middle East policy is actually pro-Israeli. --Michael Fleischhacker (talk) 22:14, 2 November 2012 (UTC)[reply]
Nope, nothing is wrong. FPÖ is able to unity contradictory opinions. In 2000, Israel declared Austrian politician Jörg Haider persona non grata following the Austrian legislative election of 1999, due to his anti-antisemitic speeches, but they are also able to praise the perceived fight of Islam against foreign interference. Philoknow (talk) 22:30, 2 November 2012 (UTC)[reply]
I'm missing something. Why would Israel want to keep him out for anti-antisemitic speeches? --Trovatore (talk) 23:15, 2 November 2012 (UTC)[reply]
Sorry, I've already corrected it. Philoknow (talk) 23:43, 2 November 2012 (UTC)[reply]
The National Front had a stint in the 1980s went it began to praise Gaddafi and Khomenei, as part of its turn away from classical fascism to Third Position. They are all history now, however. --Soman (talk) 22:18, 2 November 2012 (UTC)[reply]

Animism

Why is it that, other than Japan, every society with an animist religion has failed to advance as much as other areas? (Sub-Saharan Africa, Native Americans, etc.) --168.7.234.172 (talk) 20:12, 2 November 2012 (UTC)[reply]

Define "advance." There is a case to be made that what you imply is advancement could be seen as regression: Neither of the cultures who you mention ever came to the verge of engaging in nuclear holocaust, and those cultures who remain animist often see the rest of the world just that way. — TransporterMan (TALK) 20:49, 2 November 2012 (UTC)[reply]
Native Americans had pretty "advanced" societies by 15th century standards. They didn't advance beyond that because they got wiped out by germs and conquest, but their religions didn't have much to do with that. Some animist societies, such as the Mongols, managed to put under the yoke plenty of non-animist societies, as well. Certain religions strains historically have seemed to confer advantages to their societies (I am not positing anything divine in this — they just result in different sorts of mass behavior), but I'm not sure that one can so easily conclude that it is a question of monotheism vs. polytheism vs. animism in a strict way. Historically the monotheistic religions have been the ones that have urged their followers to convert or murder others, which drove several centuries of world history and development (for better or ill). --Mr.98 (talk) 23:06, 2 November 2012 (UTC)[reply]
"Pretty advanced societies by 15th century standards"? What are you talking about? They didn't have sailing ships, metallurgy to speak of, writing, mathematics. As far as I can see they can't compete with Europe, North Africa, China, or Japan, during the same time period. --Trovatore (talk) 23:24, 2 November 2012 (UTC)[reply]
Some did have writing, but I agree that technologically they were quite behind a number of Old World civilizations. - Lindert (talk) 23:33, 2 November 2012 (UTC)[reply]
Some people think that "native american" is equal to US native American, but it actually refers to all indigenous peoples of the Americas, South and North, and some indeed were civilized. Philoknow (talk) 00:20, 3 November 2012 (UTC)[reply]
Some of the civilizations in modern Mexico and points south had a little more technology than their northern counterparts, but they still didn't have seafaring, metallurgy beyond primarily decorative uses, firearms, or mathematics beyond simple arithmetic. --Trovatore (talk) 00:56, 3 November 2012 (UTC)[reply]
You are right that technologically they were less developed, however, since they were aborted by an external force, we cannot conclude that there were something internal, like their animist religion, limiting their development. We don't know what would have happen. Philoknow (talk) 01:28, 3 November 2012 (UTC)[reply]
I wasn't claiming it had anything at all to do with their religion. I just don't agree with Mr.98 that they had "advanced societies by 15th century standards". --Trovatore (talk) 02:01, 3 November 2012 (UTC)[reply]
You have to define "advanced". It's been noted that Roanoke had a problem with settlers "going native" [50], and Benjamin Franklin himself wrote: "no European who has tasted Savage Life can afterwards bear to live in our societies". So yes, the monotheistic societies had a clear advantage in technology, but many who encountered these "primitive" societies concluded they had a superior way of life. So perhaps they were more advanced in the individual-happiness department. Someguy1221 (talk) 04:41, 3 November 2012 (UTC)[reply]

168.7.234.172 -- "Animism" is a vague catch-all term (maybe a little old-fashioned by now) basically referring to religions of mostly non-literate peoples without any recognizable affiliation with a "major" or "world" religion. So the religion of a civilization or empire with major world influence would pretty much by definition be unlikely to be called "animist"... AnonMoos (talk) 00:30, 3 November 2012 (UTC)[reply]

Ancestry of Robert Browning

I am looking for the ancestry of poet Robert Browning, especially in his Browning line, but can't find anything beyond his father (also Robert) and some mention of his grandparents. Is there any compiled genealogy going back many generations? Edison (talk) 22:08, 2 November 2012 (UTC)[reply]

Have a look at this, or this, or this. Apparently Frederick James Furnivall (a friend of Browning) published a biographical essay in The Academy for 12 Apr 1902, but I haven't found an on-line edition yet. Zoonoses (talk) 05:14, 3 November 2012 (UTC)[reply]

Robot president

Simple hypothetical. Say in the future, robotics gets to a phase where they are sentient enough to take on certain tasks, analyze the needs of citizens, and wanted to become the president of the United States (provided it was manufactured there.) Constitutionally, could the robot run for office? 64.229.183.165 (talk) 22:26, 2 November 2012 (UTC)[reply]

No possible way of knowing, but I'd suggest that governments all over the world would be forced to define "human being" well before such a possibility was ever realised, and to make it clear that eligibility for certain offices did not extend to other beings. Of course, that would open up a huge can of worms: such as, when does an unborn foetus become a human being? -- Jack of Oz [Talk] 22:47, 2 November 2012 (UTC)[reply]
Given that the question is about the US, and that the US already requires a minimum age for presidential candidates, and that an unborn fetus will never reach that age (35) by definition, your last question would be mercifully academic. But the only future I can predict is one where this question will be hatted. Orange Suede Sofa (talk) 22:55, 2 November 2012 (UTC)[reply]
For the United States, see Article_Two_of_the_United_States_Constitution#Clause_5:_Qualifications_for_office for the requirements. It strikes me that making such a clause viable for a robot AI would require a very liberal interpretation of the Constitution. (Plus, it would have to be a 35-year-old robot — it'd surely not be the most up to date model at that point!) You would have to argue that the robot was 1. a person, 2. natural born, and 3. a citizen. Not one of those things is a distinction currently given to non-living objects. Long before any Constitutional question about robots running for President would be the clarification of the legal code for robots or AIs of significant advancement to qualify for any one of those distinctions. --Mr.98 (talk) 23:00, 2 November 2012 (UTC)[reply]
But for a few votes in Florida, we might have had one starting in 2001. --Trovatore (talk) 23:06, 2 November 2012 (UTC) [reply]
Yeh, Al Gort. ←Baseball Bugs What's up, Doc? carrots02:09, 3 November 2012 (UTC)[reply]
For many social procedures, you need personhood. There is nothing new to be defined. Non-humans are not allowed to obtain it. And for many other procedures you need legal personality, which a robot could be, if a person creates a company that owes the robot and put a straw man director at charge. But a legal personality cannot be president, so you won't go that far with your autonomous robot. Philoknow (talk) 23:12, 2 November 2012 (UTC)[reply]
Is there any court precedence, US or otherwise, that explicitly established that only humans are people? Perhaps a case regarding animal rights? Someguy1221 (talk) 23:49, 2 November 2012 (UTC)[reply]
I would be careful in this context to say persons rather than people. People is not quite a plural of person; it has other nuances. --Trovatore (talk) 23:52, 2 November 2012 (UTC)[reply]
Exactly, it's all about being a legal person. There are certainly many laws and court precedence which clearly imply a difference between humans and non-humans. If you kill a dog, you won't be charged with murder. Birth certificates cannot be issued at will either and many legal procedures depend on a birth certificate. Philoknow (talk) 00:03, 3 November 2012 (UTC)[reply]
There have been some small efforts in Europe, actually. Check out Great ape personhood. (There is also, conversely, corporate personhood.) --Mr.98 (talk) 02:45, 3 November 2012 (UTC)[reply]
In Alexis Gilliland's Rosinante trilogy, sentient machines are legally corporations ... —Tamfang (talk) 06:08, 3 November 2012 (UTC)[reply]
The Bicentennial Man didn't run for president, but perhaps some other robot following in his footsteps could have done so. 24.23.145.28 (talk) 01:00, 3 November 2012 (UTC)[reply]
Robots at that level don't run for president because accepting power over humans conflicts with the first law. A robot would never put itself in a position where it might be necessary to order the deaths of humans. Paum89 (talk) 05:55, 3 November 2012 (UTC)[reply]
In The Simulacra, the president was an Android (robot). Mitch Ames (talk) 08:34, 3 November 2012 (UTC)[reply]

Stays of executions in the United States

The article on Ted Bundy reads as follow Minutes before his execution, Hagmaier queried Bundy about unsolved homicides in New Jersey, Illinois, Vermont (the Curran case), Texas, and Miami, Florida. Bundy denied involvement in any of them.

My question is, if Bundy claimed responsibility in some of such cases, could his execution have been halted? — Preceding unsigned comment added by 190.178.143.191 (talk) 23:25, 2 November 2012 (UTC)[reply]

Not that it's the norm to answer a question with a question, but my question is "Why did they wait till so late to ask him?". -- Jack of Oz [Talk] 23:34, 2 November 2012 (UTC)[reply]
Guessing: Hagmaier (whoever he might be; I haven't looked it up) probably figured that Bundy might be more forthcoming, having nothing more to lose (and possibly wanting to clear his conscience). To the original poster, I would ask, why would you think that would delay the execution? --Trovatore (talk) 23:37, 2 November 2012 (UTC)[reply]
I'm guessing they meant so that Ted could have time to give the details, burial sites, etc. Somehow I doubt that such a delay would be granted, as it's obvious he could have given the details earlier, without a delay. StuRat (talk) 23:41, 2 November 2012 (UTC)[reply]
An execution can be halted at any time by the Governor or a judge. They are not legally forbidden from halting an execution after previously declining to do so, they just usually don't. Someguy1221 (talk) 23:46, 2 November 2012 (UTC)[reply]
Actually, they probably asked him several times before, that was just another chance of obtaining the information. — Preceding unsigned comment added by Philoknow (talkcontribs) 00:17, 3 November 2012 (UTC)[reply]
OK, thanks, But if he had answered in the affirmative, and he said he was the only one who knew where the bodies were, and if they executed him today they'd never find out, wouldn't that have been enough reason to call a halt? One could argue that he could have been lying in order to postpone his death, but then, why ask him in the first place? Knowing that he committed the murders but knowing no other details, would almost be worse than not knowing at all. -- Jack of Oz [Talk] 00:40, 3 November 2012 (UTC)[reply]
Yeah, he could have said: "wait, wait, don't execute me, I still have something to say", get a postponement and the next time the same thing all over again. Ted Bundy got information for a handful of postponements. Philoknow (talk) 01:26, 3 November 2012 (UTC)[reply]
Sure. So again I ask: Why ask him the question? It couldn't just have been for him to shrive his conscience. Could it? -- Jack of Oz [Talk] 01:44, 3 November 2012 (UTC)[reply]
Conventional wisdom at the time was that he was trying to stave off the execution by suddenly deciding to spill some details. The authorities didn't buy, and he fry. ←Baseball Bugs What's up, Doc? carrots02:07, 3 November 2012 (UTC)[reply]
That's not really responsive. Jack wasn't asking for an explanation of Bundy's actions, but of Hagmaier's. It seems perfectly plausible to me that Hagmaier just wanted to clarify the facts, and figured this was the moment he might get the truth. --Trovatore (talk) 04:06, 3 November 2012 (UTC)[reply]
The OP's actual question was whether the execution could be halted. And since the possibility was debated at the time, it's clear that it could have been halted if someone in authority had decided to do so. And as I said just above your snippy and useless comment, those who were in position to make that decision concluded that he was just stalling, so they decided to let the execution go on as planned. ←Baseball Bugs What's up, Doc? carrots05:26, 3 November 2012 (UTC)[reply]
By indentation, you were responding to Jack, but your comment was not responsive to Jack. If you don't like that being commented on, then either indent correctly, or respond to the people you're responding to. --Trovatore (talk) 06:11, 3 November 2012 (UTC)[reply]
You might want to run that question past an attorney instead of eliciting speculation from people who claim to not give legal advice. Paum89 (talk) 05:56, 3 November 2012 (UTC)[reply]
When did "legal advice" become a euphemism for "anything even remotely related to the law"? Orange Suede Sofa (talk) 06:08, 3 November 2012 (UTC)[reply]

November 3

Turin Cannon Pyramid Construction Details.

To whom it may concern / the reader. Hello. Will attempt to keep this a brief as possible. Just to show that One's homework has been done, as well as to provide a contextual picture, here follows a brief string of details that will lead in to the question, followed by an unconfirmed answer. The last two decades have seen an Intellectual war being fought by the "Scientific Archaeologists" on the one side, and the "Pseudo-Archaeologists" on the other. The former group claims that the Great Pyramid was constructed by the Pharaoh "Khufu" around 2500 BC, while the latter group highlights certain design features that seem to indicate a much earlier date of construction. The former groups claims are based on the evidence provided by the "Turin Cannon", which makes reference to Khufu's construction work on the Great Pyramid, taking around 25 ish years. Unfortunately this would involve 1 block of stone being quarried, transported and moved in to place every 5 minutes, day and night. One's question is: "What's the probability, that the details highlighted in the Turin Cannon, have been mistranslated or perhaps misinterpreted?" As in, is it possible that the details provided by the Cannon are more by way of a reference to Reconstruction work or Renovation, as opposed to their original manufacture? If, as the "Pseudo Archaeologists" suggest, the Pyramids were constructed around 10,500 BC, then during their 8,000 year temporal journey to Khufu's reign, they would undoubtedly have experienced a similar number of earthquakes as they have on their trip from Khufu's time to present day. As a result of these earthquakes, back in Khufu's day, they may well have looked a lot like they do now, in need of a good face lift. Perhaps Khufu looked from his palace balcony one morning and realized that if they were left as they were, then one day they wouldn't be there any more. So, he instigates their Renovation in order that future generations and even Civilizations, could stand in their shadow and marvel at their splendor. This Reconstruction Hypothesis goes well to explain why Khufu's name appears but once inside the Great pyramid, (by way of saying "Khufu was here, but I didn't build it") and not painted on every wall and carved in to every block of stone in order to leave future Intellects in no doubt whatsoever, as to Who did build the things. 25 odd years to replace the outer casing stones, is a much easier pill to swallow than the one currently being offered. Have reached One's limits with regard to further investigation and/or confirmation of this theory and so am handing over to someone who may be in a better position to pass it on to somebody that has longer arms. Thank you for your time.

Yours sincerely, Jaaar. (Just Another Anonymous Amateur Researcher)Jaaar. (talk) 00:36, 3 November 2012 (UTC)[reply]

Placing a stone in place every 5 minutes isn't impossible, if you assume many teams were working in parallel. At least on the lower levels, this would have been easy to do. StuRat (talk) 01:13, 3 November 2012 (UTC)[reply]
There is no intellectual war. There are actual archaeologists doing actual archaeological research, and there are crazy pseudoscientists screaming from the sidelines. The actual archaeologists don't even argue about one stone every five minutes being too fast. In fact, many have argued this is too slow, and could have been done once every two minutes. See Egyptian pyramid construction techniques. As for the idea of resurfacing, not all stone is made equal. I have a feeling it would be quite obvious if the surface stones were quarried 8,000 years after the underlying ones. Just look at something more modern. The Washington Monument was surfaced with marble quarried from two different locations just 20 years and a few miles apart, but you can see the difference (see File:Washington Monument (from Jefferson Memorial) IMG 4731.JPG). Yes, it's a different stone, but that was actually the closest the builders could get to making the new stones look like the old ones. I doubt that Khufu could have done such a spectacular job that modern archaeologists couldn't spot an 8,000 year difference. Someguy1221 (talk) 02:13, 3 November 2012 (UTC)[reply]
It looks like the current estimate for the number of blocks is about 600,000. If it took 20 years to build, that averages out to about 3 blocks per hour. Given that the great majority of stones were in the interior and didn't have to be matched perfectly, that doesn't seem impossible. Looie496 (talk) 05:03, 3 November 2012 (UTC)[reply]
By the way, one (pronoun) does not take a capital. Nor is it a synonym for "I". Marnanel (talk) 10:48, 3 November 2012 (UTC)[reply]

American School of Design, New York

I am writing an article on illustrator Adrienne Adams and my sources (http://theartofchildrenspicturebooks.blogspot.co.uk/2011/05/going-barefoot.html & http://www.findagrave.com/cgi-bin/fg.cgi?page=gr&GRid=58180908) say she attended the American School of Design in New York, is this likely to be the National Academy of Design ? Thanks GrahamHardy (talk) 09:05, 3 November 2012 (UTC)[reply]

Proof of chastity

On our article about the Dominican Order, there is a picture of Aquinas with the caption:

St. Thomas Aquinas (1225-1274), considered by the Catholic Church to be its greatest medieval theologian, is girded by angels with a mystical belt of purity after his proof of chastity.

What is a proof of chastity, in this context? The link leads only to Chastity, which doesn't explain. Marnanel (talk) 10:41, 3 November 2012 (UTC)[reply]

Why do campaigns bother with internal polls?

Every major political campaign (Presidential, Senate and House AFAIK) has internal polling, polls which they pay for and get to choose whether to release. But why bother? If there's one thing that 538 and so on show us is that you can get all the information you need from publicly available polls which are common. Given the campaigns, especially the smaller ones need to spend every dollar they can on advertising and so on, why do they persist in wasting money on frequent internal polls, even when the public pollsters are providing the information for free (from their perspective)? They aren't even very accurate from what I've seen. 86.163.43.112 (talk) 11:26, 3 November 2012 (UTC)[reply]